NSG 321 NCLEX Practice Questions (CH. 18, 21, 22, 24-26, 30, 33, 38-50, 55, 60-64)

Lakukan tugas rumah & ujian kamu dengan baik sekarang menggunakan Quizwiz!

19. A patient with a fractured tibia accompanied by extensive soft tissue damage initially has a splint applied and held in place with an elastic bandage. What early sign should alert the nurse that the patient is developing compartment syndrome? a. Paralysis of the toes b. Absence of peripheral pulses c. Distal pain unrelieved by opioid analgesics d. Skin over the injury site is blanched when the bandage is removed

19. c. Pain that is distal to the injury and is unrelieved by opioid analgesics is the earliest sign of compartment syndrome. Paralysis and absence of peripheral pulses will eventually occur if it is not treated but these are late signs that often appear after permanent damage has occurred. The overlying skin may appear normal because the surface vessels are not occluded.

21. A 68-year-old man is admitted to the emergency department with multiple blunt trauma following a one-vehicle car accident. He is restless; disoriented to person, place, and time; and agitated. He resists attempts at examination and calls out the name "Janice." Why should the nurse suspect delirium rather than dementia in this patient? a. The fact that he wouldn't have been allowed to drive if he had dementia b. His hyperactive behavior, which differentiates his condition from the hypoactive behavior of dementia c. The report of emergency personnel that he was noncommunicative when they arrived at the accident scene d. The report of his family that although he has heart disease and is "very hard of hearing," this behavior is unlike him

21. d. Delirium is an acute problem that usually has a rapid onset in response to a precipitating event, especially when the patient has underlying health problems, such as heart disease and sensory limitations. In the absence of prior cognitive impairment, a sudden onset of confusion, disorientation, and agitation is usually delirium. Delirium may manifest with both hypoactive and hyperactive symptoms.

21. When providing discharge instructions to a patient who had a subtotal thyroidectomy for hyperthyroidism, what should the nurse teach the patient? a. Never miss a daily dose of thyroid replacement therapy. b. Avoid regular exercise until thyroid function is normalized. c. Use warm saltwater gargles several times a day to relieve throat pain. d. Substantially reduce caloric intake compared to what was eaten before surgery.

21. d. With the decrease in thyroid hormone postoperatively, calories need to be reduced substantially to prevent weight gain. When a patient has had a subtotal thyroidectomy, thyroid replacement therapy is not given because exogenous hormone inhibits pituitary production of TSH and delays or prevents the restoration of thyroid tissue regeneration. Regular exercise stimulates the thyroid gland and is encouraged. Saltwater gargles are used for dryness and irritation of the mouth and throat following radioactive iodine therapy.

22. The woman with osteoporosis slipped on the ice and now her wrist hurts. If there is a fracture, what type of fracture is expected? a. Dislocation b. Open fracture c. Colles' fracture d. Incomplete fracture

22. c. A Colles' fracture most often occurs in patients over 50 years of age with osteoporosis and frequently when the patient attempts to break a fall with an outstretched arm and hand. Dislocation is the complete separation of articular surfaces of the joint caused by a ligament injury. Open fracture is when there is communication with the external environment. A fracture is incomplete if only part of the bone shaft is fractured and the bone is still in one piece.

What should be included in the management of a patient with delirium? a. The use of restraints to protect the patient from injury b. The use of short-acting benzodiazepines to sedate the patient c. Identification and treatment of underlying causes when possible d. Administration of high doses of an antipsychotic drug such as haloperidol (Haldol)

22. c. Care of the patient with delirium is focused on identifying and eliminating precipitating factors if possible. Treatment of underlying medical conditions, changing environmental conditions, and discontinuing medications that induce delirium are important. Drug therapy is reserved for those patients with severe agitation because the drugs themselves may worsen delirium.

22. What is a cause of primary hypothyroidism in adults? a. Malignant or benign thyroid nodules b. Surgical removal or failure of the pituitary gland c. Surgical removal or radiation of the thyroid gland d. Autoimmune-induced atrophy of the thyroid gland

22. d. Both Graves' disease and Hashimoto's thyroiditis are autoimmune disorders that eventually destroy the thyroid gland, leading to primary hypothyroidism. Thyroid tumors most often result in hyperthyroidism. Secondary hypothyroidism occurs as a result of pituitary failure and iatrogenic hypothyroidism results from thyroidectomy or radiation of the thyroid gland.

22. A lymph node biopsy is most often performed to diagnose a. leukemia. b. cause of lymphedema. c. hemorrhagic tendencies. d. neoplastic cells in lymph nodes.

22. d. Lymph node biopsy is usually done to determine whether malignant cells are present in lymph nodes and can be used to diagnose lymphomas as well as metastatic spread from any malignant tumor in the body. Leukemias may infiltrate lymph nodes but biopsy of the nodes is more commonly used to detect any type of neoplastic cells.

23. The nurse has identified the nursing diagnosis of fatigue for a patient who is hypothyroid. What should the nurse do while caring for this patient? a. Monitor for changes in orientation, cognition, and behavior. b. Monitor for vital signs and cardiac rhythm response to activity. c. Monitor bowel movement frequency, consistency, shape, volume, and color. d. Assist in developing well-balanced meal plans consistent with level of energy expenditure.

23. b. Cardiorespiratory response to activity is important to monitor in this patient to determine the effect of activities and plan activity increases. Monitoring changes in orientation, cognition, and behavior are interventions for impaired memory. Monitoring bowels is needed to plan care for the patient with constipation. Assisting with meal planning will help the patient with imbalanced nutrition: more than body requirements to lose weight if needed.

23. The patient's laboratory results show a marked decrease in RBCs, WBCs, and platelets. What term should the nurse use when reporting the results to the physician? a. Hemolysis b. Leukopenia c. Pancytopenia d. Thrombocytosis

23. c. Pancytopenia is decreased RBCs, WBCs, and platelets. Hemolysis is RBC destruction. Leukopenia is WBC <4000/μL. Thrombocytosis is increased platelets and thrombocytopenia is decreased platelets.

23. When caring for a patient in the severe stage of AD, what diversion or distraction activities would be appropriate? a. Watching TV b. Playing games c. Books to read d. Mobiles or dangling ribbons

23. d. In the severe stage of AD, the patient is at a developmental level of 15 months or less; therefore appropriate distractions would be infant toys. Watching TV and playing games are more appropriate in the mild stage. Books to read would need to be at developmentally appropriate levels to be used as a diversion.

24. Molecular cytogenetics and gene analysis may be done to diagnose, stage, and help to determine treatment options for various hematologic disorders. Which sites are preferred to obtain the sample for this testing (select all that apply)? a. Skin sample b. Lymph node c. Bone marrow d. Arterial blood e. Inner cheek mucosa

24. b, c. Bone marrow and lymph node biopsies are preferred methods to obtain the sample for gene analysis. If a large number of abnormal cells are circulating in the blood, peripheral blood may be used. The other options will not provide the desired information.

24. In a patient with a stable vertebral fracture, what should the nurse teach the patient to do? a. Remain on bed rest until the pain is gone. b. Logroll to keep the spine straight when turning. c. How to use bone cement to correct the problem. d. Take as much analgesic as needed to relieve the pain.

24. b. The spine should be kept straight by turning the shoulders and hips together (logrolling). This keeps the spine in good alignment until union has been accomplished. Bed rest may be required for a short time but not until the pain is gone. Analgesics should be taken only as ordered. If they do not relieve the pain, the health care provider should be notified. Bone cement is used by the surgeon to stabilize vertebral compression fractures.

24. Priority Decision: When replacement therapy is started for a patient with long-standing hypothyroidism, what is most important for the nurse to monitor the patient for? a. Insomnia b. Weight loss c. Nervousness d. Dysrhythmias

24. d. All these manifestations may occur with treatment of hypothyroidism. However, as a result of the effects of hypothyroidism on the cardiovascular system, when thyroid replacement therapy is started myocardial oxygen consumption is increased and the resultant oxygen demand may cause angina, cardiac dysrhythmias, and heart failure, so monitoring for dysrhythmias is most important.

When is a fat embolism most likely to occur? a. 24 to 48 hours following a fractured tibia b. 36 to 72 hours following a skull fracture c. 4 to 5 days following a fractured femur d. 5 to 6 days following a pelvic fracture

25. a. Initial manifestations of fat embolism usually occur 24 to 48 hours after injury and are associated with fractures of long bones and multiple fractures related to pelvic injuries, including fractures of the femur, tibia, ribs, and pelvis.

25. A patient with hypothyroidism is treated with levothyroxine (Synthroid). What should the nurse include when teaching the patient about this therapy? a. Explain that alternate-day dosage may be used if side effects occur. b. Provide written instruction for all information related to the drug therapy. c. Assure the patient that a return to normal function will occur with replacement therapy. d. Inform the patient that the drug must be taken until the hormone balance is reestablished.

25. b. Because of the mental sluggishness, inattentiveness, and memory loss that occur with hypothyroidism, it is important to provide written instructions and repeat information when teaching the patient. Replacement therapy must be taken for life and alternate-day dosing is not therapeutic. Although most patients return to a normal state with treatment, cardiovascular conditions and psychoses may persist.

What nursing responsibilities are done to obtain a clean-catch urine specimen from a patient (select all that apply)? a. Use sterile container. b. Must start the test with full bladder. c. Insert catheter immediately after voiding. d. Have the patient void, stop, and void in container. e. Have the patient clean the meatus before voiding.

26. a, d, e. A clean-catch urine specimen is obtained in a sterile container after cleaning the meatus. The patient will void a small amount in the toilet, stop, and then void in the container to catch the urine midstream. The first morning specimen is best for a urinalysis. A full bladder is necessary for a urine flow study. A urinary catheter is inserted immediately after voiding to assess residual urine.

26. The nurse suspects a fat embolism rather than a pulmonary embolism from a venous thrombosis when the patient with a fracture develops what? a. Tachycardia and dyspnea b. A sudden onset of chest pain c. Petechiae around the neck and upper chest d. Electrocardiographic (ECG) changes and decreased PaO2

26. c. Patients with fractures are at risk for both fat embolism and pulmonary embolism from venous thromboembolism but there is a difference in the time of occurrence, with fat embolism occurring shortly after the injury and thrombotic embolism occurring several days after immobilization. They both may cause pulmonary symptoms of chest pain, tachypnea, dyspnea, apprehension, tachycardia, and cyanosis. However, fat embolism may cause petechiae located around the neck, anterior chest wall, axilla, buccal membrane of the mouth, and conjunctiva of the eye, which differentiates it from thrombotic embolism.

26. A patient who recently had a calcium oxalate renal stone had a bone density study, which showed a decrease in her bone density. What endocrine problem could this patient have? a. SIADH b. Hypothyroidism c. Cushing syndrome d. Hyperparathyroidism

26. d. The patient with hyperparathyroidism may have calcium nephrolithiasis, skeletal pain, decreased bone density, psychomotor retardation, or cardiac dysrhythmias. The other endocrine problems would not be related to calcium kidney stones or decreased bone density.

27. Which kind of hip fracture is usually repaired with a hip prosthesis? a. Intracapsular b. Extracapsular c. Subtrochanteric d. Intertrochanteric

27. a. A hip prosthesis is usually used for intracapsular fractures. The other options are all extracapsular fractures.

28. A patient has been diagnosed with hypoparathyroidism. What manifestations should the nurse expect to observe (select all that apply)? a. Skeletal pain b. Dry, scaly skin c. Personality changes d. Abdominal cramping e. Cardiac dysrhythmias f. Muscle spasms and stiffness

28. b, c, d, e, f. In hypoparathyroidism the patient has inadequate circulating parathyroid hormone (PTH) that leads to hypocalcemia from the inability to maintain serum calcium levels. With hypocalcemia there is muscle stiffness and spasms, which can lead to cardiac dysrhythmias and abdominal cramps. There can also be personality and visual changes and dry, scaly skin.

28. An older adult woman is admitted to the emergency department after falling at home. The nurse cautions her not to put weight on the leg after finding what in the patient assessment? a. Inability to move the toes and ankle b. Edema of the thigh extending to the knee c. Internal rotation of the leg with groin pain d. Shortening and external rotation of the leg

28. d. The classic signs of a hip fracture are shortening of the leg and external rotation accompanied by severe pain at the fracture site and additional injury could be caused by weight bearing on the extremity. The patient may not be able to move the hip or the knee but movement in the ankle and toes is not affected.

29. When the patient with parathyroid disease experiences symptoms of hypocalcemia, what is a measure that can be used to temporarily raise serum calcium levels? a. Administer IV normal saline. b. Have patient rebreathe in a paper bag. c. Administer furosemide (Lasix) as ordered. d. Administer oral phosphorus supplements.

29. b. Rebreathing in a paper bag promotes carbon dioxide retention in the blood, which lowers pH and creates an acidosis. An acidemia enhances the solubility and ionizationof calcium, increasing the proportion of total body calcium available in physiologically active form and relieving the symptoms of hypocalcemia. Saline promotes calcium excretion, as does furosemide. Phosphate levels in the blood are reciprocal to calcium and an increase in phosphate promotes calcium excretion.

29. A patient with an extracapsular hip fracture is admitted to the orthopedic unit and placed in Buck's traction. The nurse explains to the patient that the purpose of the traction is to do what? a. Pull bone fragments back into alignment b. Immobilize the leg until healing is complete c. Reduce pain and muscle spasms before surgery d. Prevent damage to the blood vessels at the fracture site

29. c. Although surgical repair is the preferred method of managing intracapsular and extracapsular hip fractures, initially patients frequently may be treated with skin traction, such as Buck's traction or Russell's traction, to immobilize the limb temporarily and to relieve the painful muscle spasms before surgery is performed. Prolonged traction would be required to reduce the fracture or immobilize it for healing, creating a very high risk for complications of immobility.

3. A patient with Alzheimer's disease (AD) dementia has manifestations of depression. The nurse knows that treatment of the patient with antidepressants will most likely do what? a. Improve cognitive function b. Not alter the course of either condition c. Cause interactions with the drugs used to treat the dementia d. Be contraindicated because of the central nervous system (CNS)-depressant effect of antidepressants

3. a. Depression is often associated with AD, especially early in the disease when the patient has awareness of the diagnosis and the progression of the disease. When dementia and depression occur together, intellectual deterioration may be more extreme. Depression is treatable and use of antidepressants often improves cognitive function.

3. The patient asks, "What does the doctor mean when he says that I have an avulsion fracture in my leg? I thought I had a sprain!" What is the best response by the nurse? a. "It is a fracture with more than two fragments." b. "It means that a ligament pulled a bone fragment loose." c. "The line of the fracture is twisted along the shaft of the bone." d. "The line of the fracture is at right angles to the longitudinal axis of the bone."

3. b. An avulsion fracture occurs when a ligament pulls a bone fragment loose, with pain similar to a sprain. A fracture with two or more fragments is a comminuted fracture. It is a spiral fracture when it is twisted around a bone shaft. It is a transverse fracture when the line of fracture is at right angles to the longitudinal axis.

30. A patient with a fractured right hip has an anterior open reduction and internal fixation of the fracture. What should the nurse plan to do postoperatively? a. Get the patient up to the chair on the first postoperative day. b. Position the patient only on the back and the unoperative side. c. Keep the leg abductor pillow on the patient even when bathing. d. Ambulate the patient with partial weight bearing by discharge.

30. a. Because the fracture site is internally fixed with pins or plates, the fracture site is stable and the patient is moved from the bed to the chair on the first postoperative day. Ambulation begins on the first or second postoperative day without weight bearing on the affected leg. Weight bearing on the affected extremity is usually restricted for 6 to 12 weeks until adequate healing is evident on x-ray. The patient may be positioned on the operative side following internal fixation and abductor pillows are used for patients who have total hip replacements.

30. A patient with hypoparathyroidism resulting from surgical treatment of hyperparathyroidism is preparing for discharge. What should the nurse teach the patient? a. Milk and milk products should be increased in the diet. b. Parenteral replacement of parathyroid hormone will be required for life. c. Calcium supplements with vitamin D can effectively maintain calcium balance. d. Bran and whole-grain foods should be used to prevent GI effects of replacement therapy.

30. c. The hypocalcemia that results from PTH deficiency is controlled with calcium and vitamin D supplementation and possibly oral phosphate binders. Replacement with PTH is not used because of antibody formation to PTH, the need for parenteral administration, and cost. Milk products, although good sources of calcium, also have high levels of phosphate, which reduce calcium absorption. Whole grains and foods containing oxalic acid also impair calcium absorption.

31. A patient is admitted to the hospital with a diagnosis of Cushing syndrome. On physical assessment of the patient, what should the nurse expect to find? a. Hypertension, peripheral edema, and petechiae b. Weight loss, buffalo hump, and moon face with acne c. Abdominal and buttock striae, truncal obesity, and hypotension d. Anorexia, signs of dehydration, and hyperpigmentation of the skin

31. a. The effects of adrenocortical hormone excess, especially glucocorticoid excess, include weight gain from accumulation and redistribution of adipose tissue, sodium and water retention, glucose intolerance, protein wasting,loss of bone structure, loss of collagen, and capillary fragility leading to petechiae. Clinical manifestations of adrenocortical hormone deficiency include hypotension, dehydration, weight loss, and hyperpigmentation of the skin.

31. What should the nurse include in discharge instructions for the patient following a hip prosthesis with a posterior approach? a. Restrict walking for 2 to 3 months. b. Take a bath rather than a shower to prevent falling. c. Keep the leg internally rotated while sitting and standing. d. Have a family member put on the patient's shoes and socks.

31. d. Patients with hip prostheses with a posterior approach must avoid extreme flexion, adduction, or internal rotation for at least 6 weeks to prevent dislocation of the prosthesis. Gradual weight bearing on the limb is allowed and ambulation should be encouraged. The leg should be not be externally rotated.

32. A patient is scheduled for a bilateral adrenalectomy. During the postoperative period, what should the nurse expect related to the administration of corticosteroids? a. Reduced to promote wound healing b. Withheld until symptoms of hypocortisolism appear c. Increased to promote an adequate response to the stress of surgery d. Reduced because excessive hormones are released during surgical manipulation of adrenal glands

32. c. Although the patient with Cushing syndrome has excess corticosteroids, removal of the glands and the stress of surgery require that high doses of corticosteroids (cortisone) be administered postoperatively for several days before weaning the dose. The nurse should monitor the patient's vital signs postoperatively to detect whether large amounts of hormones were released during surgical manipulation, obtain morning urine specimens for cortisol measurement to evaluate the effectiveness of the surgery, and provide dressing changes with aseptic technique to avoid infection as usual inflammatory responses are suppressed.

32. When preparing a patient for discharge following fixation of a mandibular fracture, the nurse determines that teaching has been successful when the patient says what? a. "I can keep my mouth moist by sucking on hard candy." b. "I should cut the wires with scissors if I begin to vomit." c. "I may use a bulk-forming laxative if my liquid diet causes constipation." d. "I should use a moist swab to clean my mouth every time I eat something."

32. c. The low-bulk, high-carbohydrate liquid diet and intake of air through a straw required during mandibular fixation often lead to constipation and flatus, which may be relieved with bulk-forming laxatives, prune juice, or ambulation. Wires or rubber bands should be cut only in the case of cardiac or respiratory arrest and patients should be taught to clear their mouth of vomitus or secretions. The mouth should be thoroughly cleaned with water, saline, or alkaline mouthwashes or using a Water Pik as necessary to remove food debris. Hard candy should not be held in the mouth.

33. A patient with Addison's disease comes to the emergency department with complaints of nausea, vomiting, diarrhea, and fever. What collaborative care should the nurse expect? a. IV administration of vasopressors b. IV administration of hydrocortisone c. IV administration of D5W with 20 mEq KCl d. Parenteral injections of adrenocorticotropic hormone (ACTH)

33. b. Vomiting and diarrhea are early indicators of Addisonian crisis and fever indicates an infection, which is causing additional stress for the patient. Treatment of a crisis requires immediate glucocorticoid replacement and IV hydrocortisone, fluids, sodium, and glucose are necessary for 24 hours. Addison's disease is a primary insufficiency of the adrenal gland and adrenocorticotropic hormone (ACTH) is not effective, nor would vasopressors be effective with the fluid deficiency of Addison's disease. Potassium levels are increased in Addison's disease and KCl would be contraindicated.

33. Priority Decision: Twenty-four hours after a below-the-knee amputation, a patient uses the call system to tell the nurse that his dressing (a compression bandage) has fallen off. What is the first action that the nurse should take? a. Apply ice to the site. b. Cover the incision with dry gauze. c. Reapply the compression dressing. d. Elevate the extremity on a couple of pillows.

33. c. The compression dressing or bandage supports the soft tissues, reduces edema, hastens healing, minimizes pain, and promotes residual limb shrinkage. If the dressing is left off, edema will form quickly and may delay rehabilitation. Elevation and ice will not be as effective at preventing the edema that will form. Dressing the incision with dry gauze will not provide the benefits of a compression dressing.

A patient complains of pain in the foot of a leg that was recently amputated. What should the nurse recognize about this pain? a. It is caused by swelling at the incision. b. It should be treated with ordered analgesics. c. It will become worse with the use of a prosthesis. d. It can be managed with diversion because it is psychologic.

34. b. Phantom sensation or phantom pain may occur following amputation, especially if pain was present in the affected limb preoperatively. The pain is a real sensation to the patient and will first be treated with analgesics and other pain interventions (i.e., tricyclic antidepressants, antiseizure drugs, transcutaneous electrical nerve stimulation [TENS], mirror therapy, acupuncture). As recovery and ambulation progress, phantom limb sensation usually subsides.

35. Priority Decision: An immediate prosthetic fitting during surgery is used for a patient with a traumatic below-theknee amputation. During the immediate postoperative period, what is a priority nursing intervention? a. Monitor the patient's vital signs. b. Assess the incision for hemorrhage. c. Elevate the residual limb on pillows. d. Have the patient flex and extend the knee every hour.

35. a. Because the device covers the residual limb, the surgical site cannot be directly seen and postoperative hemorrhage is not apparent on dressings, requiring vigilant assessment of vital signs for signs of bleeding. Elevation of the residual limb with an immediate prosthetic fitting is not necessary because the device itself prevents edema formation. Exercises to the leg are not performed in the immediate postoperative period to avoid disruption of ligatures and the suture line.

35. A patient who is on corticosteroid therapy for treatment of an autoimmune disorder has the following additional drugs ordered. Which one is used to prevent corticosteroid-induced osteoporosis? a. Potassium b. Furosemide (Lasix) c. Alendronate (Fosamax) d. Pantoprazole (Protonix)

35. c. Alendronate (Fosamax) is used to prevent corticosteroidinduced osteoporosis. Potassium is used to prevent the mineralocorticoid effect of hypokalemia. Furosemide (Lasix) is used to decrease sodium and fluid retention from the mineralocorticoid effect. Pantoprazole (Protonix) is used to prevent gastrointestinal (GI) irritation from an increase in secretion of pepsin and hydrochloric acid.

36. Why does a nurse position a patient with an above-the-knee amputation with a delayed prosthetic fitting prone several times a day? a. To prevent flexion contractures b. To assess the posterior skin flap c. To reduce edema in the residual limb d. To relieve pressure on the incision site

36. a. Flexion contractures, especially of the hip, may be debilitating and delay rehabilitation of the patient with a leg amputation. To prevent hip flexion, the patient should avoid sitting in a chair with the hips flexed or having pillows under the surgical extremity for prolonged periods and the patient should lie on the abdomen for 30 minutes three to four times a day to extend the hip.

A patient with mild iatrogenic Cushing syndrome is on an alternate-day regimen of corticosteroid therapy. What does the nurse explain to the patient about this regimen? a. It maintains normal adrenal hormone balance. b. It prevents ACTH release from the pituitary gland. c. It minimizes hypothalamic-pituitary-adrenal suppression. d. It provides a more effective therapeutic effect of the drug.

36. c. Taking corticosteroids on an alternate-day schedule for pharmacologic purposes is less likely to suppress ACTH production from the pituitary and prevent adrenal atrophy. Normal adrenal hormone balance is not maintained during glucocorticoid therapy because excessive exogenous hormone is used.

37. When caring for a patient with primary hyperaldosteronism, the nurse would question a health care provider's prescription for which drug? a. Furosemide (Lasix) b. Amiloride (Midamor) c. Spironolactone (Aldactone) d. Aminoglutethimide (Cytadren)

37. a. Hyperaldosteronism is an excess of aldosterone, which is manifested by sodium and water retention and potassium excretion. Furosemide is a potassium-wasting diuretic that would increase the potassium deficiency. Aminoglutethimide blocks aldosterone synthesis. Spironolactone and amiloride are potassium-sparing diuretics.

37. A patient who had a below-the-knee amputation is to be fitted with a temporary prosthesis. It is most important for the nurse to teach the patient to do what? a. Inspect the residual limb daily for irritation. b. Apply an elastic shrinker before applying the prosthesis. c. Perform range-of-motion (ROM) exercises to the affected leg four times a day. d. Apply alcohol to the residual limb every morning and evening to toughen the skin.

37. a. Skin breakdown on the residual limb can prevent the use of a prosthesis so the limb should be inspected every day for signs of irritation or pressure areas. No substances except water and mild soap should be used on the residual limb and range-of-motion (ROM) exercises are not necessary when the patient is using a prosthesis. A residual limb shrinker is an elastic stocking that is used to mold the limb in preparation for prosthesis use but a cotton residual limb sock is worn with the prosthesis.

38. Priority Decision: What is the priority nursing intervention during the management of the patient with pheochromocytoma? a. Administering IV fluids b. Monitoring blood pressure c. Administering β-adrenergic blockers d. Monitoring intake and output and daily weights

38. b. Pheochromocytoma is a catecholamine-producing tumor of the adrenal medulla, which may cause severe, episodic hypertension; severe, pounding headache; and profuse sweating. Monitoring for a dangerously high BP before surgery is critical, as is monitoring for BP fluctuations during medical and surgical treatment.

38. Which joint surgery is used to arthroscopically remove degenerative tissue in joints? a. Osteotomy b. Arthrodesis c. Debridement d. Synovectomy

38. c. Debridement removes degenerative tissue from joints. Osteotomy corrects bone deformity by removal of a wedge or slice of bone. Arthrodesis surgically fuses a joint to relieve pain. Synovectomy removes tissue involved in joint destruction from rheumatoid arthritis (RA).

39. When the nursing student asks the RN what an arthroplasty is, what is the best description the RN can give the student? a. Surgical fusion of a joint to relieve pain b. Correction of bone deformity by removal of a wedge or slice of bone c. Reconstruction or replacement of a joint to relieve pain and correct deformity d. Used in rheumatoid arthritis to remove the tissue involved in joint destruction

39. c. An arthroplasty is reconstruction or replacement of a joint to relieve pain and correct deformity, especially with osteoarthritis, RA, avascular necrosis, congenital deformities, or dislocations. Arthrodesis is the surgical fusion of a joint to relieve pain. An osteotomy removes a wedge of bone to correct a bone deformity. Synovectomy is used in RA to remove the tissue involved in joint destruction.

4. After a woman had a right breast mastectomy, her right arm became severely swollen. What hematologic problem caused this? a. Lymphedema b. Right-sided heart failure c. Wound on her right hand d. Refusal to use her right arm

4. a. Lymphedema is the obstruction of lymph flow that results in accumulation of lymph fluid for the patient in the right arm following a right-sided breast mastectomy. The other options are not hematologic problems that would cause extreme swelling.

4. The patient with osteoporosis had a spontaneous hip fracture. How should the nurse document this before the x-ray results return? a. Open fracture b. Oblique fracture c. Pathologic fracture d. Greenstick fracture

4. c. A pathologic fracture is a spontaneous fracture at the site of bone disease, such as osteoporosis. An open fracture is when there is communication with the external environment. The oblique fracture has a slanted fracture line. A greenstick fracture is splintered on one side and the other side is bent.

4. For what purpose would the nurse use the Mini-Mental State Examination to evaluate a patient with cognitive impairment? a. It is a good tool to determine the etiology of dementia. b. It is a good tool to evaluate mood and thought processes. c. It can help to document the degree of cognitive impairment in delirium and dementia. d. It is useful for initial evaluation of mental status but additional tools are needed to evaluate changes in cognition over time.

4. c. The Mini-Mental State Examination is a tool to document the degree of cognitive impairment and it can be used to determine a baseline from which changes over time can be evaluated. It does not evaluate mood or thought processes but can detect dementia and delirium and differentiate these from psychiatric mental illness. It cannot help to determine etiology.

4. What findings are commonly found in a patient with a prolactinoma? a. Gynecomastia in men b. Profuse menstruation in women c. Excess follicle-stimulating hormone (FSH) and luteinizing hormone (LH) d. Signs of increased intracranial pressure, including headache, nausea, and vomiting

4. d. Compression of the optic chiasm can cause visual problems as well as signs of increased intracranial pressure, including headache, nausea, and vomiting. About 30% of prolactinomas will have excess prolactin secretion with manifestations of impotence in men, galactorrhea or amenorrhea in women without relationship to pregnancy, and decreased libido in both men and women. There is decreased follicle-stimulating hormone (FSH) and luteinizing hormone (LH).

40. A 65-year-old patient has undergone a right total hip arthroplasty with a cemented prosthesis for treatment of severe osteoarthritis of the hip. What is included in the activity the nurse anticipates for the patient on the patient's first or second postoperative day? a. Transfer from the bed to the chair twice a day only b. Turning from the back to the unaffected side q2hr only c. Crutch walking with non-weight bearing on the operative leg d. Ambulation and weight bearing on the right leg with a walker

40. d. Physical therapy is initiated on the first postoperative day with ambulation and weight bearing using a walker for a patient with a cemented prosthesis and non-weight bearing on the operative side for an uncemented prosthesis. In addition, the patient sits in the chair at least twice a day and is turned to both sides and back with the operative leg supported.

41. When positioning the patient after a total hip arthroplasty with a posterior approach, it is important that the nurse maintain the affected extremity in what position? a. Adduction and flexion b. Abduction and extension c. Abduction and internal rotation d. Adduction and external rotation

41. b. Following a total hip arthroplasty with a posterior approach, during hospitalization an abduction pillow is placed between the legs to maintain abduction and the leg is extended. Extremes of internal rotation, adduction, and 90-degree flexion of the hip must be avoided for 4 to 6 weeks postoperatively to prevent dislocation of the prosthesis.

42. Following a knee arthroplasty, a patient has a continuous passive motion machine for the affected joint. The nurse explains to the patient that this device is used for what purpose? a. To relieve edema and pain at the incision site b. To promote early joint mobility and increase knee flexion c. To prevent venous stasis and the formation of a deep venous thrombosis d. To improve arterial circulation to the affected extremity to promote healing

42. b. Continuous passive motion machines are frequently used following knee surgery to promote earlier joint mobility. Because joint dislocation is not a problem with knee replacements, early exercise with straight leg raises and gentle ROM is also encouraged postoperatively.

Priority Decision: A patient with severe ulnar deviation of the hands undergoes an arthroplasty with reconstruction and replacement of finger joints. Postoperatively, what is it most important for the nurse to do? a. Position the fingers lower than the elbow. b. Perform neurovascular assessments of the fingers q2-4hr. c. Encourage the patient to gently flex, extend, abduct, and adduct the fingers q4hr. d. Remind the patient that function of the hands is more important than their cosmetic appearance.

43. b. Neurovascular checks of the fingers following surgery of the hands are essential to detect compromised vascular and neurologic function caused by trauma or edema. Postoperatively, the hands are elevated with a bulky dressing in place and when the dressing is removed, a guided splinting program is started. Exercises are performed three to four times a day when the splints are removed and the patient is discharged. Before surgery, it must be made clear to the patient that the goal of the surgery is to restore function related to grasp, pinch, stability, and strength and the hands will not necessarily have good cosmetic appearance.

44. Priority Decision: Following change-of-shift handoff, which patient should the nurse assess first? a. A 58-year-old male experiencing phantom pain and requesting analgesic b. A 72-year-old male being transferred to a skilled nursing unit following repair of a hip fracture c. A 25-year-old female in left leg skeletal traction asking for the weights to be lifted for a few minutes d. A 68-year-old male with a new lower leg cast complaining that the cast is too tight and he cannot feel his toes

44. d. The patient with a tight cast may be at risk for neurovascular compromise (impaired circulation and peripheral nerve damage) and should be assessed first. The other patients should be seen as soon as possible. Providing analgesia for the patient with phantom pain would be the next priority. The patient in skeletal traction needs explanation of the purpose and functioning of the traction. She may need analgesia or muscle relaxants to help tolerate the traction.

The family of a patient newly diagnosed with hepatitis A asks the nurse what they can do to prevent becoming ill themselves. Which response by the nurse is most appropriate? "The hepatitis vaccine will provide immunity from this exposure and future exposures." "I am afraid there is nothing you can do since the patient was infectious before admission." "You will need to be tested first to make sure you don't have the virus before we can treat you." "An injection of immunoglobulin will need to be given to prevent or minimize the effects from this exposure."

"An injection of immunoglobulin will need to be given to prevent or minimize the effects from this exposure." Immunoglobulin provides temporary (1-2 months) passive immunity and is effective for preventing hepatitis A if given within 2 weeks after exposure. It may not prevent infection in all persons, but it will at least modify the illness to a subclinical infection. The hepatitis vaccine is only used for preexposure prophylaxis.

The nurse provides discharge instructions for a 64-year-old woman with ascites and peripheral edema related to cirrhosis. Which statement, if made by the patient, indicates teaching was effective? "It is safe to take acetaminophen up to four times a day for pain." "Lactulose (Cephulac) should be taken every day to prevent constipation." "Herbs and other spices should be used to season my foods instead of salt." "I will eat foods high in potassium while taking spironolactone (Aldactone)."

"Herbs and other spices should be used to season my foods instead of salt." A low-sodium diet is indicated for the patient with ascites and edema related to cirrhosis. Table salt is a well-known source of sodium and should be avoided. Alternatives to salt to season foods include the use of seasonings such as garlic, parsley, onion, lemon juice, and spices. Pain medications such as acetaminophen, aspirin, and ibuprofen should be avoided as these medications may be toxic to the liver. The patient should avoid potentially hepatotoxic over-the-counter drugs (e.g., acetaminophen) because the diseased liver is unable to metabolize these drugs. Spironolactone is a potassium-sparing diuretic. Lactulose results in the acidification of feces in bowel and trapping of ammonia, causing its elimination in feces.

When teaching the patient with acute hepatitis C (HCV), the patient demonstrates understanding when the patient makes which statement? "I will use care when kissing my wife to prevent giving it to her." "I will need to take adofevir (Hepsera) to prevent chronic HCV." "Now that I have had HCV, I will have immunity and not get it again." "I will need to be checked for chronic HCV and other liver problems."

"I will need to be checked for chronic HCV and other liver problems." The majority of patients who acquire HCV usually develop chronic infection, which may lead to cirrhosis or liver cancer. HCV is not transmitted via saliva, but percutaneously and via high-risk sexual activity exposure. The treatment for acute viral hepatitis focuses on resting the body and adequate nutrition for liver regeneration. Adofevir (Hepsera) is taken for severe hepatitis B (HBV) with liver failure. Chronic HCV is treated with pegylated interferon with ribavirin. Immunity with HCV does not occur as it does with HAV and HBV, so the patient may be reinfected with another type of HCV.

The patient with cirrhosis is being taught self-care. Which statement indicates the patient needs more teaching? "If I notice a fast heart rate or irregular beats, this is normal for cirrhosis." "I need to take good care of my belly and ankle skin where it is swollen." "A scrotal support may be more comfortable when I have scrotal edema." "I can use pillows to support my head to help me breathe when I am in bed."

"If I notice a fast heart rate or irregular beats, this is normal for cirrhosis." Correct If the patient with cirrhosis experiences a fast or irregular heart rate, it may be indicative of hypokalemia and should be reported to the health care provider, as this is not normal for cirrhosis. Edematous tissue is subject to breakdown and needs meticulous skin care. Pillows and a semi-Fowler's or Fowler's position will increase respiratory efficiency. A scrotal support may improve comfort if there is scrotal edema.

The nurse instructs a 50-year-old woman about cholestyramine to reduce pruritis caused by gallbladder disease. Which statement by the patient to the nurse indicates she understands the instructions? "This medication will help me digest fats and fat-soluble vitamins." "I will apply the medicated lotion sparingly to the areas where I itch." "The medication is a powder and needs to be mixed with milk or juice." "I should take this medication on an empty stomach at the same time each day."

"The medication is a powder and needs to be mixed with milk or juice." For treatment of pruritus, cholestyramine may provide relief. This is a resin that binds bile salts in the intestine, increasing their excretion in the feces. Cholestyramine is in powder form and should be mixed with milk or juice before oral administration.

2. An increase in which blood cell indicates an increased rate of erythropoiesis? a. Basophil b. Monocyte c. Reticulocyte d. Lymphocyte

2. c. Increased reticulocytes, or immature RBCs, indicate an increased rate of erythropoiesis or stimulation of erythrocyte (RBC) production by the bone marrow. Basophils release granules that increase allergic and inflammatory responses and are stimulated by granulocyte colony-stimulating factor. Monocytes may become tissue macrophages. Lymphocytes are primarily responsible for the immune response.

2. During assessment of the patient with acromegaly, what should the nurse expect the patient to report? a. Infertility b. Dry, irritated skin c. Undesirable changes in appearance d. An increase in height of 2 to 3 inches a year

2. c. The increased production of GH in acromegaly causes an increase in thickness and width of bones and enlargement of soft tissues, resulting in marked changes in facial features, oily and coarse skin, and speech difficulties. Infertility is not a common finding because GH is usually the only pituitary hormone involved in acromegaly. Height is not increased in adults with GH excess because the epiphyses of the bones are closed.

2. The nurse teaches individuals that one of the best ways to prevent musculoskeletal injuries during physical exercise is by doing what? a. Increase muscle strength with daily isometric exercise. b. Avoid exercising on concrete or hard pavement surfaces. c. Perform stretching and warm-up exercises before exercise. d. Wrap susceptible joints with elastic bandages or adhesive tape before exercise.

2. c. Warm-up exercises "prelengthen" potentially strained tissues by avoiding the quick stretch often encountered in sports and also increase the temperature of muscle, resulting in increased speed of cell metabolism, increased speed of nerve impulses, and improved oxygenation of muscle fibers. Stretching is also thought to improve kinesthetic awareness, lessening the chance of uncoordinated movement. Muscle strength is not a key factor in soft tissue injuries and taping or wrapping joints may actually predispose a person to injury by weakening the joint, unless a previous injury is being treated.

2. Which statement accurately describes dementia? a. Overproduction of β-amyloid protein causes all dementias. b. Dementia resulting from neurodegenerative causes can be prevented. c. Dementia caused by hepatic or renal encephalopathy cannot be reversed. d. Vascular dementia can be diagnosed by brain lesions identified with neuroimaging.

2. d. The diagnosis of vascular dementia can be aided by neuroimaging studies showing vascular brain lesions along with exclusion of other causes of dementia. Overproduction of β-amyloid protein contributes to Alzheimer's disease (AD). Vascular dementia can be prevented or slowed by treating underlying diseases (e.g., diabetes mellitus, cardiovascular disease). Dementia caused by hepatic or renal encephalopathy potentially can be reversed.

ONLINE REMEMBER 34 AND 26 QUESTIONS ON CD

...

20. Priority Decision: A patient is undergoing a contrast computed tomography (CT) of the spleen. What is most important for the nurse to ask the patient about before the test? a. Iodine sensitivity b. Prior blood transfusions c. Phobia of confined spaces d. Internal metal implants or appliances

20. a. A contrast CT scan involves the use of an iodine-based dye that could cause a reaction if the patient is sensitive to iodine. Metal implants or internal, appliances and claustrophobia should be determined before magnetic resonance imaging (MRI). Prior blood transfusions are not a factor in this diagnostic test.

10. The x-ray shows that the patient's fracture is at the remodeling stage. What characteristics of the fracture healing process are happening at this stage (select all that apply)? a. Radiologic union b. Absorption of excess cells c. Return to preinjury strength and shape d. Semisolid blood clot at the ends of fragments e. Deposition and absorption of bone in response to stress f. Unorganized network of bone woven around fracture parts

10. a, b, c, e. When the remodeling stage of healing occurs, radiologic union is present. Excess callus is reabsorbed, trabecular bone is laid, and the bone returns to its preinjury structure strength and shape. The osteoblasts and osteoclasts function normally in response to stress. The fracture hematoma stage is when the hematoma at the ends of the fragments becomes a semisolid blood clot. There is an unorganized network of bone composed of cartilage, osteoblasts, calcium, and phosphorus woven around fracture parts in the callus formation stage.

When reviewing the results of an 83-year-old patient's blood tests, which finding would be of most concern to the nurse? a. Platelets 150,000/μL b. Serum iron 50 mcg/dL c. Partial thromboplastin time (PTT) 60 seconds d. Erythrocyte sedimentation rate (ESR) 35 mm in 1 hour

10. c. As a person ages the partial thromboplastin time (PTT) is normally decreased, so an abnormally high PTT of 60 seconds is an indication that bleeding could readily occur. Platelets are unaffected by aging and 150,000/μL is a normal count. Serum iron levels are decreased and the erythrocyte sedimentation rate (ESR) is significantly increased with aging, as are reflected in these values.

10. The patient is receiving donepezil (Aricept), lorazepam (Ativan), risperidone (Risperdal), and sertraline (Zoloft) for the management of AD. What benzodiazepine medication is being used to help manage this patient's behavior? a. Sertraline (Zoloft) b. Donepezil (Aricept) c. Lorazepam (Ativan) d. Risperidone (Risperdal)

10. c. Lorazepam (Ativan) is a benzodiazepine used to manage behavior with AD. Sertraline (Zoloft) is a selective serotonin reuptake inhibitor used to treat depression. Donepzil (Aricept) is a cholinesterase inhibitor used for decreased memory and cognition. Risperidone (Risperdal) is an antipsychotic used for behavior management.

10. In a patient with central diabetes insipidus, what will the administration of ADH during a water deprivation test result in? a. Decrease in body weight b. Increase in urinary output c. Decrease in blood pressure d. Increase in urine osmolality

10. d. A patient with central diabetes insipidus has a deficiency of ADH with excessive loss of water from the kidney, hypovolemia, hypernatremia, and dilute urine with a low specific gravity. When vasopressin is administered, the symptoms are reversed, with water retention, decreased urinary output that increases urine osmolality, and an increase in BP.

11. A patient with a bone marrow disorder has an overproduction of myeloblasts. The nurse would expect the results of a complete blood count (CBC) to include an increase in which cell types (select all that apply)? a. Basophils b. Eosinophils c. Monocytes d. Neutrophils e. Lymphocytes

11. a, b, d. The myeloblast is a committed hematopoietic cell found in the bone marrow from which granulocytes develop. A disorder in which myeloblasts are overproduced would result in increased basophils, eosinophils, and neutrophils.

11. A patient is brought to the emergency department with an injured lower left leg following a fall while rock climbing. The nurse identifies the presence of a fracture based on what cardinal sign of fracture? a. Muscle spasms b. Obvious deformity c. Edema and swelling d. Pain and tenderness

11. b. Deformity is the cardinal sign of fracture but may not be apparent in all fractures. Other supporting signs include edema and swelling, localized pain and tenderness, muscle spasm, ecchymosis, loss of function, crepitation, and an inability to bear weight.

11. What N-methyl-d-aspartate (NMDA) receptor antagonist is frequently used for a patient with AD who is experiencing decreased memory and cognition? a. Trazodone (Desyrel) b. Olanzapine (Zyprexa) c. Rivastigmine (Exelon) d. Memantine (Namenda)

11. d. Memantine (Namenda) is the N-methyl-d-aspartate (NMDA) receptor antagonist frequently used for AD patients with decreased memory and cognition. Trazodone (Desyrel) is an atypical antidepressant that may help with sleep problems. Olanzapine (Zyprexa) is an antipsychotic medication used for behavior management. Rivastigmine (Exelon) is a cholinesterase inhibitor used for decreased memory and cognition.

12. A patient with a fractured femur experiences the complication of malunion. The nurse recognizes that what happens with this complication? a. The fracture heals in an unsatisfactory position. b. The fracture fails to heal properly despite treatment. c. Fracture healing progresses more slowly than expected. d. Loss of bone substances occurs as a result of immobilization.

12. a. A malunion occurs when the bone heals in the expected time but in an unsatisfactory position, possibly resulting in deformity or dysfunction. Nonunion occurs when the fracture fails to heal properly despite treatment and delayed union is healing of the fracture at a slower rate than expected. In posttraumatic osteoporosis, the loss of bone substances occurs as a result of immobilization.

12. During the nursing assessment of a patient with anemia, what specific information should the nurse ask the patient about? a. Stomach surgery b. Recurring infections c. Corticosteroid therapy d. Oral contraceptive use

12. a. The parietal cells of the stomach secrete intrinsic factor, a substance necessary for the absorption of cobalamin (vitamin B12), and if all or part of the stomach is removed, the lack of intrinsic factor can lead to impaired RBC production and pernicious anemia. Recurring infections indicate decreased WBCs and immune response and corticosteroid therapy may cause a neutrophilia and lymphopenia. Oral contraceptive use is strongly associated with changes in blood coagulation.

12. When caring for a patient with nephrogenic diabetes insipidus, what should the nurse expect the treatment to include? a. Fluid restriction b. Thiazide diuretics c. A high-sodium diet d. Chlorpropamide (Diabinese)

12. b. In nephrogenic diabetes insipidus, the kidney is unable to respond to ADH, so vasopressin or hormone analogs are not effective. Thiazide diuretics slow the glomerular filtration rate (GFR) in the kidney and produce a decrease in urine output. Low-sodium diets (<3 g/day) are also thought to decrease urine output. Fluids are not restricted because the patient could easily become dehydrated.

12. A patient with AD in a long-term care facility is wandering the halls very agitated, asking for her "mommy" and crying. What is the best response by the nurse? a. Ask the patient, "Why are you behaving this way?" b. Tell the patient, "Let's go get a snack in the kitchen." c. Ask the patient, "Wouldn't you like to lie down now?" d. Tell the patient, "Just take some deep breaths and calm down."

12. b. Patients with moderate to severe AD frequently become agitated but because their short-term memory loss is so pronounced, distraction is a very good way to calm them. "Why" questions are upsetting to them because they don't know the answer and they cannot respond to normal relaxation techniques.

13. The sister of a patient with AD asks the nurse whether prevention of the disease is possible. In responding, the nurse explains that there is no known way to prevent AD but there are ways to keep the brain healthy. What is included in the ways to keep the brain healthy (select all that apply)? a. Avoid trauma to the brain. b. Recognize and treat depression early. c. Avoid social gatherings to avoid infections. d. Do not overtax the brain by trying to learn new skills. e. Daily wine intake will increase circulation to the brain. f. Exercise regularly to decrease the risk for cognitive decline

13. a, b, f. Avoiding trauma to the brain, treating depression early, and exercising regularly can maintain cognitive function. Staying socially active, avoiding intake of harmful substances, and challenging the brain to keep its connections active and create new ones also help to keep the brain healthy.

13. What is a disadvantage of open reduction and internal fixation of a fracture compared to closed reduction? a. Infection b. Skin irritation c. Nerve impairment d. Complications of immobility

13. a. Open reduction uses a surgical incision to correct bone alignment but infection is the main disadvantage, as well as anesthesia complications or the effect of preexisting medical conditions. Skin irritation and nerve impairment is most likely with skin traction. Prolonged immobility is possible with skeletal traction.

13. What characteristic is related to Hashimoto's thyroiditis? a. Enlarged thyroid gland b. Viral-induced hyperthyroidism c. Bacterial or fungal infection of thyroid gland d. Chronic autoimmune thyroiditis with antibody destruction of thyroid tissue

13. d. In Hashimoto's thyroiditis, thyroid tissue is destroyed by autoimmune antibodies. An enlarged thyroid gland is a goiter. Viral-induced hyperthyroidism is subacute granulomatous thyroiditis. Acute thyroiditis is caused by bacterial or fungal infection.

14. Which statement accurately describes Graves' disease? a. Exophthalmos occurs in Graves' disease. b. It is an uncommon form of hyperthyroidism. c. Manifestations of hyperthyroidism occur from tissue desensitization to the sympathetic nervous system. d. Diagnostic testing in the patient with Graves' disease will reveal an increased thyroid-stimulating hormone (TSH) level.

14. a. Exophthalmos or protrusion of the eyeballs may occur in Graves' disease from increased fat deposits and fluid in the orbital tissues and ocular muscles, forcing the eyeballs outward. Graves' disease is the most common form of hyperthyroidism. Increased metabolic rate and sensitivity of the sympathetic nervous system lead to the clinical manifestations. Thyroid-stimulating hormone (TSH) level is decreased in Graves' disease.

14. The son of a patient with early-onset AD asks if he will get AD. What should the nurse tell this man about the genetics of AD? a. The risk of early-onset AD for the children of parents with it is about 50%. b. Women get AD more often than men do, so his chances of getting AD are slim. c. The blood test for the ApoE gene to identify this type of AD can predict who will develop it. d. This type of AD is not as complex as regular AD, so he does not need to worry about getting AD.

14. a. The risk of early-onset AD for the children of parents with it is 50%. Women do get AD more often than men but that is more likely related to women living longer than men than to the type of AD. ApoE gene testing is used for research with late-onset AD but does not predict who will develop the disease. Late-onset AD is more genetically complex than early-onset AD and is more common in those over age 60 but because his parent has early-onset AD he is at a 50% risk of getting it.

14. A young patient with a fractured femur has a hip spica cast applied. While the cast is drying, what should the nurse do? a. Elevate the legs above the level of the heart for 24 hours. b. Turn the patient to both sides and prone to supine every 2 hours. c. Cover the cast with a light blanket to avoid chilling from evaporation. d. Assess the patient frequently for abdominal pain, nausea, and vomiting.

14. d. Complaints of abdominal pain or pressure, nausea, and vomiting are signs of cast syndrome that occur when hip spica casts or body jacket braces are applied too tightly, causing compression of the superior mesenteric artery against the duodenum. The cast may need to be split or removed and the health care provider should be notified. Elevation is not indicated for a spica cast and the patient with a spica cast should not be placed in the prone position during the initial drying stage because the cast is so large and heavy it may break. A cast should never be covered with a blanket because heat builds up in the cast and may increase edema.

15. A patient with moderate AD has a nursing diagnosis of impaired memory related to effects of dementia. What is an appropriate nursing intervention for this patient? a. Post clocks and calendars in the patient's environment. b. Establish and consistently follow a daily schedule with the patient. c. Monitor the patient's activities to maintain a safe patient environment. d. Stimulate thought processes by asking the patient questions about recent activities

15. b. Adhering to a regular, consistent daily schedule helps the patient to avoid confusion and anxiety and is important both during hospitalization and at home. Clocks and calendars may be useful in early AD but they have little meaning to a patient as the disease progresses. Questioning the patient about activities and events they cannot remember is threatening and may cause severe anxiety. Maintaining a safe environment for the patient is important but does not change the disturbed thought processes.

15. A patient is admitted with an open fracture of the tibia following a bicycle accident. During assessment of the patient, what specifically should the nurse question the patient about? a. Any previous injuries to the leg b. The status of tetanus immunization c. The use of antibiotics in the last month d. Whether the injury was exposed to dirt or gravel

15. b. Infection is the greatest risk with an open fracture and all open fractures are considered contaminated. Tetanus prevention is always indicated if the patient has not been immunized or does not have current boosters. Prophylactic antibiotics are often used in management of open fractures but recent antibiotic therapy is not relevant, nor is previous injury to the site.

15. During physical assessment of a patient with thrombocytopenia, what would the nurse expect to find? a. Sternal tenderness b. Petechiae and purpura c. Jaundiced sclera and skin d. Tender, enlarged lymph nodes

15. b. Petechiae are small, flat, red, or reddish-brown pinpoint microhemorrhages that occur on the skin when platelet levels are low. When petechiae are numerous, they group, causing reddish bruises known as purpura. Sternal tenderness is associated with leukemias. Jaundice occurs when anemias are of a hemolytic origin, resulting in accumulation of bile pigments from RBCs. Enlarged, tender lymph nodes are associated with infection or cancer.

15. A patient with Graves' disease asks the nurse what caused the disorder. What is the best response by the nurse? a. "The cause of Graves' disease is not known, although it is thought to be genetic." b. "It is usually associated with goiter formation from an iodine deficiency over a long period of time." c. "Antibodies develop against thyroid tissue and destroy it, causing a deficiency of thyroid hormones." d. "In genetically susceptible persons, antibodies are formed that cause excessive thyroid hormone secretion."

15. d. In Graves' disease, antibodies to the TSH receptor are formed, attach to the receptors, and stimulate the thyroid gland to release triiodothyronine (T3), thyroxine (T4), or both, creating hyperthyroidism. The disease is not directly genetic but individuals appear to have a genetic susceptibility to develop autoimmune antibodies. Goiter formation from insufficient iodine intake is usually associated with hypothyroidism.

16. A patient with a hematologic disorder has a smooth, shiny, red tongue. Which laboratory result would the nurse expect to see? a. Neutrophils 45% b. Hgb 9.6 g/dL (96 g/L) c. WBC count 13,500/μL d. Red blood cell (RBC) count 6.4 × 106/μL

16. b. A smooth, shiny, reddened tongue is an indication of iron-deficiency anemia or pernicious anemia that would be reflected by a decreased hemoglobin level. The decreased neutrophils would be indicative of neutropenia. The increased WBC count could be indicative of an infection and the increased RBC count of polycythemia.

16. A patient is admitted to the hospital with thyrotoxicosis. On physical assessment of the patient, what should the nurse expect to find? a. Hoarseness and laryngeal stridor b. Bulging eyeballs and dysrhythmias c. Elevated temperature and signs of heart failure d. Lethargy progressing suddenly to impairment of consciousness

16. c. A hyperthyroid crisis results in marked manifestations of hyperthyroidism, with severe tachycardia, heart failure, shock, hyperthermia, restlessness, irritability, abdominal pain, vomiting, diarrhea, delirium, and coma. Although exophthalmos may be present in the patient with Graves' disease, it is not a significant factor in hyperthyroid crisis. Hoarseness and laryngeal stridor are characteristic of the tetany of hypoparathyroidism and lethargy progressing to coma is characteristic of myxedema coma, a complication of hypothyroidism.

Priority Decision: A patient has fallen in the bathroom of the hospital room and reports pain in the upper right arm and elbow. Before splinting the injury, the nurse knows that the priority management of a possible fracture should include which action? a. Elevation of the arm b. Application of ice to the site c. Notification of the health care provider d. Neurovascular checks below the site of the injur

16. d. Pulses, sensation, and motor function distal to the injury should be checked before and after splinting to assess for nerve or vascular damage and documented to avoid doubts about whether a problem discovered later was missed during the original examination or was caused by the treatment. Elevation of the limb and application of ice should be instituted after the extremity is splinted.

17. The wife of a man with moderate AD has a nursing diagnosis of social isolation related to diminishing social relationships and behavioral problems of the patient with AD. What is a nursing intervention that would be appropriate to provide respite care and allow the wife to have satisfactory contact with significant others? a. Help the wife to arrange for adult day care for the patient. b. Encourage permanent placement of the patient in the Alzheimer's unit of a long-term care facility. c. Refer the wife to a home health agency to arrange daily home nursing visits to assist with the patient's care. d. Arrange for hospitalization of the patient for 3 or 4 days so that the wife can visit out-of-town friends and relatives.

17. a. Adult day care is an option to provide respite for caregivers and a protective environment for the patient during the early and middle stages of AD. There are also in-home respite care providers. The respite from the demands of care allows the caregiver to maintain social contacts, perform normal tasks of living, and be more responsive to the patient's needs. Visits by home health nurses involve the caregiver and cannot provide adequate respite. Institutional placement is not always an acceptable option at earlier stages of AD, nor is hospitalization available for respite care.

17. To assess for neurologic status in a patient with a fractured humerus, what should the nurse ask the patient to do? a. Evert, invert, dorsiflex, and plantar flex the foot. b. Abduct, adduct, and oppose the fingers and pronate and supinate the hand. c. Assess the location, quality, and intensity of pain below the site of the injury. d. Assess the color, temperature, capillary refill, peripheral pulses, and presence of edema in the extremity.

17. b. Neurologic assessment includes evaluation of motor and sensory function and, in the upper extremity, includes abduction and adduction of the fingers, opposition of the fingers, and supination and pronation of the hands. It also includes sensory perception in the fingers. Evaluation of the feet would occur in lower extremity injuries. Assessment of color, temperature, capillary refill, peripheral pulses, and edema evaluates vascular status.

17. What medication is used with thyrotoxicosis to block the effects of the sympathetic nervous stimulation of the thyroid hormones? a. Potassium iodide b. Atenolol (Tenormin) c. Propylthiouracil (PTU) d. Radioactive iodine (RAI)

17. b. The β-adrenergic blocker atenolol is used to block the sympathetic nervous system stimulation by thyroid hormones. Potassium iodide is used to prepare the patient for thyroidectomy or for treatment of thyrotoxic crisis to inhibit the synthesis of thyroid hormones. Antithyroid medications inhibit the synthesis of thyroid hormones. Radioactive iodine (RAI) therapy destroys thyroid tissue, which limits thyroid hormone secretion.

Which characteristics describe the use of RAI (select all that apply)? a. Often causes hypothyroidism over time b. Decreases release of thyroid hormones c. Blocks peripheral conversion of T4 to T3 d. Treatment of choice in nonpregnant adults e. Decreases thyroid secretion by damaging thyroid gland f. Often used with iodine to produce euthyroid before surgery

18. a, d, e. RAI causes hypothyroidism over time by damaging thyroid tissue and is the treatment of choice for nonpregnant adults. Potassium iodide decreases the release of thyroidhormones and decreases the size of the thyroid gland preoperatively. Propylthiouracil (PTU) blocks peripheral conversion of T4 to T3 and may be used with iodine to produce a euthyroid state before surgery.

20. As a precaution for vocal cord paralysis from damage to the recurrent laryngeal nerve during thyroidectomy surgery, what equipment should be in the room in case it is needed for this emergency situation? a. Tracheostomy tray b. Oxygen equipment c. IV calcium gluconate d. Paper and pencil for communication

20. a. A tracheostomy tray is in the room to use if vocal cord paralysis occurs from recurrent laryngeal nerve damage or for laryngeal stridor from tetany. The oxygen equipment may be useful but will not improve oxygenation with vocal cord paralysis without a tracheostomy. IV calcium salts will be used if hypocalcemia occurs from parathyroid damage. The paper and pencil for communication may be helpful, especially if a tracheostomy is performed, but will not aid in emergency oxygenation of the patient.

18. The health care provider is trying to differentiate the diagnosis of the patient between dementia and dementia with Lewy bodies (DLB). What observations by the nurse support a diagnosis of DLB (select all that apply)? a. Tremors b. Fluctuating cognitive ability c. Disturbed behavior, sleep, and personality d. Symptoms of pneumonia, including congested lung sounds e. Bradykinesia, rigidity, and postural instability without tremor

18. b, e. Dementia with Lewy bodies (DLB) is diagnosed with dementia plus two of the following symptoms: (1) extrapyramidal signs such as bradykinesia, rigidity, and postural instability but not always a tremor, (2) fluctuating cognitive ability, and (3) hallucinations. The extrapyramidal signs plus tremors would more likely indicate Parkinson's disease. Disturbed behavior, sleep, personality, and eventually memory are characteristics of frontotemporal lobe degeneration (FTLD).

20. A 72-year-old woman is hospitalized in the intensive care unit (ICU) with pneumonia resulting from chronic obstructive pulmonary disease (COPD). She has a fever, productive cough, and adventitious breath sounds throughout her lungs. In the past 24 hours her fluid intake was 1000 mL and her urine output was 700 mL. She was diagnosed with early-stage AD 6 months ago but has been able to maintain her activities of daily living (ADLs) with supervision. Identify at least six risk factors for the development of delirium in this patient. (Fill in the blanks.) a. b. c. d. e. f.

20. a. Age; b. infection; c. hypoxemia (lung disease); d. intensive care unit (ICU) hospitalization (change in environment, sensory overload); e. preexisting dementia; f. dehydration. Also: hyperthermia and potentially medications to treat chronic obstructive pulmonary disease (COPD) and pneumonia.

20. What surgical treatment will the nurse prepare the patient for in the presence of compartment syndrome? a. Fasciotomy b. Amputation c. Internal fixation d. Release of tendons

20. a. Soft tissue edema in the area of the injury may cause an increase of pressure within the closed spaces of the tissue compartments formed by the nonelastic fascia, creating compartment syndrome. If symptoms occur, it may be necessary to incise the fascia surgically, a procedure known as a fasciotomy. Amputation is usually necessary only if the limb becomes septic because of untreated compartment syndrome.

18. A patient is discharged from the outpatient clinic following application of a synthetic fiberglass long arm cast for a fractured ulna. Before discharge, the nurse instructs the patient to do what? a. Never get the cast wet. b. Move the shoulder and fingers frequently. c. Place tape petals around the edges of the cast when it is dry. d. Use a sling to support the arm at waist level for the first 48 hours.

18. b. A patient with any type of cast should exercise the joints above and below the cast frequently and moving the fingers frequently will improve circulation and help to prevent edema. Unlike plaster casts, thermoplastic resin or fiberglass casts are relatively waterproof and, if they become wet, can be dried with a hair dryer on low setting. Tape petals are used on plaster casts to protect the edges from breaking and crumbling but are not necessary for synthetic casts. After the cast is applied, the extremity should be elevated at about the level of the heart to promote venous return and ice may be used to prevent edema.

21. When teaching a patient about a bone marrow examination, what should the nurse explain? a. The procedure will be done under general anesthesia because it is so painful. b. The patient will not have any pain after the area at the puncture site is anesthetized. c. The patient will experience a brief, very sharp pain during aspiration of the bone marrow. d. There will be no pain during the procedure, but an ache will be present several days afterward.

21. c. The aspiration of bone marrow content is done with local anesthesia at the site of the puncture but the aspiration causes a suction pain that is quite painful but very brief. There generally is no residual pain following the test.

21. Which type of fracture occurred when there is radial nerve and brachial artery damage and the fracture is reduced with a hanging arm cast? a. Fractured tibia b. Colles' fracture c. Fractured humerus d. Femoral shaft fracture

21. c. The fractured humerus may cause radial nerve and brachial artery damage and it may be reduced nonsurgically with a hanging arm cast. A fractured tibia and femoral shaft are in the leg. The Colles' fracture is in the wrist and manifests with pronounced swelling and obvious deformity of the wrist; it is treated with closed manipulation and immobilization.

19. Delegation Decision: The RN in charge at a long-term care facility could delegate which activities to unlicensed assistive personnel (UAP) (select all that apply)? a. Assist the patient with eating. b. Provide personal hygiene and skin care. c. Check the environment for safety hazards. d. Assist the patient to the bathroom at regular intervals. e. Monitor for skin breakdown and swallowing difficulties.

19. a, b, d. All caregivers are responsible for the patient's safety. Basic care activities, such as those associated with personal hygiene and activities of daily living (ADLs) can be delegated to unlicensed assistive personnel (UAP). The RN will perform ongoing assessments and develop and revise the plan of care as needed. The RN will assess the patient's safety risk factors, provide education, and make referrals. The licensed practical nurse (LPN) could check the patient's environment for potential safety hazards.

The nurse is caring for a woman recently diagnosed with viral hepatitis A. Which individual should the nurse refer for an immunoglobin (IG) injection? A caregiver who lives in the same household with the patient A friend who delivers meals to the patient and family each week A relative with a history of hepatitis A who visits the patient daily A child living in the home who received the hepatitis A vaccine 3 months ago

A caregiver who lives in the same household with the patient IG is recommended for persons who do not have anti-HAV antibodies and are exposed as a result of close contact with persons who have HAV or foodborne exposure. Persons who have received a dose of HAV vaccine more than 1 month previously or who have a history of HAV infection do not require IG.

When providing discharge teaching for the patient after a laparoscopic cholecystectomy, what information should the nurse include? A lower-fat diet may be better tolerated for several weeks. Correct Do not return to work or normal activities for 3 weeks. Bile-colored drainage will probably drain from the incision. Keep the bandages on and the puncture site dry until it heals.

A lower-fat diet may be better tolerated for several weeks. Correct Although the usual diet can be resumed, a low-fat diet is usually better tolerated for several weeks following surgery. Normal activities can be gradually resumed as the patient tolerates. Bile-colored drainage or pus, redness, swelling, severe pain, and fever may all indicate infection. The bandage may be removed the day after surgery, and the patient can shower.

A 40-year-old man tells the nurse he has a diagnosis for the color and temperature changes of his limbs but can't remember the name of it. He says he must stop smoking and avoid trauma and exposure of his limbs to cold temperatures to get better. This description should allow the nurse to ask the patient if he has which diagnosis? A) Buerger's disease B) Venous thrombosis C) Acute arterial ischemia D) Raynaud's phenomenon

A) Buerger's disease is a nonatherosclerotic, segmental, recurrent inflammatory disorder of small and medium-sized veins and arteries of upper and lower extremities leading to color and temperature changes of the limbs, intermittent claudication, rest pain, and ischemic ulcerations. It primarily occurs in men younger than 45 years old with a long history of tobacco and/or marijuana use. Buerger's disease treatment includes smoking cessation, trauma and cold temperature avoidance, and a walking program. Venous thrombosis is the formation of a thrombus in association with inflammation of the vein. Acute arterial ischemia is a sudden interruption in arterial blood flow to a tissue caused by embolism, thrombosis, or trauma. Raynaud's phenomenon is characterized by vasospasm-induced color changes of the fingers, toes, ears, and nose.

The nurse is admitting a 68-year-old preoperative patient with a suspected abdominal aortic aneurysm (AAA). The medication history reveals that the patient has been taking warfarin (Coumadin) on a daily basis. Based on this history and the patient's admission diagnosis, the nurse should prepare to administer which medication? A) Vitamin K B) Cobalamin C) Heparin sodium D) Protamine sulfate

A) Coumadin is a Vitamin K antagonist anticoagulant that could cause excessive bleeding during surgery if clotting times are not corrected before surgery. For this reason, vitamin K is given as the antidote for warfarin (Coumadin).

After administering a dose of promethazine (Phenergan) to a patient with nausea and vomiting, the nurse explains that which of the following may be experienced as a common temporary adverse effect of the medication? A) Drowsiness B) Reduced hearing C) Sensation of falling D) Photosensitivity

A) Drowsiness (Although being given to this patient as an antiemetic, promethazine also has sedative and amnesic properties. For this reason, the patient is likely to experience drowsiness as an adverse effect of the medication.)

A patient has sought care because of a loss of 25 lb over the past 6 months, during which the patient claims to have made no significant dietary changes. What potential problem should the nurse assess the patient for? A) Thyroid disorders B) Diabetes insipidus C) Pituitary dysfunction D) Parathyroid dysfunction

A) Hyperthyroidism is associated with weight loss. Alterations in pituitary function, such as diabetes insipidus, and parathyroid dysfunction are not commonly associated with this phenomenon.

A patient with a history of peptic ulcer disease has presented to the emergency department with complaints of severe abdominal pain and a rigid, boardlike abdomen, prompting the health care team to suspect a perforated ulcer. Which of the following actions should the nurse anticipate? A) Providing IV fluids and inserting a nasogastric tube B) Administering oral bicarbonate and testing the patient's gastric pH level C) Performing a fecal occult blood test and administering IV calcium gluconate D) Starting parenteral nutrition and placing the patient in a high-Fowler's position

A) Providing IV fluids and inserting a nasogastric tube A perforated peptic ulcer requires IV replacement of fluid losses and continued gastric aspiration by NG tube. Nothing is given by mouth and gastric pH testing is not a priority. Calcium gluconate is not a medication directly relevant to the patient's suspected diagnosis and parenteral nutrition is not a priority in the short term.

The patient has CVI and a venous ulcer. The unlicensed assistive personnel (UAP) decides to apply compression stockings because that is what these patients always have ordered. What assessment by the nurse would cause the application of compression stockings to harm the patient? A) Rest pain B) High blood pressure C) Elevated blood sugar D) Dry, itchy, flaky skin

A) Rest pain occurs as peripheral artery disease (PAD) progresses and involves multiple arterial segments. Compression stockings should not be used on patients with PAD. Elevated blood glucose, possibly indicating uncontrolled diabetes mellitus, and hypertension may or may not indicate arterial problems. Dry, itchy, flaky skin indicates venous insufficiency. The RN should be the one to obtain the order and instruct the UAP to apply compression stockings if they are ordered.

When the patient is being examined for venous thromboembolism (VTE) in the calf, what diagnostic test should the nurse expect to teach the patient about first? A) Duplex ultrasound B) Contrast venography C) Magnetic resonance venography D) Computed tomography venography

A) The duplex ultrasound is the most widely used test to diagnose VTE. Contrast venography is rarely used now. Magnetic resonance venography is less accurate for calf veins than pelvic and proximal veins. Computed tomography venography may be used but is invasive and much more expensive than the duplex ultrasound.

When the nurse assesses the patient that has pancreatitis, what function may be altered related to the endocrine function of the pancreas? A) Blood glucose regulation B) Increased response to stress C) Fluid and electrolyte regulation D) Regulates metabolic rate of cells

A) The endocrine functions of the pancreas are regulated by α cells that produce and secrete glucagon, β cells that produce and secrete insulin and amylin, delta cells that produce and secrete somatostatin, and F cells that secrete pancreatic polypeptide. Glucagon, insulin, and amylin, and somatostatin all affect blood glucose. Pancreatic polypeptide regulates appetite. Increased response to stress occurs from epinephrine secreted by the adrenal medulla. Fluid and electrolyte regulation occurs in response to several hormones (mineralocorticoids, antidiuretic hormone, parathyroid hormone, calcitonin) from several organs (adrenal cortex, posterior pituitary, parathyroid, thyroid). The metabolic rate of cells is regulated by triiodothyronine (T3) from the thyroid.

The patient has been feeling tired lately and has gained weight; reports thickened, dry skin and increased cold sensitivity even though it is now summer. Which endocrine diagnostic test should be done first? A) Free thyroxine (FT4 ) B) Serum growth hormone (GH) C) Follicle stimulating hormone (FSH) D) Magnetic resonance imaging (MRI) of the head

A) The manifestations the patient is experiencing could be related to hypothyroidism. Free thyroxine (FT4) is considered a better indicator of thyroid function than total T4 and could be done to evaluate the patient for hypothyroidism. Growth hormone excess could cause thick, leathery, oily skin but does not demonstrate the other manifestations. FSH is manifest with menstrual irregularity and would be useful in distinguishing primary gonadal problems from pituitary insufficiency. MRI is the examination of choice for radiologic evaluation of the pituitary gland and the hypothalamus but would not be the first diagnostic study to further explore the basis of these manifestations.

The surgeon was unable to save a patient's parathyroid gland during a radical thyroidectomy. The nurse should consequently pay particular attention to which laboratory value? A) Calcium levels B) Potassium levels C) Blood glucose levels D) Sodium and chloride levels

A) The parathyroid gland plays a key role in maintaining calcium levels. Potassium, sodium, glucose, and chloride are not directly influenced by the loss of the parathyroid gland.

Assessment of a patient's peripheral IV site reveals that phlebitis has developed over the past several hours. Which intervention should the nurse implement first? A) Remove the patient's IV catheter. B) Apply an ice pack to the affected area. C) Decrease the IV rate to 20 to 30 mL/hr. D) Administer prophylactic anticoagulants.

A) The priority intervention for superficial phlebitis is removal of the offending IV catheter. Decreasing the IV rate is insufficient. Anticoagulants are not normally required, and warm, moist heat is often therapeutic.

The nurse is reviewing the laboratory test results for a 68-year-old patient whose warfarin (Coumadin) therapy was terminated during the preoperative period. The nurse concludes that the patient is in the most stable condition for surgery after noting which INR (international normalized ratio) result? A) 1.0 B) 1.8 C) 2.7 D) 3.4

A) The therapeutic range for INR is 2.0 to 3.0 for many clinical diagnoses. The larger the INR number, the greater the amount of anticoagulation. For this reason, the safest value before surgery is 1.0, meaning that the anticoagulation has been reversed.

When administered a dose of metoclopramide (Reglan), a patient complains of nausea. The nurse would teach the patient to report which of the following potential adverse effects? A) Tremors B) Constipation C) Double vision D) Numbness in the fingers and dose

A) Tremors (Extrapyramidal side effects, including tremors and dyskinesias, may occur as a result of metoclopramide (Reglan) administration.)

Which individuals would be at high risk for low back pain (select all that apply)? a.A 63-year-old man who is a long-distance truck driver b.A 36-year-old 6 ft, 2 in construction worker who weighs 260 lb c.A 28-year-old female yoga instructor who is 5 ft, 6 in and weighs 130 lb d.A 30-year-old male nurse who works on an orthopedic unit and smokes e.A 44-year-old female chef with prior compression fracture of the spine

A, B, D, E Risk factors associated with low back pain include a lack of muscle tone and excess body weight, stress, poor posture, cigarette smoking, pregnancy, prior compression fractures of the spine, spinal problems since birth, and a family history of back pain. Jobs that require repetitive heavy lifting, vibration (such as a jackhammer operator), and prolonged periods of sitting are also associated with low back pain. Low back pain is most often caused by a musculoskeletal problem. The causes of low back pain of musculoskeletal origin include (1) acute lumbosacral strain, (2) instability of the lumbosacral bony mechanism, (3) osteoarthritis of the lumbosacral vertebrae, (4) degenerative disc disease, and (5) herniation of an intervertebral disc. Health care personnel are at high risk for the development of low back pain. Lifting and moving patients, excessive time being stooped over or leaning forward, and frequent twisting can result in low back pain.

An acoustic neuroma is removed from a patient, and the nurse instructs the patient about tumor recurrence. What should the nurse instruct the patient to monitor (select all that apply)? A. Episodes of dizziness B. Lack of coordination C. Worsening of hearing D. Inability to close the eye E. Clear drainage from the nose

A, C, D An acoustic neuroma is a unilateral benign tumor that occurs where the vestibulocochlear nerve (CN VIII) enters the internal auditory canal. Clinical manifestations of tumor recurrence including facial nerve (CN VII) paralysis can be manifested by intermittent vertigo, hearing loss, and inability to close the eye. Lack of coordination and clear nasal drainage do not manifest with acoustic neuroma.

What medications should the nurse expect to include in the teaching plan to decrease the risk of cardiovascular events and death for PAD patients (select all that apply)? A) Ramipril (Altace) B) Cilostazol (Pletal) C) Simvastatin (Zocor) D) Clopidogrel (Plavix) E) Warfarin (Coumadin) F) Aspirin (acetylsalicylic acid)

A, F) Angiotensin-converting enzyme inhibitors (e.g., ramipril [Altace]) are used to control hypertension. Statins (e.g., simvastatin [Zocor]) are used for lipid management. Aspirin is used as an antiplatelet agent. Cilostazol (Pletal) is used for intermittent claudication, but it does not reduce CVD morbidity and mortality risks. Clopidogrel may be used if the patient cannot tolerate aspirin. Anticoagulants (e.g., warfarin [Coumadin]) are not recommended to prevent CVD events in PAD patients.

A therapeutic measure used to prevent hypertrophic scarring during the rehabilitative phase of burn recover is: A applying pressure garments B. repositioning the patient every 2 hours C. performing active ROM at least every 4 hours D. massaging the new tissue with water-based moisturizers

A, applying pressure garments

A 32-year-old patient is prescribed IV gentamicin (Garamycin) after repair of an intestinal perforation. The nurse should assess for which adverse effect of this medication? A. Hearing loss B. Exophthalmos C. Conjunctivitis D. Recurrent fever

A. Aminoglycosides such as gentamicin are drugs that are potentially ototoxic and may cause damage to the auditory nerve. When this drug is used, careful monitoring for hearing and balance problems (e.g., hearing loss, tinnitus, vertigo) is essential.

A college student has gone to the nurse complaining of eye pain after studying for finals. What assessment should the nurse make first in determining the possible etiology of this eye pain? A. Do you wear contacts? B. Do you have any allergies? C. Do you have double vision? D. Describe the change in your vision.

A. College students frequently wear contact lenses and will be up late or all night studying for finals. If the student wears contacts, the wearing of them while studying, care of them, and length of wear time will be assessed before looking for a corneal abrasion from extended wear with fluorescein dye. There are no manifestations of allergies, diplopia, or visual changes mentioned.

A 76-year-old male patient's wife is complaining that he is not being able to hear very well. He speaks with a quiet voice. With knowledge about geriatric changes in the auditory system, what should the nurse do first to determine the cause of the hearing loss? A. Look for cerumen in the ear. B. Assess for increased hair growth in the ear. C. Tell the patient it is probably related to aging. D. Ask the patient if he has fallen because of dizziness.

A. Gerontologic differences in the assessment of the auditory system include increased production of and drier cerumen, which can become impacted in the ear canal and contribute to hearing loss. Conductive hearing loss with impacted cerumen may lead to speaking softly as the patient's voice conducted through bone seems loud to the patient. Although increased hair growth occurs, it will not impact the hearing. Presbycusis may be occurring, but it should not be assumed. There is no reason to ask the patient if he has fallen because of dizziness, and vertigo is not a normal change of aging of the ear.

A 64-year-old female patient reporting frequent vertigo is scheduled for electronystagmography to test vestibular function. Which instructions should the nurse provide to the patient before the procedure? A. Eat a light meal before the procedure. B. Avoid carbonated beverages before the procedure. C. Take nothing by mouth for 3 hours before the procedure. D. No special dietary restrictions are needed until after the procedure.

A. Instruct patient to eat a light meal before the test to avoid nausea. Results of vestibular tests can be altered by use of caffeine, other stimulants, sedatives, and antivertigo drugs.

Which finding related to primary open-angle glaucoma would the nurse expect to find when reviewing a patient's history and physical examination report? A. Absence of pain or pressure B. Blurred vision in the morning C. Seeing colored halos around lights D. Eye pain accompanied with nausea and vomiting

A. Primary open-angle glaucoma is typically symptom-free, which explains why patients can have significant vision loss before a diagnosis is made unless regular eye examinations are being performed. Primary angle-closure glaucoma manifestations include sudden, excruciating pain in or around the eye, seeing colored halos around lights, and nausea and vomiting.

The patient has described a loss of central vision. What test should the nurse teach the patient about to identify changes in macular function? A. Amsler grid test B. B-scan ultrasonography C. Fluorescein angiography D. Intraocular pressure testing with Tono-pen

A. The Amsler grid test is self-administered and regular testing is necessary to identify any changes in macular function. B-scan ultrasonography is used to diagnose ocular pathologic conditions (e.g., intraocular foreign bodies or tumors, vitreous opacities, retinal detachments). Fluorescein angiography is used to diagnose problems related to the flow of blood through pigment epithelial and retinal vessels. Intraocular pressure testing with a Tono-pen is done to test for glaucoma.

Which instruction is most appropriate for a patient using contact lenses who is diagnosed with bacterial conjunctivitis? A. Discard all opened or used lens care products. B. Disinfect contact lenses by soaking in a cleaning solution for 48 hours. C. Put all used cosmetics in a plastic bag for 1 week to kill any bacteria before reusing. D. Disinfect all lens care products with the prescribed antibiotic drops for 1 week after infection.

A. The patient who wears contact lenses and develops infections should discard all opened or used lens care products and cosmetics to decrease the risk of reinfection from contaminated products. The risk of conjunctivitis is increased with not disinfecting lenses properly, wearing contact lenses too long, or using water or homemade solutions to store and clean lenses.

The nurse provides instructions to a 30-year-old female office worker who has low back pain. Which statement by the patient requires an intervention by the nurse? A. "Acupuncture to the lower back would cause irreparable nerve damage." B. "Smoking may aggravate back pain by decreasing blood flow to the spine." C. "Sleeping on my side with knees and hips bent reduces stress on my back." D. "Switching between hot and cold packs provides relief of pain and stiffness."

A. "Acupuncture to the lower back would cause irreparable nerve damage." Acupuncture is a safe therapy when the practitioner has been appropriately trained. Very fine needles are inserted into the skin to stimulate specific anatomic points in the body for therapeutic purposes.

A 54-year-old patient with acute osteomyelitis asks the nurse how this problem will be treated. Which response by the nurse is most appropriate? A. "IV antibiotics are usually required for several weeks." B. "Oral antibiotics are often required for several months." C. "Surgery is almost always necessary to remove the dead tissue that is likely to be present." D. "Drainage of the foot and instillation of antibiotics into the affected area is the usual therapy."

A. "IV antibiotics are usually required for several weeks." The standard treatment for acute osteomyelitis consists of several weeks of IV antibiotic therapy. This is because bone is denser and less vascular than other tissues, and it takes time for the antibiotic therapy to eradicate all of the microorganisms. Surgery may be used for chronic osteomyelitis, which may include debridement of the devitalized and infected tissue and irrigation of the affected bone with antibiotics.

For a 65-year-old female patient who has lived with a T1 spinal cord injury for 20 years, which health teaching information should you emphasize? A. A mammogram is needed every year. B. Bladder function tends to improve with age. C. Heart disease is not common in persons with spinal cord injury. D. As a person ages, the need to change body position is less important.

A. A mammogram is needed every year. Health promotion and screening are important for the older patient with a spinal cord injury. Older adult women with spinal cord injuries should perform monthly breast examinations and yearly mammograms

When teaching a patient about the most important respiratory defense mechanism distal to the respiratory bronchioles, which topic would the nurse discuss? A. Alveolar macrophages B. Impaction of particles C. Reflex bronchoconstriction D. Mucociliary clearance mechanism

A. Alveolar macrophages Respiratory defense mechanisms are efficient in protecting the lungs from inhaled particles, microorganisms, and toxic gases. Because ciliated cells are not found below the level of the respiratory bronchioles, the primary defense mechanism at the alveolar level is alveolar macrophages.

What is the best method to assess a patient with trigeminal neuralgia (tic douloureux)? A. Ask the patient what the triggering factors are. B. Have the patient open and close the jaw while palpating the mandible. C. Instruct the patient to touch the examiner's finger and then the patient's nose. D. Look at the optic disk with an ophthalmoscope

A. Ask the patient what the triggering factors are. The main symptom is spasms of severe, unilateral facial pain. Palpating the face may trigger a painful episode; touch is a common trigger. The other options do not reveal information about the trigeminal nerve effect.

The nurse admits a 55-year-old female with multiple sclerosis to a long-term care facility. Which finding is of most immediate concern to the nurse? A. Ataxic gait B. Radicular pain C. Severe fatigue D. Urinary retention

A. Ataxis gait An ataxic gait is a staggering, uncoordinated gait. Fall risk is the highest in individuals with gait instability or visual or cognitive impairments. The other signs and symptoms (e.g., fatigue, urinary retention, radicular pain) may also occur in the patient with multiple sclerosis and need to be managed, but are not the priority.

The nurse is obtaining a focused respiratory assessment of a 44-year-old female patient who is in severe respiratory distress 2 days after abdominal surgery. What is most important for the nurse to assess? A. Auscultation of bilateral breath sounds B. Percussion of anterior and posterior chest wall C. Palpation of the chest bilaterally for tactile fremitus D. Inspection for anterior and posterior chest expansion

A. Auscultation of bilateral breath sounds Important assessments obtained during a focused respiratory assessment include auscultation of lung (breath) sounds. Assessment of tactile fremitus has limited value in acute respiratory distress. It is not necessary to assess for both anterior and posterior chest expansion. Percussion of the chest wall is not essential in a focused respiratory assessment.

A frail 82-year-old female patient develops sudden shortness of breath while sitting in a chair. What location on the chest should the nurse begin auscultation of the lung fields? A. Bases of the posterior chest area B. Apices of the posterior lung fields C. Anterior chest area above the breasts D. Midaxillary on the left side of the chest

A. Bases of the posterior chest area Baseline data with the most information is best obtained by auscultation of the posterior chest, especially in female patients because of breast tissue interfering with the assessment or if the patient may tire easily (e.g., shortness of breath, dyspnea, weakness, fatigue). Usually auscultation proceeds from the lung apices to the bases unless it is possible the patient will tire easily. In this case the nurse should start at the bases.

The nurse is admitting a patient to the nursing unit with a history of a herniated lumbar disc and low back pain. In completing a more thorough pain assessment, the nurse should ask the patient if which action aggravates the pain? A. Bending or lifting B. Application of warm moist heat C. Sleeping in a side-lying position D. Sitting in a fully extended recliner

A. Bending or lifting Back pain that is related to a herniated lumbar disc often is aggravated by events and activities that increase the stress and strain on the spine, such as bending or lifting, coughing, sneezing, and lifting the leg with the knee straight (straight leg-raising test). Application of moist heat, sleeping position, and ability to sit in a fully extended recliner do not aggravate the pain of a herniated lumbar disc.

Which is most important to respond to in a patient presenting with a T3 spinal injury? A. Blood pressure of 88/60 mm Hg, pulse of 56 beats/minute B. Deep tendon reflexes of 1+, muscle strength of 1+ C. Pain rated at 9 D. Warm, dry skin

A. Blood pressure of 88/60 mm Hg, pulse of 56 beats/minute Neurogenic shock is a loss of vasomotor tone caused by injury, and it is characterized by hypotension and bradycardia. The loss of sympathetic nervous system innervations causes peripheral vasodilation, venous pooling, and a decreased cardiac output. The other options can be expected findings and are not as significant. Patients in neurogenic shock have pink and dry skin, instead of cold and clammy, but this sign is not as important as the vital signs.

Which clinical manifestation do you interpret as representing neurogenic shock in a patient with acute spinal cord injury? A. Bradycardia B. Hypertension C. Neurogenic spasticity D. Bounding pedal pulses

A. Bradycardia Neurogenic shock results from loss of vasomotor tone caused by injury and is characterized by hypotension and bradycardia. Loss of sympathetic innervation causes peripheral vasodilation, venous pooling, and a decreased cardiac output.

The nurse is caring for a patient hospitalized with exacerbation of chronic bronchitis and herniated lumbar disc. Which breakfast choice would be most appropriate for the nurse to encourage the patient to check on the breakfast menu? A. Bran muffin B. Scrambled eggs C. Puffed rice cereal D. Buttered white toast

A. Bran muffin Each meal should contain one or more sources of fiber, which will reduce the risk of constipation and straining with defecation, which increases back pain. Bran is typically a high-fiber food choice and is appropriate for selection from the menu. Scrambled eggs, puffed rice cereal, and buttered white toast do not have as much fiber.

A 50-year-old patient is reporting a sore shoulder after raking the yard. The nurse should suspect which problem? A. Bursitis B. Fasciitis C. Sprained ligament D. Achilles tendonitis

A. Bursitis Bursitis is common in adults over age 40 and with repetitive motion, such as raking. Plantar fasciitis frequently occurs as a stabbing pain at the heel caused by straining the ligament that supports the arch. Achilles tendonitis is an inflammation of the tendon that attaches the calf muscle to the heel bone, not the shoulder, and causes pain with walking or running. A sprained ligament occurs when a ligament is stretched or torn from a direct injury or sudden twisting of the joint, not repetitive motion.

What should the patient with Bell's palsy be cautioned against? A. Cornea dryness B. Driving while experiencing diplopia C. Sudden movement of the head when bending over D. Contamination from the affected eye to the other eye

A. Cornea dryness With Bell's palsy, the eyelid on the affected side often does not close tightly. Eye drops are used during the daytime, and patches are worn at night. The other options are not related to Bell's palsy.

Musculoskeletal assessment is an important component of care for patients on what type of long-term therapy? A. Corticosteroids B. β-Adrenergic blockers C. Antiplatelet aggregators D. Calcium-channel blockers

A. Corticosteroids Corticosteroids are associated with avascular necrosis and decreased bone and muscle mass. β-blockers, calcium-channel blockers, and antiplatelet aggregators are not commonly associated with damage to the musculoskeletal system.

The patient is calling the clinic with a cough. What assessment should be made first before the nurse advises the patient? A. Cough sound, sputum production, pattern B. Frequency, a family history, hematemesis C. Smoking, medications, residence location D. Weight loss, activity tolerance, orthopnea

A. Cough sound, sputum production, pattern The sound of the cough, sputum production and description, as well as pattern of the cough's occurrence (including acute or chronic) and what its occurrence is related to are the first assessments to be made to determine the severity. Frequency of the cough will not provide a lot of information. Family history can help to determine a genetic cause of the cough. Hematemesis is vomiting blood and not as important as hemoptysis. Smoking is an important risk factor for COPD and lung cancer and may cause a cough. Medications may or may not contribute to a cough as does residence location. Weight loss, activity intolerance, and orthopnea may be related to respiratory or cardiac problems, but are not as important when dealing with a cough.

What is the classic presentation of botulism? A. Descending flaccid paralysis B. Ascending bilateral paralysis C. Homonymous hemianopsia D. Vomiting and diarrhea

A. Descending flaccid paralysis Neurologic manifestations can develop rapidly or evolve over several days. They include the development of a descending flaccid paralysis with intact sensation, photophobia, ptosis, paralysis of extraocular muscles, blurred vision, diplopia, dry mouth, sore throat, and difficulty in swallowing. Ascending bilateral paralysis is associated with Guillain-Barré syndrome. Homonymous hemianopsia, the loss of one half of the field of vision, can occur in a stroke

When assessing a patient's sleep-rest pattern related to respiratory health, what should the nurse ask the patient about (select all that apply)? A. Have trouble falling asleep? B. Need to urinate during the night? C. Awaken abruptly during the night? D. Sleep more than 8 hours per night? E. Need to sleep with the head elevated?

A. Have trouble falling asleep? C. Awaken abruptly during the night? E. Need to sleep with the head elevated? The patient with sleep apnea may have insomnia and/or abrupt awakenings. Patients with cardiovascular disease (e.g., heart failure that may affect respiratory health) may need to sleep with the head elevated on several pillows (orthopnea). Sleeping more than 8 hours per night or needing to urinate during the night is not indicative of impaired respiratory health.

Which signs and symptoms in a patient with a T4 spinal cord injury should alert you to the possibility of autonomic dysreflexia? A. Headache and rising blood pressure B. Irregular respirations and shortness of breath C. Decreased level of consciousness or hallucinations D. Abdominal distention and absence of bowel sounds

A. Headache and rising blood pressure Among the manifestations of autonomic dysreflexia are hypertension (up to 300 mm Hg systolic) and throbbing headache. Respiratory manifestations, decreased level of consciousness, and gastrointestinal manifestations are not characteristic.

When working with patients, the nurse knows that patients have the most difficulties with diarthrodial joints. Which joints are included in this group of joints? (Select all that apply.) A. Hinge joint of the knee B. Ligaments joining the vertebrae C. Fibrous connective tissue of the skull D. Ball and socket joint of the shoulder or hip E. Cartilaginous connective tissue of the pubis joint

A. Hinge joint of the knee D. Ball and socket joint of the shoulder or hip The diarthrodial joints include the hinge joint of the knee and elbow, the ball and socket joint of the shoulder and hip, the pivot joint of the radioulnar joint, and the condyloid, saddle, and gliding joints of the wrist and hand. The ligaments and cartilaginous connective tissue joining the vertebrae and pubis joint and the fibrous connective tissue of the skull are synarthrotic joints.

During assessment of the patient with trigeminal neuralgia, what should you do (select all that apply)? A. Inspect all aspects of the mouth and teeth. B. Assess the gag reflex and respiratory rate and depth. C. Lightly palpate the affected side of the face for edema. D. Test for temperature and sensation perception on the face. E. Ask the patient to describe factors that initiate an episode.

A. Inspect all aspects of the mouth and teeth. D. Test for temperature and sensation perception on the face. E. Ask the patient to describe factors that initiate an episode. Assessment of the attacks, including the triggering factors, characteristics, frequency, and pain management techniques, helps you plan patient care. The painful episodes are usually initiated by a triggering mechanism of light cutaneous stimulation at a specific point (trigger zone) along the distribution of the nerve branches. Precipitating stimuli include chewing, tooth brushing, a hot or cold blast of air on the face, washing the face, yawning, and talking. Touch and tickle seem to predominate as causative triggers, rather than pain or changes in temperature.

A patient with recurrent shortness of breath has just had a bronchoscopy. What is a priority nursing action immediately following the procedure? A. Monitor the patient for laryngeal edema. B. Assess the patient's level of consciousness. C. Monitor and manage the patient's level of pain. D. Assess the patient's heart rate and blood pressure.

A. Monitor the patient for laryngeal edema. Priorities for assessment are the patient's airway and breathing, both of which may be compromised after bronchoscopy by laryngeal edema. These assessment parameters supersede the importance of loss of consciousness (LOC), pain, heart rate, and blood pressure, although the nurse should also be assessing these.

The patient with Parkinson's disease has a pulse oximetry reading of 72%, but he is not displaying any other signs of decreased oxygenation. What is most likely contributing to his low SpO2 level? A. Motion B. Anemia C. Dark skin color D. Thick acrylic nails

A. Motion Motion is the most likely cause of the low SpO2 for this patient with Parkinson's disease. Anemia, dark skin color, and thick acrylic nails as well as low perfusion, bright fluorescent lights, and intravascular dyes may also cause an inaccurate pulse oximetry result. There is no mention of these or reason to suspect these in this question.

The nurse is performing a musculoskeletal assessment of an 81-year-old female patient whose mobility has been progressively decreasing in recent months. How should the nurse best assess the patient's range of motion (ROM) in the affected leg? A. Observe the patient's unassisted ROM in the affected leg. B. Perform passive ROM, asking the patient to report any pain. C. Ask the patient to lift progressive weights with the affected leg. D. Move both of the patient's legs from a supine position to full flexion.

A. Observe the patient's unassisted ROM in the affected leg. Passive ROM should be performed with extreme caution and may be best avoided when assessing older patients. Observing the patient's active ROM is more accurate and safe than asking the patient to lift weights with her legs.

Which nursing intervention is most appropriate when turning a patient following spinal surgery? A. Placing a pillow between the patient's legs and turning the body as a unit B. Having the patient turn to the side by grasping the side rails to help turn over C. Elevating the head of bed 30 degrees and having the patient extend the legs while turning D. Turning the patient's head and shoulders and then the hips, keeping the patient's body centered in the bed

A. Placing a pillow between the patient's legs and turning the body as a unit Placing a pillow between the legs and turning the patient as a unit (logrolling) helps to keep the spine in good alignment and reduces pain and discomfort following spinal surgery. Having the patient turn by grasping the side rail to help, elevating the head of the bed, and turning with extended legs or turning the patient's head and shoulders and then the hips will not maintain proper spine alignment and may cause damage.

The patient's arterial blood gas results show the PaO2 at 65 mmHg and the SaO2 at 80%. What early manifestations should the nurse expect to observe in this patient? A. Restlessness, tachypnea, tachycardia, and diaphoresis B. Unexplained confusion, dyspnea at rest, hypotension, and diaphoresis C. Combativeness, retractions with breathing, cyanosis, and decreased output D. Coma, accessory muscle use, cool and clammy skin, and unexplained fatigue

A. Restlessness, tachypnea, tachycardia, and diaphoresis With inadequate oxygenation, early manifestations include restlessness, tachypnea, tachycardia, and diaphoresis, decreased urinary output, and unexplained fatigue. The unexplained confusion, dyspnea at rest, hypotension, and diaphoresis; combativeness, retractions with breathing, cyanosis, and decreased urinary output; coma, accessory muscle use, cool and clammy skin, and unexplained fatigue occur as later manifestations of inadequate oxygenation.

What is a classic manifestation of tetanus infection? A. Rigidity and seizures B. Bloody diarrhea and vomiting C. Pulmonary edema D. Sepsis

A. Rigidity and seizures The manifestations are mainly neurologic. They include stiffness in the jaw (trismus), generalized tonic convulsions, and opisthotonos. Laryngeal and respiratory spasms can also occur. The other options are not related to tetanus.

The nurse is preparing the patient for a diagnostic procedure to remove pleural fluid for analysis. The nurse would prepare the patient for which test? A. Thoracentesis B. Bronchoscopy C. Pulmonary angiography D. Sputum culture and sensitivity

A. Thoracentesis Thoracentesis is the insertion of a large-bore needle through the chest wall into the pleural space to obtain specimens for diagnostic evaluation, remove pleural fluid, or instill medication into the pleural space.

During the assessment in the ED, the nurse is palpating the patient's chest. Which finding is a medical emergency? A. Trachea moved to the left B. Increased tactile fremitus C. Decreased tactile fremitus D. Diminished chest movement

A. Trachea moved to the left Tracheal deviation is a medical emergency when it is caused by a tension pneumothorax. Tactile fremitus increases with pneumonia or pulmonary edema and decreases in pleural effusion or lung hyperinflation. Diminished chest movement occurs with barrel chest, restrictive disease, and neuromuscular disease.

Which intervention should you perform in the acute care of a patient with autonomic dysreflexia? A. Urinary catheterization B. Administration of benzodiazepines C. Suctioning of the patient's upper airway D. Placement of the patient in the Trendelenburg position

A. Urinary catheterization Because the most common cause of autonomic dysreflexia is bladder irritation, immediate catheterization to relieve bladder distention may be necessary. The patient should be positioned upright. Benzodiazepines are contraindicated, and suctioning is likely unnecessary.

A student nurse asks the RN what can be measured by arterial blood gases (ABGs). The RN tells the student that the ABGs can measure (select all that apply). A. acid-base balance. B. oxygenation status. C. acidity of the blood. D. glucose bound to hemoglobin. E. bicarbonate (HCO3-) in arterial blood.

A. acid-base balance. B. oxygenation status. C. acidity of the blood. E. bicarbonate (HCO3-) in arterial blood. Arterial blood gases (ABGs) are measured to determine oxygenation status and acid-base balance. ABG analysis includes measurement of the PaO2, the partial pressure of carbon dioxide in arterial blood (PaCO2), acidity (pH), and bicarbonate (HCO3-) in arterial blood.

When assessing a patient with a partial-thickness burn, the nurse would expect to find (select all that apply): A. blisters B. exposed fascia C. exposed muscles D. intact nerve endings E. red, shiny, wet appearance

A. blisters D. intact nerve endings E. red, shiny, wet appearance

When assessing activity-exercise patterns related to respiratory health, the nurse inquires about: A. dyspnea during rest or exercise. B. recent weight loss or weight gain. C. ability to sleep through the entire night. D. willingness to wear oxygen equipment in public.

A. dyspnea during rest or exercise. In this functional health pattern, determine whether the patient's activity is limited by dyspnea at rest or during exercise.

The injury the is least likely to result in a full-thickness burn is: A. sunburn B. scald injury C. chemical burn D. electrical burn

A. sunburn

To promote the release of surfactant, the nurse encourages the patient to: A. take deep breaths. B. cough five times per hour to prevent alveolar collapse. C. decrease fluid intake to reduce fluid accumulation in the alveoli. D. sit with head of bed elevated to promote air movement through the pores of Kohn.

A. take deep breaths. Surfactant is a lipoprotein that lowers the surface tension in the alveoli. It reduces the amount of pressure needed to inflate the alveoli and decreases the tendency of the alveoli to collapse. Deep breaths stretch the alveoli and promote surfactant secretion.

Despite a high dosage, a male patient who is taking nifedipine (Procardia XL) for antihypertensive therapy continues to have blood pressures over 140/90 mmHg. What should the nurse do next? A: Assess his adherence to therapy. B: Ask him to make an exercise plan. C: Instruct him to use the DASH diet. D: Request a prescription for a thiazide diuretic.

A: Assess his adherence to therapy. A long-acting calcium-channel blocker such as nifedipine causes vascular smooth muscle relaxation resulting in decreased SVR and arterial BP and related side effects. The patient data the nurse has about this patient is very limited, so the nurse needs to assess his adherence to therapy.

The nurse teaches a patient with hypertension that uncontrolled hypertension may damage organs in the body primarily by which mechanism? A: Hypertension promotes atherosclerosis and damage to the walls of the arteries. B: Hypertension causes direct pressure on organs, resulting in necrosis and replacement of cells with scar tissue. C: Hypertension causes thickening of the capillary membranes, leading to hypoxia of organ systems. D: Hypertension increases blood viscosity, which contributes to intravascular coagulation and tissue necrosis distal to occlusions.

A: Hypertension promotes atherosclerosis and damage to the walls of the arteries. Hypertension is a major risk factor for the development of atherosclerosis by mechanisms not yet fully known. However, once atherosclerosis develops, it damages the walls of arteries and reduces circulation to target organs and tissues.

The nurse is caring for a patient with hypertension who is scheduled to receive a dose of esmolol (Brevibloc). The nurse should withhold the dose and consult the prescribing physician for which vital sign taken just before administration? A:Pulse 48 B: Respirations 24 C: Blood pressure 118/74 D: Oxygen saturation 93%

A:Pulse 48 Because esmolol is a β1-adrenergic blocking agent, it can cause hypotension and bradycardia as adverse effects. The nurse should withhold the dose and consult with the health care provider for parameters regarding pulse rate limits.

A 54-year-old patient admitted with diabetes mellitus, malnutrition, osteomyelitis, and alcohol abuse has a serum amylase level of 280 U/L and a serum lipase level of 310 U/L. To what diagnosis does the nurse attribute these findings? Malnutrition Osteomyelitis Alcohol abuse Diabetes mellitus

Alcohol abuse The patient with alcohol abuse could develop pancreatitis as a complication, which would increase the serum amylase (normal 30-122 U/L) and serum lipase (normal 31-186 U/L) levels as shown.

A 38-year-old female is admitted for an elective surgical procedure. Which information obtained by the nurse during the preoperative assessment is most important to report to the anesthesiologist before surgery? a. The patient's lack of knowledge about postoperative pain control measures b. The patient's statement that her last menstrual period was 8 weeks previously c. The patient's history of a postoperative infection following a prior cholecystectomy d. The patient's concern that she will be unable to care for her children postoperatively

B

A patient who has diabetes and uses insulin to control blood glucose has been NPO since midnight before having a knee replacement surgery. Which action should the nurse take? a. Withhold the usual scheduled insulin dose because the patient is NPO. b. Obtain a blood glucose measurement before any insulin administration. c. Give the patient the usual insulin dose because stress will increase the blood glucose. d. Administer a lower dose of insulin because there will be no oral intake before surgery.

B

A patient who has never had any prior surgeries tells the nurse doing the preoperative assessment about an allergy to bananas and avocados. Which action is most important for the nurse to take? a. Notify the dietitian about the food allergies. b. Alert the surgery center about a possible latex allergy. c. Reassure the patient that all allergies are noted on the medical record. d. Ask whether the patient uses antihistamines to reduce allergic reactions.

B

Five minutes after receiving the ordered preoperative midazolam (Versed) by IV injection, the patient asks to get up to go to the bathroom to urinate. Which action by the nurse is most appropriate? a. Assist the patient to the bathroom and stay with the patient to prevent falls. b. Offer a urinal or bedpan and position the patient in bed to promote voiding. c. Allow the patient up to the bathroom because medication onset is 10 minutes. d. Ask the patient to wait because catheterization is performed just before the surgery.

B

The nurse is preparing to witness the patient signing the operative consent form when the patient says, "I do not really understand what the doctor said." Which action is best for the nurse to take? a. Provide an explanation of the planned surgical procedure. b. Notify the surgeon that the informed consent process is not complete. c. Administer the prescribed preoperative antibiotics and withhold any ordered sedative medications. d. Notify the operating room staff that the surgeon needs to give a more complete explanation of the procedure.

B

Which information in the preoperative patient's medication history is most important to communicate to the health care provider? a. The patient uses acetaminophen (Tylenol) occasionally for aches and pains. b. The patient takes garlic capsules daily but did not take any on the surgical day. c. The patient has a history of cocaine use but quit using the drug over 10 years ago. d. The patient took a sedative medication the previous night to assist in falling asleep.

B

The results of a patient's recent endoscopy indicate the presence of peptic ulcer disease (PUD). Which of the following teaching points should the nurse provide to the patient in light of his new diagnosis? A) "You'll need to drink at least two to three glasses of milk daily." B) "It would likely be beneficial for you to eliminate drinking alcohol." C) "Many people find that a minced or pureed diet eases their symptoms of PUD." D) "Your medications should allow you to maintain your present diet while minimizing symptoms."

B) "It would likely be beneficial for you to eliminate drinking alcohol." Although there is no specific recommended dietary modification for PUD, most patients find it necessary to make some sort of dietary modifications to minimize symptoms. Milk may exacerbate PUD and alcohol is best avoided because it can delay healing.

Which assessment parameter is of highest priority when caring for a patient undergoing a water deprivation test? A) Serum glucose B) Patient weight C) Arterial blood gases D) Patient temperature

B) A patient is at risk for severe dehydration during a water deprivation test. The test should be discontinued and the patient rehydrated if the patient's weight drops more than 2 kg at any time. The other assessment parameters do not assess fluid balance.

The nurse is providing discharge instructions to a patient with diabetes insipidus. Which instructions regarding desmopressin acetate (DDAVP) would be most appropriate? A) The patient can expect to experience weight loss resulting from increased diuresis. B) The patient should alternate nostrils during administration to prevent nasal irritation. C) The patient should monitor for symptoms of hypernatremia as a side effect of this drug. D) The patient should report any decrease in urinary elimination to the health care provider.

B) DDAVP is used to treat diabetes insipidus by replacing the antidiuretic hormone that the patient is lacking. Inhaled DDAVP can cause nasal irritation, headache, nausea, and other signs of hyponatremia. Diuresis will be decreased and is expected, and hypernatremia should not occur.

The nurse is caring for a patient treated with intravenous fluid therapy for severe vomiting. As the patient recovers and begins to tolerate oral intake, the nurse understands that which of the following food choices would be most appropriate? A) Ice tea B) Dry toast C) Warm broth D) Plain hamburger

B) Dry toast (Dry toast or crackers may alleviate the feeling of nausea and prevent further vomiting. Extremely hot or cold liquids and fatty foods are generally not well tolerated.)

A nurse is caring for a patient with a diagnosis of deep venous thrombosis (DVT). The patient has an order to receive 30 mg enoxaparin (Lovenox). Which injection site should the nurse use to administer this medication safely? A) Buttock, upper outer quadrant B) Abdomen, anterior-lateral aspect C) Back of the arm, 2 inches away from a mole D) Anterolateral thigh, with no scar tissue nearby

B) Enoxaparin (Lovenox) is a low-molecular-weight (LMW) heparin that is given as a deep subcutaneous injection in the right and left anterolateral abdomen. All subcutaneous injections should be given away from scars, lesions, or moles.

3.The patient who is admitted with a diagnosis of diverticulitis and a history of irritable bowel disease and gastroesophageal reflux disease (GERD) has received a dose of Mylanta 30 ml PO. The nurse would evaluate its effectiveness by questioning the patient as to whether which of the following symptoms has been resolved? A) Diarrhea B) Heartburn C) Constipation D) Lower abdominal pain

B) Heartburn (Mylanta is an antacid that contains both aluminum and magnesium. It is indicated for the relief of GI discomfort, such as with heartburn associated with GERD.)

A 62-year-old Hispanic male patient with diabetes mellitus has been diagnosed with peripheral artery disease (PAD). The patient is a smoker and has a history of gout. What should the nurse focus her teaching on to prevent complications for this patient? A) Gender B) Smoking C) Ethnicity D) Co-morbidities

B) Smoking is the most significant factor for this patient. PAD is a marker of advanced systemic atherosclerosis. Therefore tobacco cessation is essential to reduce PAD progression, CVD events, and mortality. Diabetes mellitus and hyperuricemia are also risk factors. Being male or Hispanic are not risk factors for PAD.

The nurse is caring for a newly admitted patient with vascular insufficiency. The patient has a new order for enoxaparin (Lovenox) 30 mg subcutaneously. What should the nurse do to correctly administer this medication? A) Spread the skin before inserting the needle. B) Leave the air bubble in the prefilled syringe. C) Use the back of the arm as the preferred site. D) Sit the patient at a 30-degree angle before administration.

B) The nurse should not expel the air bubble from the prefilled syringe because it should be injected to clear the needle of medication and avoid leaving medication in the needle track in the tissue.

The patient is brought to the ED following a car accident and is wearing medical identification that says she has Addison's disease. What should the nurse expect to be included in the collaborative care of this patient? A) Low sodium diet B) Increased glucocorticoid replacement C) Suppression of pituitary ACTH synthesis D) Elimination of mineralocorticoid replacement

B) The patient with Addison's disease needs lifelong glucocorticoid and mineralocorticoid replacement and has an increased need with illness, injury, or stress, as this patient is experiencing. The patient with Addison's may also need a high sodium diet. Suppression of pituitary ACTH synthesis is done for Cushing syndrome. Elimination of mineralocorticoid replacement cannot be done for Addison's disease.

A patient with varicose veins has been prescribed compression stockings. How should the nurse teach the patient to use these? A) "Try to keep your stockings on 24 hours a day, as much as possible." B) "While you're still lying in bed in the morning, put on your stockings." C) "Dangle your feet at your bedside for 5 minutes before putting on your stockings." D) "Your stockings will be most effective if you can remove them for a few minutes several times a day."

B) The patient with varicose veins should apply stockings in bed, before rising in the morning. Stockings should not be worn continuously, but they should not be removed several times daily. Dangling at the bedside prior to application is likely to decrease their effectiveness.

A patient was just diagnosed with acute arterial ischemia in the left leg secondary to atrial fibrillation. Which early clinical manifestation must be reported to the physician immediately to save the patient's limb? A) Paralysis B) Paresthesia D) Crampiness E) Referred pain

B) The physician must be notified immediately if any of the six Ps of acute arterial ischemia occur to prevent ischemia from quickly progressing to tissue necrosis and gangrene. The six Ps are paresthesia, pain, pallor, pulselessness, and poikilothermia, with paralysis being a very late sign indicating the death of nerves to the extremity. Crampy leg sensation is more common with varicose veins. The pain is not referred.

The patient had aortic aneurysm repair. What priority nursing action will the nurse use to maintain graft patency? Assess output for renal dysfunction. Use IV fluids to maintain adequate BP. Use oral antihypertensives to maintain cardiac output. Maintain a low BP to prevent pressure on surgical site

B) The priority is to maintain an adequate BP (determined by the surgeon) to maintain graft patency. A prolonged low BP may result in graft thrombosis, and hypertension may cause undue stress on arterial anastomoses resulting in leakage of blood or rupture at the suture lines, which is when IV antihypertensives may be used. Renal output will be assessed when the aneurysm repair is above the renal arteries to assess graft patency, not maintain it.

A female patient with critical limb ischemia has had peripheral artery bypass surgery to improve her circulation. What care should the nurse provide on postoperative day 1? A) Keep the patient on bed rest. B) Assist the patient with walking several times. C) Have the patient sit in the chair several times. D) Place the patient on her side with knees flexed.

B) To avoid blockage of the graft or stent, the patient should walk several times on postoperative day 1 and subsequent days. Having the patient's knees flexed for sitting in a chair or in bed increase the risk of venous thrombosis and may place stress on the suture lines.

The patient with systemic lupus erythematosus had been diagnosed with syndrome of inappropriate antidiuretic hormone (SIADH). What should the nurse expect to include in this patient's plan of care (select all that apply)? A) Obtain weekly weights. B) Limit fluids to 1000 mL per day.* C) Monitor for signs of hypernatremia. D) Minimize turning and range of motion. E) Keep the head of the bed at 10 degrees or less elevation.

B, E) The care for the patient with SIADH will include limiting fluids to 1000 mL per day or less to decrease weight, increase osmolality, and improve symptoms; and keeping the head of the bed elevated at 10 degrees or less to enhance venous return to the heart and increase left atrial filling pressure, thereby reducing the release of ADH. The weights should be done daily along with intake and output. Signs of hyponatremia should be monitored, and frequent turning, positioning, and range-of-motion exercises are important to maintain skin integrity and joint mobility.

The patient calls the clinic about a "sty" that she has had for some time on her upper eyelid. She says she has used warm moist compresses, but it is no better. What should the nurse tell her to do? A. "Go to the pharmacy to get some eye drops." B. "Come in so the ophthalmologist can remove the lesion for you." C. "The health care provider will need to inject it with an antibiotic." D. "Wash the lid margins with baby shampoo to remove the crusting."

B. A chalazion may evolve from a "sty" or hordeolum as it did for this patient. Initial treatment is with warm compresses, but when they are ineffective, the lesion may be surgically removed or injected with corticosteroids. Washing the lid margins with baby shampoo is done with blepharitis.

The patient with Ménière's disease had decompression of the endolymphatic sac to reduce the frequent and incapacitating attacks she was experiencing. What should the nurse include in the discharge teaching for this patient? A. Airplane travel will be more comfortable now. B. Avoid sudden head movements or position changes. C. Cough or blow the nose to keep the Eustachian tube clear. D. Take antihistamines, antiemetics, and sedatives for recovery.

B. After ear surgery the patient should avoid sudden head movements or position changes. The patient should not cough or blow the nose because this increases pressure in the Eustachian tube and middle ear cavity and may disrupt healing. Airplane travel should be avoided at first as increased pressure and ear popping is normally experienced, which will disrupt healing. Antihistamines, antiemetics, and sedatives are used to decrease the symptoms of acute attacks of Ménière's disease

Otoscopic examination of the patient's left ear indicates the presence of an exostosis. The nurse recognizes that this finding will likely be followed up by what? A. Surgery B. No intervention C. Electrocochleography D. Irrigation of the ear canal

B. An exostosis is a bony growth into the ear canal that normally does not require intervention or correction.

The triage nurse at an ambulatory clinic receives a call from an individual with possible metal fragments in both eyes. Which instructions would the nurse provide for emergency care of this possible eye injury? A. "Remove any visible metal fragments." B. "Apply a loose dressing over your eyes." C. "Rinse your eyes immediately with water." D. "Keep your eyes open to allow tears to form."

B. An initial intervention for a penetrating eye injury includes covering the eye(s) with a dry, sterile patch and protective shield. The fragments should not be removed by the individual or others. Penetrating eye injuries should not be irrigated (only irrigate for chemical eye injuries).

A 28-year-old female patient complains of intermittent eye dryness. Which question should the nurse ask the patient to determine the etiology of this symptom? A. "Do you take ginkgo to treat asthma or tinnitus"? B. "What do you take if you have allergy symptoms"? C. "Are you taking propranolol for an anxiety disorder"? D. "How long have you been taking prednisone (Deltasone)"?

B. Anithistamines or decongestants taken for allergies or colds can cause ocular dryness. Ginkgo biloba is an herbal product and has been used to treat asthma and tinnitus. Side effects of ginkgo may include headache, nausea, gastrointestinal upset, diarrhea, dizziness, allergic skin reactions, and increased bleeding. β-adrenergic blockers can potentiate drugs used to treat glaucoma. Long-term use of prednisone (corticosteroids) may contribute to the development of glaucoma or cataracts.

A patient has severe myopia. Which type of correction is the patient planning to have if she tells you, "I can't wait to be able to see after they implant a contact lens over my lens"? A. Photorefractive keratectomy (PRK) B. Phakic intraocular lenses (phakic IOLs) C. Refractive intraocular lens (refractive IOL) D. Laser-assisted in situ keratomileusis (LASIK)

B. Phakic intraocular lenses (phakic IOLs) is the implantation of a contact lens in front of the natural lens. PRK is used with low to moderate amounts of myopia, and the epithelium is removed and the laser sculpts the cornea to correct the refractive error. Refractive IOL is also for patients with a high degree of myopia or hyperopia and involves removing the natural lens and implanting an intraocular lens. LASIK surgery is similar to PRK except that the epithelium is replaced after surgery.

You are providing care for a 73-year-old male patient who has sought care because of a loss in his hearing acuity over the past several years. Which statement by the nurse is most accurate? A. "This is often due to an infection that will resolve on its own." B. "Many people experience an age-related decline in their hearing." C. "This is likely an effect of your medications. Try stopping them for a few days." D. "You can likely accommodate for your hearing loss with a few small changes in your routine."

B. Presbycusis is a loss of hearing that is both common and age-related. Infections are most often accompanied by different symptoms. It would be inappropriate to counsel the patient to stop his medications. It would be simplistic to advise the patient to accommodate the hearing loss rather than seek intervention.

When examining the patient's ear with an otoscope, there is discharge in the canal and the patient noted pain with the examination. What should the nurse next assess the patient for? A. Sebaceous cyst B. Swimmer's ear C. Metabolic disorder D. Serous otitis media

B. Swimmer's ear or an infection of the external ear is probably the cause of the discharge and pain. Asking the patient about swimming, ear protection, and exposure to types of water can identify contact with contaminated water. After clearing the discharge, the tympanic membrane can be assessed for otitis media. A sebaceous cyst and metabolic disorders would not cause drainage or discomfort in the external ear canal.

When using the otoscope, the nurse is unable to see the landmarks or light reflex of the tympanic membrane. The tympanic membrane is bulging and red. What does the nurse think is most likely occurring in the patient's ear? A. Swimmer's ear B. Acute otitis media C. Impacted cerumen D. Chronic otitis media

B. The manifestations of inability to see the landmarks or light reflex of the tympanic membrane and the bulging and redness of the tympanic membrane are those of acute otitis media. With swimmer's ear and chronic otitis media, there is frequently drainage in the external auditory canal. Impacted cerumen would block the visualization of the tympanic membrane.

The nurse is examining a 20-year-old woman's ear in the student health clinic to determine if recent treatment for acute otitis media has been effective. Which assessment finding indicates resolution of the middle ear infection? A. Fenestrations are visible in the tympanic membrane. B. Tympanic membrane is gray, shiny, and translucent. C. Cone of light is not visible on the tympanic membrane. D. Tympanic membrane is blue and bulging with no landmarks.

B. The tympanic membrane (TM) is normally pearly gray, white or pink, shiny, and translucent. Perforation of the TM that has not healed will appear as open areas of the tympanic membrane. The absence of the cone of light indicates a retracted TM. A bulging red or blue eardrum and lack of landmarks indicates a fluid-filled middle ear. The fluid may be pus or blood.

When teaching a patient about the pathophysiology related to open-angle glaucoma, which statement is most appropriate? A. "The retinal nerve is damaged by an abnormal increase in the production of aqueous humor." B. "There is decreased draining of aqueous humor in the eye, causing pressure damage to the optic nerve." C. "The lens enlarges with normal aging, pushing the iris forward, which then covers the outflow channels of the eye." D. "There is a decreased flow of aqueous humor into the anterior chamber by the lens of the eye blocking the papillary opening."

B. With primary open-angle glaucoma, there is increased intraocular pressure because the aqueous humor cannot drain from the eye. This leads to damage to the optic nerve over time. Primary angle-closure glaucoma is caused by the lens bulging forward and blocking the flow of aqueous humor into the anterior chamber.

What aspects of the patient's medical history are most likely to have potential consequences for the patient's visual system? A. Hypothyroidism and polycythemia B. Hypertension and diabetes mellitus C. Atrial fibrillation and atherosclerosis D. Vascular dementia and chronic fatigue

B. Hypertension and diabetes frequently contribute to visual pathologies. The other cited health problems are less likely to have a direct, deleterious effect on a patient's vision.

A patient has ptosis resulting from myasthenia gravis. Which assessment finding would the nurse expect to see in this patient? A. Redness and swelling of the conjunctiva B. Drooping of the upper lid margin in one or both eyes C. Redness, swelling, and crusting along the lid margins D. Small, superficial white nodules along the lid margin

B. Ptosis is the term used to describe drooping of the upper lid margin, which may be either unilateral or bilateral. Ptosis can be a result of mechanical causes, such as an eyelid tumor or excess skin, or from myogenic causes such as myasthenia gravis.

A 42-year-old man who is scheduled for an arthrocentesis arrives at the outpatient surgery unit and states, "I do not want this procedure done today." Which response by the nurse is most appropriate? A. "When would you like to reschedule the procedure?" B. "Tell me what your concerns are about this procedure." C. "The procedure is safe, so why should you be worried?" D. "The procedure is not painful because an anesthetic is used."

B. "Tell me what your concerns are about this procedure." The nurse should use therapeutic communication to determine the patient's concern about the procedure. The nurse should not provide false reassurance. It is not appropriate for the nurse to assume the patient is concerned about pain or to assume the patient is asking to reschedule the procedure.

A 57-year-old postmenopausal woman is scheduled for dual-energy x-ray absorptiometry (DXA). Which statement, if made by the patient to the nurse, indicates understanding of the procedure? A. "The bone density in my heel will be measured." B. "This procedure will not cause any pain or discomfort." C. "I will not be exposed to any radiation during the procedure." D. "I will need to remove my hearing aids before the procedure."

B. "This procedure will not cause any pain or discomfort." Dual-energy x-ray absorptiometry (DXA) is painless and measures the bone mass of spine, femur, forearm, and total body with minimal radiation exposure. A quantitative ultrasound (QUS) evaluates density, elasticity, and strength of bone using ultrasound of the calcaneus (heel). Magnetic resonance imaging would require removal of objects such as hearing aids that have metal parts.

The nurse is caring for a patient with chronic obstructive pulmonary disorder (COPD) and pneumonia who has an order for arterial blood gases to be drawn. What is the minimum length of time the nurse should plan to hold pressure on the puncture site? A. 2 minutes B. 5 minutes C. 10 minutes D. 15 minutes

B. 5 minutes After obtaining blood for an arterial blood gas measurement, the nurse should hold pressure on the puncture site for 5 minutes by the clock to be sure that bleeding has stopped. An artery is an elastic vessel under much higher pressure than veins, and significant blood loss or hematoma formation could occur if the time is insufficient.

Which patient is exhibiting an early clinical manifestation of hypoxemia? A. A 48-year-old patient who is intoxicated and acutely disoriented to time and place B. A 72-year-old patient who has four new premature ventricular contractions per minute C. A 67-year-old patient who has dyspnea while resting in the bed or in a reclining chair D. A 94-year-old patient who has renal insufficiency, anemia, and decreased urine output

B. A 72-year-old patient who has four new premature ventricular contractions per minute Early clinical manifestations of hypoxemia include dysrhythmias (e.g., premature ventricular contractions), unexplained decreased level of consciousness (e.g., disorientation), dyspnea on exertion, and unexplained decreased urine output.

During a health screening event which assessment finding would alert the nurse to the possible presence of osteoporosis in a white 61-year-old female? A. The presence of bowed legs B. A measurable loss of height C. Poor appetite and aversion to dairy products D. Development of unstable, wide-gait ambulation

B. A measurable loss of height A gradual but measurable loss of height and the development of kyphosis or "dowager's hump" are indicative of the presence of osteoporosis in which the rate of bone resorption is greater than bone deposition. Bowed legs may be caused by abnormal bone development or rickets but is not indicative of osteoporosis. Lack of calcium and Vitamin D intake may cause osteoporosis but are not indicative it is present. A wide gait is used to support balance and does not indicate osteoporosis.

A female patient with a long-standing history of rheumatoid arthritis has sought care because of increasing stiffness in her right knee that has culminated in complete fixation of the joint. The nurse would document the presence of which problem? A. Atrophy B. Ankylosis C. Crepitation D. Contracture

B. Ankylosis Ankylosis is stiffness or fixation of a joint, whereas contracture is reduced movement as a consequence of fibrosis of soft tissue (muscles, ligaments, or tendons). Atrophy is a flabby appearance of muscle leading to decreased function and tone. Crepitation is a grating or crackling sound that accompanies movement.

Which is a classic presentation of Guillain-Barré syndrome? A. Acute change in level of consciousness B. Ascending, symmetric paralysis C. Acute onset of paralysis in lower extremities D. Paresthesias in legs starting with feet and radiating to groin area

B. Ascending, symmetric paralysis Guillain-Barré syndrome is an acute, rapidly progressing polyneuritis with ascending, symmetric paralysis. The other options are not related to Guillain-Barré syndrome.

The patient had abdominal surgery yesterday. Today the lung sounds in the lower lobes have decreased. The nurse knows this could be due to what occurring? A. Pain B. Atelectasis C. Pneumonia D. Pleural effusion

B. Atelectasis Postoperatively there is an increased risk for atelectasis from anesthesia as well as restricted breathing from pain. Without deep breathing to stretch the alveoli, surfactant secretion to hold the alveoli open is not promoted. Pneumonia will occur later after surgery. Pleural effusion occurs because of blockage of lymphatic drainage or an imbalance between intravascular and oncotic fluid pressures, which is not expected in this case

What is the most common early symptom of a spinal cord tumor? A. Urinary incontinence B. Back pain that worsens with activity C. Paralysis below the level of involvement D. Impaired sensation of pain, temperature, and light touch

B. Back pain that worsens with activity The most common early symptom of a spinal cord tumor outside the cord is pain in the back, with radicular pain simulating intercostal neuralgia, angina, or herpes zoster infection. The location of the pain depends on the level of compression. The pain worsens with activity, coughing, straining, and lying down.

What is a common treatment for trigeminal neuralgia? A. Warm, moist compressions B. Carbamazepine (Tegretol) C. Ice packs applied intermittently D. Vitamin D

B. Carbamazepine (Tegretol) Carbamazepine (Tegretol) or oxcarbazepine (Trileptal) is the usual first-line therapy for trigeminal neuralgia. By acting on sodium channels, these drugs lengthen the time needed for neuron repolarization and decrease neuron firing. Some patients take megavitamins as an adjunct therapy. Temperature extremes are often a trigger for painful episodes.

The 24-year-old male patient who was successfully treated for Paget's disease has come to the clinic with a gradual onset of pain and swelling around the left knee. The patient is diagnosed with osteosarcoma without metastasis. The patient wants to know why he will be given chemotherapy before the surgery. What is the best rationale the nurse should tell the patient? A. The chemotherapy is being used to save your left leg. B. Chemotherapy is being used to decrease the tumor size. C. The chemotherapy will increase your 5-year survival rate. D. Chemotherapy will help decrease the pain before and after surgery.

B. Chemotherapy is being used to decrease the tumor size. Preoperative chemotherapy is used to decrease tumor size before surgery. The chemotherapy will not save his leg if the lesion is too big or there is neurovascular or muscle involvement. Adjunct chemotherapy after amputation or limb salvage has increased 5-year survival rate in people without metastasis. Chemotherapy is not used to decrease pain before or after surgery.

What is the most common cause of botulism? A. Contamination from Escherichia coli from improper hand washing B. Contamination from spores from improper home canning C. Dairy foods kept at room temperature D. Eating undercooked poultry

B. Contamination from spores from improper home canning Botulism is caused by gastrointestinal absorption of the neurotoxin produced by Clostridium botulinum. The organism is found in the soil and can grow in any food contaminated with the spores. Improper home canning of foods is often the cause. The other options are not related to botulism.

What is the main source of tetanus in the U.S. population? A. Tic bites B. Deep, penetrating wounds C. Unprotected sex D. Improperly prepared food

B. Deep, penetrating wounds Tetanus results from a potent neurotoxin released by an anaerobic bacillus, Clostridium tetani. The spores enter the body through a traumatic or suppurative wound. The most common source in the United States is deep, penetrating wounds or intravenous drug use. The other options are not related to tetanus.

The patient is diagnosed with Guillain-Barré syndrome and admitted to the inpatient unit from the emergency department. What is the most important nursing observation? A. Urine output B. Depth of respiration C. Bowel sounds D. Lower extremity strength

B. Depth of respiration The most serious complication is paralysis progressing to the nerves that innervate the thoracic area and causing respiratory failure. You must constantly monitor the respiratory system by checking respiratory rate and depth, forced vital capacity, and negative inspiratory force. The other options may be affected, but respiratory function is most important.

The nurse identifies a nursing diagnosis of pain related to muscle spasms for a 45-year-old patient who has low back pain from a herniated lumbar disc. What would be an appropriate nursing intervention to treat this problem? A. Provide gentle ROM to the lower extremities. B. Elevate the head of the bed 20 degrees and flex the knees. C. Place the bed in reverse Trendelenburg with the feet firmly against the footboard. D. Place a small pillow under the patient's upper back to gently flex the lumbar spine.

B. Elevate the head of the bed 20 degrees and flex the knees. The nurse should elevate the head of the bed 20 degrees and flex the knees to avoid extension of the spine and increasing the pain. The slight flexion provided by this position often is comfortable for a patient with a herniated lumbar disc. ROM to the lower extremities will be limited to prevent extremes of spinal movement. Reverse Trendelenburg and a pillow under the patient's upper back will more likely increase pain.

What are the goals of rehabilitation for the patient with an injury at the C6 level (select all that apply)? A. Stand erect with leg brace B. Feed self with hand devices C. Drive an electric wheelchair D. Assist with transfer activities E. Drive adapted van from wheel chair

B. Feed self with hand devices C. Drive an electric wheelchair D. Assist with transfer activities E. Drive adapted van from wheel chair Rehabilitation goals for a patient with a spinal cord injury at the C6 level include ability to assist with transfer and perform some self-care; feed self with hand devices; push wheelchair on smooth, flat surface; drive adapted van from wheelchair; independent computer use with adaptive equipment; and needing attendant care only for 6 hours per day.

The nurse is caring for a patient admitted to the nursing unit with osteomyelitis of the tibia. Which symptom will the nurse most likely find on physical examination of the patient? A. Nausea and vomiting B. Localized pain and warmth C. Paresthesia in the affected extremity D. Generalized bone pain throughout the leg

B. Localized pain and warmth Osteomyelitis is an infection of bone and bone marrow that can occur with trauma, surgery, or spread from another part of the body. Because it is an infection, the patient will exhibit typical signs of inflammation and infection, including localized pain and warmth. Nausea and vomiting and paresthesia of the extremity are not expected to occur. Pain occurs, but it is localized, not generalized throughout the leg.

The home care nurse visits an 84-year-old woman with pneumonia after her discharge from the hospital. Which assessment finding would the nurse expect because of age-related changes in the musculoskeletal system? A. Positive straight-leg-raising test B. Muscle strength is scale grade 3/5 C. Lateral S-shaped curvature of the spine D. Fingers drift to the ulnar side of the forearm

B. Muscle strength is scale grade 3/5 Decreased muscle strength is an age-related change of the musculoskeletal system caused by decreased number and size of the muscle cells. The other assessment findings indicate musculoskeletal abnormalities. A positive straight-leg-raising test indicates nerve root irritation from intervertebral disk prolapse and herniation. An ulnar deviation or drift indicates rheumatoid arthritis due to tendon contracture. Scoliosis is a lateral curvature of the spine.

In reviewing bone remodeling, what should the nurse know about the involvement of bone cells? A. Osteoclasts add canaliculi. B. Osteoblasts deposit new bone. C. Osteocytes are mature bone cells. D. Osteons create a dense bone structure.

B. Osteoblasts deposit new bone. Bone remodeling is achieved when osteoclasts remove old bone and osteoblasts deposit new bone. Osteocytes are mature bone cells, and osteons or Haversian systems create a dense bone structure, but these are not involved with bone remodeling.

After assisting at the bedside with a thoracentesis, the nurse should continue to assess the patient for signs and symptoms of what? A. Bronchospasm B. Pneumothorax C. Pulmonary edema D. Respiratory acidosis

B. Pneumothorax Because thoracentesis involves the introduction of a catheter into the pleural space, there is a risk of pneumothorax. Thoracentesis does not carry a significant potential for causing bronchospasm, pulmonary edema, or respiratory acidosis.

You are caring for a patient admitted with a spinal cord injury after a motor vehicle accident. The patient exhibits a complete loss of motor, sensory, and reflex activity below the injury level. What is this condition? A. Central cord syndrome B. Spinal shock syndrome C. Anterior cord syndrome D. Brown-Séquard syndrome

B. Spinal shock syndrome About 50% of people with acute spinal cord injury experience a temporary loss of reflexes, sensation, and motor activity that is known as spinal shock. Central cord syndrome is manifested by motor and sensory loss greater in the upper extremities than the lower extremities. Anterior cord syndrome results in motor and sensory loss but not loss of reflexes. Brown-Séquard syndrome is characterized by ipsilateral loss of motor function and contralateral loss of sensory function.

You suspect Bell's palsy in which patient? A. Unilateral facial droop with contralateral extremity weakness B. Sudden onset one-sided facial weakness with ear pain and vesicles C. Sharp, knife-like facial pain when eating hot or cold foods D. Inability to shrug the shoulders against resistance

B. Sudden onset one-sided facial weakness with ear pain and vesicles Bell's palsy is an acute, peripheral facial paresis of unknown cause without systemic effects. Facial droop is found in stroke. Sharp facial pain occurs with trigeminal neuralgia. An inability to shrug the shoulders describes pathology of cranial nerve XI.

One month after a spinal cord injury, which finding is most important for you to monitor? A. Bladder scan indicates 100 mL. B. The left calf is 5 cm larger than the right calf. C. The heel has a reddened, nonblanchable area. D. Reflux bowel emptying.

B. The left calf is 5 cm larger than the right calf. Deep vein thrombosis is a common problem accompanying spinal cord injury during the first 3 months. Pulmonary embolism is one of the leading causes of death. Common signs and symptoms are absent. Assessment includes Doppler examination and measurement of leg girth. The other options are not as urgent to deal with as potential deep vein thrombosis.

In assessment of the patient with acute respiratory distress, what should the nurse expect to observe (select all that apply)? A. Cyanosis B. Tripod position C. Kussmaul respirations D. Accessory muscle use E. Increased AP diameter

B. Tripod position D. Accessory muscle use Tripod position and accessory muscle use indicate moderate to severe respiratory distress. Cyanosis may be related to anemia, decreased oxygen transfer in the lungs, or decreased cardiac output. Therefore it is a nonspecific and unreliable indicator of only respiratory distress. Kussmaul respirations occur when the patient is in metabolic acidosis to increase CO2 excretion. Increased AP diameter occurs with lung hyperinflation from COPD, cystic fibrosis, or with advanced age.

The nurse is planning health promotion teaching for a 45-year-old patient with asthma, low back pain from herniated lumbar disc, and schizophrenia. What does the nurse determine would be the best exercise to include in an individualized exercise plan for the patient? A. Yoga B. Walking C. Calisthenics D. Weight lifting

B. Walking The patient would benefit from an aerobic exercise that takes into account the patient's health status and fits the patient's lifestyle. The best exercise is walking, which builds strength in the back and leg muscles without putting undue pressure or strain on the spine. Yoga, calisthenics, and weight lifting would all put pressure on or strain the spine.

To detect early signs of symptoms of inadequate oxygenation, the nurse would examine the patient for: A. dyspnea and hypotension. B. apprehension and restlessness. C. cyanosis and cool, clammy skin. D. increased urine output and diaphoresis.

B. apprehension and restlessness. Early symptoms of inadequate oxygenation include unexplained restlessness, apprehension, and irritability.

When auscultating the chest of an older patient in respiratory distress, it is best to: A. begin listening at the apices. B. begin listening at the lung bases. C. begin listening on the anterior chest. D. ask the patient to breathe through the nose with the mouth closed.

B. begin listening at the lung bases. Normally, auscultation should proceed from the lung apices to the bases, so that opposite areas of the chest are compared. If the patient is likely to tire easily or has respiratory distress, start at the bases.

To maintain a positive nitrogen balance in a major burn, the patient must: A. increase normal caloric intake by about 3 times B. eat a high-protein, low-fat, high-carbohydrate diet C. eat at least 1500 calories per day in small, frequent meals D. eat rice and whole wheat for the chemical effect on nitrogen balance

B. eat a high-protein, low-fat, high-carbohydrate diet

A patient has 25% TBSA burned from a car fire. His wounds have been debrided and covered with a silver-impregnated dressing. The nurse's priority intervention for wound care would be to: A. reapply a new dressing without disturbing the wound bed B. observe the wound for signs of infection during dressing changes C. apply cool compresses for pain relief in between dressing changes D. wash the wound aggressively with soap and water three times a day

B. the patient has as much control over the management of the pain as possible

Pain management for the burn patient is most effective when: A. opioids are administered on a set schedule around the clock B. the patient has as much control over the management of the pain as possible C. there is a flexibility to administer opioids within a dosage and frequency range D. painful dressing changes are delayed until the patient's pain is totally relieved

B. the patient has as much control over the management of the pain as possible

The nurse is caring for a patient admitted with a history of hypertension. The patient's medication history includes hydrochlorothiazide (Hydrodiuril) daily for the past 10 years. Which parameter would indicate the optimal intended effect of this drug therapy? A: Weight loss of 2 lb B: Blood pressure 128/86 C: Absence of ankle edema D: Output of 600 mL per 8 hours

B: Blood pressure 128/86 Hydrochlorothiazide may be used alone as monotherapy to manage hypertension or in combination with other medications if not effective alone. After the first few weeks of therapy, the diuretic effect diminishes, but the antihypertensive effect remains. Since the patient has been taking this medication for 10 years, the most direct measurement of its intended effect would be the blood pressure.

When teaching a patient about dietary management of stage 1 hypertension, which instruction is most appropriate? A: Restrict all caffeine. B: Restrict sodium intake. C: Increase protein intake. D: Use calcium supplements.

B: Restrict sodium intake. The patient should decrease intake of sodium. This will help to control hypertension, which can be aggravated by excessive salt intake, which in turn leads to fluid retention. Caffeine and protein intake do not affect hypertension. Calcium supplements are not recommended to lower BP.

When providing dietary instruction to a patient with hypertension, the nurse would advise the patient to restrict intake of which meat? A: Broiled fish B: Roasted duck C: Roasted turkey D: Baked chicken breast

B: Roasted duck Roasted duck is high in fat, which should be avoided by the patient with hypertension. Weight loss may slow the progress of atherosclerosis and overall CVD risk. The other meats are lower in fat and are therefore acceptable in the diet.

When assessing the patient for orthostatic hypotension, after taking the blood pressure (BP) and pulse (P) in the supine position, what should the nurse do next? A: Repeat BP and P in this position. B: Take BP and P with patient sitting. C: Record the BP and P measurements. D: Take BP and P with patient standing.

B: Take BP and P with patient sitting. When assessing for orthostatic changes in BP after measuring BP in the supine position, the patient is placed in a sitting position and BP is measured within 1 to 2 minutes and then repositioned to the standing position with BP measured again, within 1 to 2 minutes. The results are then recorded with a decrease of 20 mm Hg or more in SBP, a decrease of 10 mm Hg or more in DBP, and/or an increase in pulse of greater than or equal to 20 beats/minute from supine to standing indicating orthostatic hypotension.

A patient undergoing an emergency appendectomy has been using St. John's wort to prevent depression. Which complication would the nurse expect in the postanesthesia care unit? a. Increased pain b. Hypertensive episodes c. Longer time to recover from anesthesia d. Increased risk for postoperative bleeding

C

The nurse plans to provide preoperative teaching to an alert older man who has hearing and vision deficits. His wife usually answers most questions that are directed to the patient. Which action should the nurse take when doing the teaching? a. Use printed materials for instruction so that the patient will have more time to review the material. b. Direct the teaching toward the wife because she is the obvious support and caregiver for the patient. c. Provide additional time for the patient to understand preoperative instructions and carry out procedures. d. Ask the patient's wife to wait in the hall in order to focus preoperative teaching with the patient himself.

C

Which topic is most important for the nurse to discuss preoperatively with a patient who is scheduled for abdominal surgery for an open cholecystectomy? a. Care for the surgical incision b. Medications used during surgery c. Deep breathing and coughing techniques d. Oral antibiotic therapy after discharge home

C

When caring for a preoperative patient on the day of surgery, which actions included in the plan of care can the nurse delegate to unlicensed assistive personnel (UAP)? (Select all that apply.) a. Teach incentive spirometer use. b. Explain preoperative routine care. c. Obtain and document baseline vital signs. d. Remove nail polish and apply pulse oximeter. e. Transport the patient by stretcher to the operating room.

C D E

What is a nursing priority in the care of a patient with a diagnosis of hypothyroidism? A) Providing a dark, low-stimulation environment B) Closely monitoring the patient's intake and output C) Patient teaching related to levothyroxine (Synthroid) D) Patient teaching related to radioactive iodine therapy

C) A euthyroid state is most often achieved in patients with hypothyroidism by the administration of levothyroxine (Synthroid). It is not necessary to carefully monitor intake and output, and low stimulation and radioactive iodine therapy are indicated in the treatment of hyperthyroidism.

When instructing a patient regarding a urine study for free cortisol, what is most important for the nurse to tell the patient? A) Save the first voided urine in the morning. B) Maintain a high-sodium diet 3 days before collection. C) Try to avoid stressful situations during the collection period. D) Complete at least 30 minutes of exercise before collecting the urine sample.

C) A urine study for free cortisol requires a 24-hour urine collection. The patient should be instructed to avoid stressful situations and excessive physical exercise that could unduly increase cortisol levels. The patient should also maintain a low-sodium diet before and during the urine collection period.

The nurse would determine that a postoperative patient is not receiving the beneficial effects of enoxaparin (Lovenox) after noting what during a routine shift assessment? A) Generalized weakness and fatigue B) Crackles bilaterally in the lung bases C) Pain and swelling in lower extremity D) Abdominal pain with decreased bowel sounds

C) Enoxaparin is a low-molecular-weight heparin used to prevent the development of deep vein thromboses (DVTs) in the postoperative period. Pain and swelling in the lower extremity can indicate development of DVT and therefore may signal ineffective medication therapy.

A postoperative patient asks the nurse why the physician ordered daily administration of enoxaparin (Lovenox). Which reply by the nurse is most appropriate? A) "This medication will help prevent breathing problems after surgery, such as pneumonia." B) "This medication will help lower your blood pressure to a safer level, which is very important after surgery." C) "This medication will help prevent blood clots from forming in your legs until your level of activity, such as walking, returns to normal." D) "This medication is a narcotic pain medication that will help take away any muscle aches caused by positioning on the operating room table."

C) Enoxaparin is an anticoagulant that is used to prevent DVTs postoperatively. All other explanations/options do not describe the action/purpose of enoxaparin.

The nurse determines that a patient has experienced the beneficial effects of medication therapy with famotidine (Pepcid) when which of the following symptoms is relieved? A) Nausea B) Belching C) Epigastric pain D) Difficulty swallowing

C) Epigastric pain Famotidine is an H2-receptor antagonist that inhibits parietal cell output of HCl acid and minimizes damage to gastric mucosa related to hyperacidity, thus relieving epigastric pain.

The nurse is caring for a patient admitted with suspected hyperparathyroidism. Because of the potential effects of this disease on electrolyte balance, the nurse should assess this patient for what manifestation? A) Neurologic irritability B) Declining urine output C) Lethargy and weakness D) Hyperactive bowel sounds

C) Hyperparathyroidism can cause hypercalcemia. Signs of hypercalcemia include muscle weakness, polyuria, constipation, nausea and vomiting, lethargy, and memory impairment. Neurologic irritability, declining urine output, and hyperactive bowel sounds do not occur with hypercalcemia.

The surgeon was unable to spare a patient's parathyroid gland during a thyroidectomy. Which assessments should the nurse prioritize when providing postoperative care for this patient? A) Assessing the patient's white blood cell levels and assessing for infection B) Monitoring the patient's hemoglobin, hematocrit, and red blood cell levels C) Monitoring the patient's serum calcium levels and assessing for signs of hypocalcemia D) Monitoring the patient's level of consciousness and assessing for acute delirium or agitation

C) Loss of the parathyroid gland is associated with hypocalcemia. Infection and anemia are not associated with loss of the parathyroid gland, whereas cognitive changes are less pronounced than the signs and symptoms of hypocalcemia.

The patient receiving chemotherapy rings the call bell and reports an onset of nausea. The nurse should prepare a prn dose of which of the following medications? A) Morphine sulfate B) Zolpidem (Ambien) C) Ondansetron (Zofran) D) Dexamethasone (Decadron)

C) Ondansetron (Zofran) (Ondansetron is a 5-HT3 receptor antagonist antiemetic that is especially effective in reducing cancer chemotherapy-induced nausea and vomiting.)

Following administration of a dose of metoclopramide (Reglan) to the patient, the nurse determines that the medication has been effective when which of the following is noted? A) Decreased blood pressure B) Absence of muscle tremors C) Relief of nausea and vomiting D) No further episodes of diarrhea

C) Relief of nausea and vomiting. (Metoclopramide is classified as a prokinetic and antiemetic medication. If it is effective, the patient's nausea and vomiting should resolve).

A patient has been taking oral prednisone for the past several weeks after having a severe reaction to poison ivy. The nurse has explained the procedure for gradual reduction rather than sudden cessation of the drug. What is the rationale for this approach to drug administration? A) Prevention of hypothyroidism B) Prevention of diabetes insipidus C) Prevention of adrenal insufficiency D) Prevention of cardiovascular complications

C) Sudden cessation of corticosteroid therapy can precipitate life-threatening adrenal insufficiency. Diabetes insipidus, hypothyroidism, and cardiovascular complications are not common consequences of suddenly stopping corticosteroid therapy.

The nurse is caring for a preoperative patient who has an order for vitamin K by subcutaneous injection. The nurse should verify that which laboratory study is abnormal before administering the dose? A) Hematocrit (Hct) B) Hemoglobin (Hgb) C) Prothrombin time (PT) D) Partial thromboplastin time (PTT)

C) Vitamin K counteracts hypoprothrombinemia and/or reverses the effects of warfarin (Coumadin) and thus decreases the risk of bleeding. High values for either the prothrombin time (PT) or the international normalized ratio (INR) demonstrates the need for this medication.

The nurse is caring for a patient who has been receiving warfarin (Coumadin) and digoxin (Lanoxin) as treatment for atrial fibrillation. Because the warfarin has been discontinued before surgery, the nurse should diligently assess the patient for which complication early in the postoperative period until the medication is resumed? A) Decreased cardiac output B) Increased blood pressure C) Cerebral or pulmonary emboli D) Excessive bleeding from incision or IV sites

C) Warfarin is an anticoagulant that is used to prevent thrombi from forming on the walls of the atria during atrial fibrillation. Once the medication is terminated, thrombi could again form. If one or more thrombi detach from the atrial wall, they could travel as cerebral emboli from the left atrium or pulmonary emboli from the right atrium.

A 44-year-old woman who works in a noisy factory reports being off balance when standing or walking but not while lying down. What term will the nurse use to document this patient's symptoms? A. Vertigo B. Syncope C. Dizziness D. Nystagmus

C. Dizziness is a sensation of being off balance that occurs when standing or walking; it does not occur when lying down. Nystagmus is an abnormal eye movement that may be observed as a twitching of the eyeball or described by the patient as a blurring of vision with head or eye movement. Vertigo is a sense that the person or objects around the person are moving or spinning and is usually stimulated by movement of the head. Syncope is a brief lapse in consciousness accompanied by a loss in postural tone (fainting).

When planning care for a patient with disturbed sensory perception related to increased intraocular pressure caused by primary open-angle glaucoma, what should the nurse focus on? A. Recognizing that eye damage caused by glaucoma can be reversed in the early stages B. Giving anticipatory guidance about the eventual loss of central vision that will occur C. Encouraging compliance with drug therapy for the glaucoma to prevent loss of vision D. Managing the pain experienced by patients with glaucoma that persists until the optic nerve atrophies

C. Drug therapy is necessary to prevent the eventual vision loss that may occur as a consequence of glaucoma. For this reason, encourage the patient to remain compliant with drug therapy.

A 64-year-old man newly diagnosed with glaucoma asks the nurse what has made the pressure in his eyes so high. Which is the nurse's most accurate response? A. Back pressure from cardiac congestion causes corneal edema. B. Cerebral venous dilation prevents normal interstitial fluid resorption. C. Increased production of aqueous humor or blocked drainage increases pressure. D. Congenital anomalies of the lacrimal gland or duct obstruct the passage of tears.

C. Intraocular pressure is increased in glaucoma as a result of excess aqueous humor production or decreased outflow. Cardiac or cerebral circulation changes do not cause glaucoma. Lacrimal anomalies do not affect aqueous humor production.

Which statement is most appropriate when teaching a patient about timolol (Timoptic) eye drops in the treatment of glaucoma? A. "You may feel some palpitations after instilling these eye drops." B. "You should withhold this medication if your blood pressure becomes elevated." C. "You may have some temporary blurring of vision after instilling these eye drops." D. "You should keep your eyes closed for 15 minutes after instilling these eye drops."

C. It is common for patients to have a temporary blurring of vision for a few minutes after instilling eye drops. This should not cause concern to the patient. Because timolol is a β-blocker, heart rate may slow, and blood pressure is more likely to decrease if absorbed systemically.

The nurse is providing discharge teaching to a 48-year-old woman with type 2 diabetes after a scleral buckling procedure. Which statement, if made by the patient, indicates that the discharge teaching is effective? A. "I doubt my other eye will ever be affected." B. "I can expect severe pain after this procedure." C. "I should avoid lifting heavy objects and straining." D. "The procedure will correct my vision immediately."

C. Patients should avoid heavy lifting (over 20 lb) and straining. A patient with a detached retina is at risk for detachment of the other retina. Patients usually have little to no discomfort after scleral buckling. Severe, persistent pain should be reported immediately to the health care provider. Vision is restored in about 90% of retinal detachments. Vision will not be restored immediately and takes days to weeks to improve.

During a health history, a 43-year-old teacher complains of increasing difficulty reading printed materials for the past year. What change related to aging does the nurse suspect? A. Myopia B. Hyperopia C. Presbyopia D. Astigmatism

C. Presbyopia is a loss of accommodation causing an inability to focus on near objects. This occurs as a normal part of aging process starting around age 40. Myopia is nearsightedness (near objects are clear and far objects are blurred). Astigmatism results in visual distortion related to unevenness in the cornea. Hyperopia is farsightedness (near objects are blurred and far objects are clearly seen).

During the course of an interview to assess vision, a patient complains of dry eyes. What should the nurse implement next? A. Assess for contact lenses B. Suggest saline eye drops C. Check the medication list D. Ask about eyeglass usage

C. The nurse should evaluate the patient's medication list to identify agents that can contribute to dry eyes so follow-up nursing care can be planned. Dry eyes aggravate wearing contact lenses but contact lenses do not normally cause dry eyes. The nurse should not suggest saline eye drops until the etiology of the dry eyes is determined. Eyeglasses do not cause dry eyes.

A male patient is recovering from a motor vehicle accident that left him blind. He is withdrawn and refuses to get out of bed. What is the nurse's priority goal for this patient? A. Use suitable coping strategies to reduce stress. B. Identify patient's strengths and support system. C. Verbalize feelings related to visual impairment. D. Transition successfully to the sudden vision loss.

C. The nurse's priority is to help the patient express his feelings about his vision loss because he is not coping effectively with his situation. Until the patient expresses how he feels, he will be unable to progress in his rehabilitation process.

When assessing an adult patient's external ear canal and tympanum, what should the nurse do? A. Ask the patient to tip his or her head toward the nurse. B. Identify a pearl gray tympanic membrane as a sign of infection. C. Gently pull the auricle up and backward to straighten the canal. D. Identify a normal light reflex by the appearance of irregular edges.

C. When examining a patient's external ear canal and tympanum, ask the patient to tilt the head toward the opposite shoulder. Grasp and gently pull the auricle up and backward to straighten the canal. A healthy, normal tympanic membrane will appear pearl gray, white, or pink and have a cone-shaped light reflex.

When administering eye drops to a patient with glaucoma, which nursing measure is most appropriate to minimize systemic effects of the medication? A. Apply pressure to each eyeball for a few seconds after administration. B. Have the patient close the eyes and move them back and forth several times. C. Have the patient put pressure on the inner canthus of the eye after administration. D. Have the patient try to blink out excess medication immediately after administration.

C. Systemic absorption can be minimized by applying pressure to the inner canthus of the eye. The other options will not minimize systemic effects of the medication.

The nurse is admitting a patient who complains of a new onset of lower back pain. To differentiate between the pain of a lumbar herniated disc and lower back pain from other causes, what would be the best question for the nurse to ask the patient? A. "Is the pain worse in the morning or in the evening?" B. "Is the pain sharp or stabbing or burning or aching?" C. "Does the pain radiate down the buttock or into the leg?" D. "Is the pain totally relieved by analgesics, such as acetaminophen (Tylenol)?"

C. "Does the pain radiate down the buttock or into the leg?" Lower back pain associated with a herniated lumbar disc is accompanied by radiation along the sciatic nerve and can be commonly described as traveling through the buttock, to the posterior thigh, or down the leg. This is because the herniated disc causes compression on spinal nerves as they exit the spinal column. Time of occurrence, type of pain, and pain relief questions do not elicit differentiating data.

A 67-year-old male patient had a right total knee replacement 2 days ago. Upon auscultation of the patient's posterior chest, the nurse detects discontinuous, high-pitched breath sounds just before the end of inspiration in the lower portion of both lungs. Which statement most appropriately reflects how the nurse should document the breath sounds? A. "Bibasilar rhonchi present on inspiration." B. "Diminished breath sounds in the bases of both lungs." C. "Fine crackles posterior right and left lower lung fields." D. "Expiratory wheezing scattered throughout the lung fields."

C. "Fine crackles posterior right and left lower lung fields." Fine crackles are described as a series of short-duration, discontinuous, high-pitched sounds heard just before the end of inspiration.

The nurse has reviewed proper body mechanics with a patient with a history of low back pain caused by a herniated lumbar disc. Which statement made by the patient indicates a need for further teaching? A. "I should sleep on my side or back with my hips and knees bent." B. "I should exercise at least 15 minutes every morning and evening." C. "I should pick up items by leaning forward without bending my knees." D. "I should try to keep one foot on a stool whenever I have to stand for a period of time."

C. "I should pick up items by leaning forward without bending my knees." The patient should avoid leaning forward without bending the knees. Bending the knees helps to prevent lower back strain and is part of proper body mechanics when lifting. Sleeping on the side or back with hips and knees bent and standing with a foot on a stool will decrease lower back strain. Back strengthening exercises are done twice a day once symptoms subside.

The nurse receives report from the licensed practical nurse about care provided to patients on the orthopedic surgical unit. It is most important for the nurse to follow up on which statement? A. "The patient who had a spinal fusion 12 hours ago has hypoactive bowel sounds and is not passing flatus." B. "The patient who had cervical spine surgery 2 days ago wants to wear her soft cervical collar when out of bed." C. "The patient who had spinal surgery 3 hours ago is complaining of a headache and has clear drainage on the dressing." D. "The patient who had a laminectomy 24 hours ago is using patient-controlled analgesia with morphine for pain management."

C. "The patient who had spinal surgery 3 hours ago is complaining of a headache and has clear drainage on the dressing." After spinal surgery there is potential for cerebrospinal fluid (CSF) leakage. Severe headache or leakage of CSF (clear or slightly yellow) on the dressing should be reported immediately. The drainage is CSF if a dipstick test is positive for glucose. Patients after spinal surgery may experience paralytic ileus and interference with bowel function for several days. Postoperatively most patients require opioids such as morphine IV for 24 to 48 hours. Patient-controlled analgesia is the preferred method for pain management during this time. After cervical spine surgery patients often wear a soft or hard cervical collar to immobilize the neck.

The nurse is interpreting a tuberculin skin test (TST) for a 58-year-old female patient with end-stage kidney disease secondary to diabetes mellitus. Which finding would indicate a positive reaction? A. Acid-fast bacilli cultured at the injection site B. 15-mm area of redness at the TST injection site C. 11-mm area of induration at the TST injection site D. Wheal formed immediately after intradermal injection

C. 11-mm area of induration at the TST injection site An area of induration ≥ 10 mm would be a positive reaction in a person with end-stage kidney disease. Reddened, flat areas do not indicate a positive reaction. A wheal appears when the TST is administered that indicates correct administration of the intradermal antigen. Presence of acid-fast bacilli in the sputum indicates active tuberculosis.

A 54-year-old patient admitted with cellulitis and probable osteomyelitis received an injection of radioisotope at 9:00 AM before a bone scan. The nurse should plan to send the patient for the bone scan at what time? A. 9:30 PM B. 10:00 AM C. 11:00 AM D. 1:00 PM

C. 11:00 AM A technician usually administers a calculated dose of a radioisotope 2 hours before a bone scan. If the patient was injected at 9:00 AM, the procedure should be done at 11:00 AM. 10:00 AM would be too early; 1:00 PM and 9:30 PM would be too late.

The nurse is caring for patients in a primary care clinic. Which individual is most at risk to develop osteomyelitis caused by Staphylococcus aureus? A. 22-year-old female with gonorrhea who is an IV drug user B. 48-year-old male with muscular dystrophy and acute bronchitis C. 32-year-old male with type 1 diabetes mellitus and a stage IV pressure ulcer D. 68-year-old female with hypertension who had a knee arthroplasty 3 years ago

C. 32-year-old male with type 1 diabetes mellitus and a stage IV pressure ulcer Osteomyelitis caused by Staphylococcus aureus is usually associated with a pressure ulcer or vascular insufficiency related to diabetes mellitus. Osteomyelitis caused by Staphylococcus epidermidis is usually associated with indwelling prosthetic devices such as joint replacements. Osteomyelitis caused by Neisseria gonorrhoeae is usually associated with gonorrhea. Osteomyelitis caused by Pseudomonas is usually associated with IV drug use. Muscular dystrophy is not associated with osteomyelitis.

The nurse determines that dietary teaching for a 75-year-old patient with osteoporosis has been successful when the patient selects which highest-calcium meal? A. Chicken stir-fry with 1 cup each onions and green peas, and 1 cup of steamed rice B. Ham and Swiss cheese sandwich on whole wheat bread, steamed broccoli, and an apple C. A sardine (3 oz) sandwich on whole wheat bread, 1 cup of fruit yogurt, and 1 cup of skim milk D. A two-egg omelet with 2 oz of American cheese, one slice of whole wheat toast, and a half grapefruit

C. A sardine (3 oz) sandwich on whole wheat bread, 1 cup of fruit yogurt, and 1 cup of skim milk The highest calcium content is present in the lunch containing milk and milk products (yogurt) and small fish with bones (sardines). Chicken, onions, green peas, rice, ham, whole wheat bread, broccoli, apple, eggs, and grapefruit each have less than 75 mg of calcium per 100 g of food. Swiss cheese and American cheese have more calcium, but not as much as the sardines, yogurt, and milk.

The patient is hospitalized with pneumonia. Which diagnostic test should be used to measure the efficiency of gas transfer in the lung and tissue oxygenation? A. Thoracentesis B. Bronchoscopy C. Arterial blood gases D. Pulmonary function tests

C. Arterial blood gases Arterial blood gases are used to assess the efficiency of gas transfer in the lung and tissue oxygenation as is pulse oximetry. Thoracentesis is used to obtain specimens for diagnostic evaluation, remove pleural fluid, or instill medication into the pleural space. Bronchoscopy is used for diagnostic purposes, to obtain biopsy specimens, and to assess changes resulting from treatment. Pulmonary function tests measure lung volumes and airflow to diagnose pulmonary disease, monitor disease progression, evaluate disability, and evaluate response to bronchodilators

A 63-year-old woman has been taking prednisone (Deltasone) daily for several years after a kidney transplant to prevent organ rejection. What is most important for the nurse to assess? A. Staggering gait B. Ruptured tendon C. Back or neck pain D. Tardive dyskinesia

C. Back or neck pain Osteoporosis with resultant fractures is a frequent and serious complication of systemic corticosteroid therapy. The ribs and vertebrae are affected the most, and patients should be observed for signs of compression fractures (back and neck pain). Phenytoin (Dilantin) is an antiseizure medication. An adverse effect of phenytoin is an ataxic (or staggering) gait. A rare adverse effect of ciprofloxacin (Cipro) and other fluoroquinolones is tendon rupture, usually of the Achilles tendon. The highest risk is in people age 60 and older and in people taking corticosteroids. Antipsychotics and antidepressants may cause tardive dyskinesia, which is characterized by involuntary movements of the tongue and face.

A 50-year-old patient sustained a large, open wound. The patient indicates his last tetanus booster was 3 years earlier. What action do you anticipate? A. Administer tetanus and diphtheria toxoid (Td) booster. B. Administer tetanus immune globulin (TIg). C. Clean the wound with soap and water. D. Clean the wound with hydrogen peroxide.

C. Clean the wound with soap and water. Immediate, thorough cleansing of all wounds with soap and water is important in the prevention of tetanus. After the adult is immunized, a booster is given every 10 years. If an open wound occurs, a booster is given if the last one was 5 or more years earlier. Immune globulin is used if immunization was never provided. Hydrogen peroxide is not ordinarily used for wound cleansing.

The nurse, when auscultating the lower lungs of the patient, hears these breath sounds. How should the nurse document these sounds? A. Stridor B. Rhonchi C. Coarse crackles D. Bronchovesicular

C. Coarse crackles Coarse crackles are a series of long-duration, discontinuous, low-pitched sounds caused by air passing through an airway intermittently occluded by mucus, an unstable bronchial wall, or a fold of mucosa. Coarse crackles are evident on inspiration and at times expiration. Stridor is a continuous crowing sound of constant pitch from partial obstruction of larynx or trachea. Rhonchi are a continuous rumbling, snoring, or rattling sound from obstruction of large airways with secretions. Bronchovesicular sounds are normal sounds heard anteriorly over the mainstem bronchi on either side of the sternum and posteriorly between the scapulae with a medium pitch and intensity.

What is essential teaching in treating a patient with Bell's palsy? A. Perform eye exercises to maintain strength. B. Obtain a herpes simplex virus (HSV) immunization. C. Do not abruptly stop the corticosteroids. D. Vigorously massage the area to promote circulation.

C. Do not abruptly stop the corticosteroids. Corticosteroids are usually started immediately. After they are no longer necessary, they should be tapered. Other treatment includes moist heat, gentle massage, and antiviral medications, such as acyclovir (Zovirax). Eye exercises are not indicated. HSV is identified in 70% of infections, but immunization is not beneficial at this point. Antiviral drugs may be used. Vigorous massage can break down tissues, but gentle upward massage has psychologic benefits.

What is the classic manifestation of a spinal cord tumor? A. Sudden onset of excruciating pain, worse at night B. Radiating pain down one leg C. Gradual onset of radicular pain, worse when lying down D. Positive Brudzinski's sign

C. Gradual onset of radicular pain, worse when lying down Tumors are slow growing. The most common early symptom is pain in the back with radicular pain. The pain worsens with activity, coughing, straining, and lying down. Sudden onset of excruciating pain is not related to spinal cord tumors. Radiating pain down one leg is a classic characteristic of sciatic nerve pathology. A positive Brudzinski's sign is seen in meningitis.

Which is the correct teaching regarding neurosyphilis (tertiary syphilis)? A. Symptoms appear 7 to 14 days after the initial infection. B. Untreated neurosyphilis is highly contagious. C. It results from an untreated syphilis infection. D. The main symptom is unilateral paralysis.

C. It results from an untreated syphilis infection. Neurosyphilis is an infection of any part of the nervous system by the organism Treponema pallidum, and it results from untreated or inadequately treated syphilis. It invades the central nervous system within a few months, and except for some minor changes, the organism lies dormant for years. It is not contagious but can be fatal. The later signs result from degenerative changes in the spinal cord and brainstem, including ataxia, loss of proprioception and deep tendon reflexes, and zones of hyperesthesia.

What is characteristic of trigeminal neuralgia (tic douloureux)? A. Unilateral facial drooping B. Inability to hear whispered speech C. One-sided facial stabbing pain D. Attacks of severe dizziness

C. One-sided facial stabbing pain Trigeminal neuralgia is usually unilateral, severe, brief, stabbing, recurrent episodes of pain in the distribution of the trigeminal nerve. Unilateral facial drooping is found in Bell's palsy or facial nerve pathology (cranial nerve [CN] VII). Inability to hear indicates pathology of the acoustic nerve (CN VIII). Attacks of severe dizziness do not occur in trigeminal neuralgia.

What is most important action for a patient who has a suspected cervical spinal injury? A. Apply a soft foam cervical collar. B. Perform a neurologic check. C. Place the patient on a firm surface. D. Assess function of cranial nerves IX and X.

C. Place the patient on a firm surface. A patient with a suspected cervical spine injury should be immobilized with a hard collar and placed on a firm surface. This takes priority over any further assessment. A soft foam collar does not provide immobilization.

You are providing care for a patient who has been diagnosed with Guillain-Barré syndrome. Which assessment should you prioritize? A. Pain assessment B. Glasgow Coma Scale C. Respiratory assessment D. Musculoskeletal assessment

C. Respiratory assessment Although all of the assessments are necessary in the care of patients with Guillain-Barré syndrome, the acute risk of respiratory failure necessitates vigilant monitoring of the patient's respiratory status.

The nurse prepares to administer IV ibandronate (Boniva) to a 67-year-old woman with osteoporosis. What is a priority laboratory assessment to make before the administration of ibandronate? A. Serum calcium B. Serum creatinine C. Serum phosphate D. Serum alkaline phosphatase

C. Serum phosphate Ibandronate is a bisphosphonate that is administered IV every 3 months and is administered slowly over 15 to 30 seconds to prevent renal damage. Ibandronate should not be used by patients taking other nephrotoxic drugs or by those with severe renal impairment (defined as serum creatinine above 2.3 mg/dL or creatinine clearance less than 30 mL/min).

What is the primary goal of nursing care for the patient with Guillain-Barré syndrome? A. Assist the patient to adapt to their lifelong paralysis. B. Teach the patient to use a communication board. C. Support body systems until the patient recovers. D. Place the patient in contact isolation to prevent spread of the condition.

C. Support body systems until the patient recovers. The objective of therapy is to support body systems until the patient recovers. Respiratory failure and infection are serious threats. Most patients recover eventually. Depending on the progression of the disease, the patient may be incapable of communicating.

A patient with a C7 spinal cord injury undergoing rehabilitation tells you he must have the flu because he has a bad headache and nausea. What is your initial action? A. Call the physician. B. Check the patient's temperature. C. Take the patient's blood pressure. D. Elevate the head of the bed to 90 degrees.

C. Take the patient's blood pressure. Autonomic dysreflexia is a massive, uncompensated cardiovascular reaction mediated by the sympathetic nervous system. Manifestations include hypertension (up to 300 mm Hg systolic), throbbing headache, marked diaphoresis above the level of the lesion, bradycardia (30 to 40 beats/minute), piloerection, flushing of the skin above the level of the lesion, blurred vision or spots in the visual fields, nasal congestion, anxiety, and nausea. It is important to measure blood pressure when a patient with a spinal cord injury complains of a headache.

The nurse can best determine adequate arterial oxygenation of the blood by assessing: A. heart rate. B. hemoglobin level. C. arterial oxygen tension. D. arterial carbon dioxide tension.

C. arterial oxygen tension. The ability of the lungs to oxygenate arterial blood adequately is determined by examination of the partial pressure of oxygen in arterial blood (PaO2) and arterial oxygen saturation (SaO2).

Knowing the most common causes of household fires, which of the following prevention strategies would the nurse focus on when teaching about fire safety? A. set hot water temperature at 140 degrees F (60*C) B. use only hardwired smoke detectors C. encourage regular home fire exit drills D. never permit older adults to cook unattended

C. encourage regular home fire exit drills

During the respiratory assessment of the older adult, the nurse would expect to find (select all that apply): A. a vigorous cough. B. increased chest expansion. C. increased residual volume. D. increased breath sounds in the lung apices. E. increased anteroposterior (AP) chest diameter.

C. increased residual volume. E. increased anteroposterior (AP) chest diameter. The anterior-posterior diameter of the thoracic cage and the residual volume increase in older adults. An older adult has a less forceful cough. The costal cartilages calcify with aging and interfere with chest expansion. Small airways in the lung bases close earlier during expiration. As a consequence, more inspired air is distributed to the lung apices, ventilation is less well matched to perfusion, and the PaO2 is lowered.

Fluid and electrolyte shifts that occur during the early emergent phase include: A. adherence of albumin to vascular walls B. movement of potassium into the vascular space C. sequestering of sodium and water into interstitial fluid D. hemolysis of red blood cells from the large volumes of rapidly administered fluid

C. sequestering of sodium and water into interstitial fluid

The nurse is caring for a patient admitted with chronic obstructive pulmonary disease (COPD), angina, and hypertension. Before administering the prescribed daily dose of atenolol 100 mg PO, the nurse assesses the patient carefully. Which adverse effect is this patient at risk for, given the patient's health history? A: Hypocapnia B: Tachycardia C: Bronchospasm D: Nausea and vomiting

C: Bronchospasm Atenolol is a cardioselective β1-adrenergic blocker that reduces blood pressure and could affect the β2-receptors in the lungs with larger doses or with drug accumulation. Although the risk of bronchospasm is less with cardioselective β-blockers than nonselective β-blockers, atenolol should be used cautiously in patients with COPD.

The patient has chronic hypertension. Today she has gone to the ED, and her blood pressure has risen to 200/140. What is the priority assessment for the nurse to make? A: Is the patient pregnant? B: Does the patient need to urinate? C: Does the patient have a headache or confusion? D: Is the patient taking antiseizure medications as prescribed?

C: Does the patient have a headache or confusion? The nurse's priority assessments include neurologic deficits, retinal damage, heart failure, pulmonary edema, and renal failure. The headache or confusion could be seen with hypertensive encephalopathy from increased cerebral capillary permeability leading to cerebral edema. Pregnancy can lead to secondary hypertension. Needing to urinate and taking antiseizure medication do not support a hypertensive emergency.

When teaching how lisinopril (Zestril) will help lower the patient's blood pressure, which mechanism of action should the nurse use to explain it? A: Blocks β-adrenergic effects. B: Relaxes arterial and venous smooth muscle. C: Inhibits conversion of angiotensin I to angiotensin II. D: Reduces sympathetic outflow from central nervous system.

C: Inhibits conversion of angiotensin I to angiotensin II. Lisinopril is an angiotensin-converting enzyme (ACE) inhibitor that inhibits the conversion of angiotensin I to angiotensin II, which reduces angiotensin II-mediated vasoconstriction and sodium and water retention. Beta blockers result in vasodilation and decreased heart rate. Direct vasodilators relax arterial and venous smooth muscle. Central acting α-adrenergic antagonists reduce sympathetic outflow from the CNS to produce vasodilation and decreased SVR and BP.

The nurse is teaching a women's group about prevention of hypertension. What information should be included in the teaching for all the women (select all that apply)? A: Lose weight. B: Limit nuts and seeds. C: Limit sodium and fat intake. D: Increase fruits and vegetables. E: Exercise 30 minutes most days.

C: Limit sodium and fat intake. D: Increase fruits and vegetables. E: Exercise 30 minutes most days. Primary prevention of hypertension is to make lifestyle modifications that prevent or delay the increase in BP. Along with exercise for 30 minutes on most days, the DASH eating plan is a healthy way to lower BP by limiting sodium and fat intake, increasing fruits and vegetables, and increasing nutrients that are associated with lowering BP. Nuts and seeds and dried beans are used for protein intake. Weight loss may or may not be necessary for the individual.

In caring for a patient admitted with poorly controlled hypertension, which laboratory test result should the nurse understand as indicating the presence of target organ damage? A: BUN of 15 mg/dL B: Serum uric acid of 3.8 mg/dL C: Serum creatinine of 2.6 mg/dL D: Serum potassium of 3.5 mEq/L

C: Serum creatinine of 2.6 mg/dL The normal serum creatinine level is 0.6-1.3 mg/dL. This elevated level indicates target organ damage to the kidneys. The other lab results are within normal limits.

The patient with suspected pancreatic cancer is having many diagnostic studies done. Which one can be used to establish the diagnosis of pancreatic adenocarcinoma and for monitoring the response to treatment? Spiral CT scan A PET/CT scan Incorrect Abdominal ultrasound Cancer-associated antigen 19-9

Cancer-associated antigen 19-9 Correct The cancer-associated antigen 19-9 (CA 19-9) is the tumor marker used for the diagnosis of pancreatic adenocarcinoma and for monitoring the response to treatment. Although a spiral CT scan may be the initial study done and provides information on metastasis and vascular involvement, this test and the PET/CT scan or abdominal ultrasound do not provide additional information.

A 22-year-old female is admitted with anorexia nervosa and a serum potassium level of 2.4 mEq/L. What complication is most important for the nurse to observe for in this patient?

Cardiac dysrhythmias A serum potassium level less than 2.5 mEq/L indicates severe hypokalemia, which can lead to life-threatening cardiac dysrhythmias (e.g., bradycardia, tachycardia, ventricular dysrhythmias). Other manifestations of potassium deficiency include muscle weakness and renal failure. Patients with anorexia nervosa commonly have iron-deficiency anemia and an elevated blood urea nitrogen level related to intravascular volume depletion and abnormal renal function.

The patient with right upper quadrant abdominal pain has an abdominal ultrasound that reveals cholelithiasis. What should the nurse expect to do for this patient? Prevent all oral intake. Control abdominal pain. Provide enteral feedings. Avoid dietary cholesterol.

Control abdominal pain. Patients with cholelithiasis can have severe pain, so controlling pain is important until the problem can be treated. NPO status may be needed if the patient will have surgery but will not be used for all patients with cholelithiasis. Enteral feedings should not be needed, and avoiding dietary cholesterol is not used to treat cholelithiasis.

When preparing a patient for a capsule endoscopy study, what should the nurse do? a. Ensure the patient understands the required bowel preparation. b. Have the patient return to the procedure room for removal of the capsule. c. Teach the patient to maintain a clear liquid diet throughout the procedure. d. Explain to the patient that conscious sedation will be used during placement of the capsule.

Correct answer: a Rationale: A capsule endoscopy study involves the patient performing a bowel prep to cleanse the bowel before swallowing the capsule. The patient will be on a clear liquid diet for 1 to 2 days before the procedure and will remain NPO for 4 to 6 hours after swallowing the capsule. The capsule is disposable and will pass naturally with the bowel movement, although the monitoring device will need to be removed.

The nurse instructs an obese 22-year-old man with a sedentary job about the health benefits of an exercise program. The nurse evaluates that teaching is effective when the patient makes which statement? a. "The goal is to walk at least 10,000 steps every day of the week." b. "Weekend aerobics for 2 hours is better than exercising every day." c. "Aerobic exercise will increase my appetite and result in weight gain." d. "Exercise causes weight loss by decreasing my resting metabolic rate."

Correct answer: a Rationale: A realistic activity goal is to walk 10,000 steps a day. Increased activity does not promote an increase in appetite or lead to weight gain. Exercise should be done daily, preferably 30 minutes to an hour a day. Exercise increases metabolic rate.

What problem should the nurse assess the patient for if the patient was on prolonged antibiotic therapy? a. Coagulation problems b. Elevated serum ammonia levels c. Impaired absorption of amino acids d. Increased mucus and bicarbonate secretion

Correct answer: a Rationale: Bacteria int he colon (1) synthesize vitamin K, which is needed for the production of prothrombin by the liver and (2) deaminate undigested or non absorbed proteins, producing ammonia, which is converted to urea by the liver. A reduction in normal flora bacteria by antibiotic therapy can lead to decreased vitamin K, resulting in decreased prothrombin and coagulation problems. Bowel bacteria do not influence protein absorption or the secretion of mucus.

A patient is jaundiced and her stools are clay colored (gray). This is most likely related to a. decreased bile flow into the intestine. b. increase production of urobilinogen. c. increased production of cholecystokinin. d. increased bile and bilirubin in the blood.

Correct answer: a Rationale: Bile is produced by the hepatocytes and is stored and concentrated in the gallbladder. When bile is released from the common bile duct, it enters the duodenum. In the intestines, bilirubin is reduced to stercobilinogen and urobilinogen by bacterial action. Stercobilinogen accounts for the brown color of stool. Stools may be clay-colored if bile is not released from the common bile duct into the duodenum. Jaundice may result if the bilirubin level in the blood is elevated.

What characterizes auscultation of the abdomen? a. The presence of borborygmi indicates hyper peristalsis. b. The bell of the stethoscope is used to auscultate high-pitched sounds. c. High-pitched, rushing, and tinkling bowel sounds are heard after eating. d. Absence of bowel sounds for 1 minute in each quadrant is reported as abnormal.

Correct answer: a Rationale: Borborygmi are loud gurgles (stomach growling) that indicate hyper peristalsis. Normal bowel sounds are relatively high-pitched and are heard best with the diaphragm of the stethoscope. High-pitched, tinkling bowel sounds occur when the intestines are under tension, as in bowel obstructions. Absent bowel sounds may be reported when no sounds are heard for 2 to 3 minutes in each quadrant.

What information should be included in the dietary teaching for the patient following a Roux-en-Y gastric bypass? a. Avoid sugary foods and limit fluids to prevent dumping syndrome. b. Gradually increase the amount of food ingested to preoperative levels. c. Maintain a long-term liquid diet to prevent damage to the surgical site. d. Consume foods high in complex carbohydrates, protein, and fiber to add bulk to contents.

Correct answer: a Rationale: Fluids and foods high in carbohydrates tend to promote diarrhea and symptoms of dumping syndrome in patients with gastric bypass surgery. The diet generally should be high in protein and low in carbohydrates, fat, and roughage and consists of six small feedings a day because of the small stomach size. Liquid diets are likely to be used longer for the patient with a gastroplasty.

A patient had a stomach resection for stomach cancer. The nurse should teach the patient about the loss of the hormone that stimulates gastric acid secretion and motility and maintains lower esophageal sphincter tone. Which hormone will be decreased with a gastric resection? a. Gastrin b. Secretin c. Cholecystokinin d. Gastric inhibitory peptide

Correct answer: a Rationale: Gastrin is the hormone activated in the stomach (and duodenal mucosa) by stomach distention that stimulates gastric acid secretion and motility and maintains lower esophageal sphincter tone. Secretin inhibits gastric motility and acid secretion and stimulates pancreatic bicarbonate secretion. Cholecystokinin allows increased flow of bile into the duodenum and release of pancreatic digestive enzymes. Gastric inhibitory peptide inhibits gastric acid secretion and motility.

The nurse is caring for a patient who is 5'6" tall and weighs 186 lb. The nurse has discussed reasonable weight loss goals and a low-calorie diet with the patient. Which statement made by the patient indicates a need for further teaching? a. "I will limit intake to 500 calories a day." b. "I will try to eat very slowly during mealtimes." c. "I'll try to pick foods from all of the basic food groups." d. "It's important for me to begin a regular exercise program."

Correct answer: a Rationale: Limiting intake to 500 calories per day is not indicated for this patient, and the severe calorie energy restriction would place this patient at risk for multiple nutrient deficiencies. Decreasing caloric intake at least 500 to 1000 calories a day is recommended for weight loss of one to two pounds per week. The other options show understanding of the teaching.

Priority Decision: Before selecting a weight reduction plan with an obese patient, what is most important for the nurse to first assess? a. The patient's motivation to lose weight b. The length of time that the patient has been obese c. Whether financial considerations will affect the patient's choices d. The patient's anthropometric measures of height, weight, BMI, waist-to-hip ratio, and skinfold thickness

Correct answer: a Rationale: Motivation is essential. Focus on the reasons for wanting to lose weight. The rest of the options will asset in planning the weight loss if the patient is motivated.

A 50-year-old African American woman has a BMI of 35 kg/m2, type 2 diabetes mellitus, hypercholesterolemia, and irritable bowel syndrome (IBS). She is seeking assistance in losing weight, because, "I have trouble stopping eating when I should, but I do not want to have bariatric surgery." Which drug therapy should the nurse question if it is prescribed for this patient? a. Orlistat (Xenical) b. Locaserin (Belviq) c. Phentermine (Adipex-P) d. Phentermine and topiramate (Qsymia)

Correct answer: a Rationale: Orlistat (Xenical), which blocks fat breakdown and absorption in the intestine, produces some unpleasant GI side effects. This drug would not be appropriate for someone with IBS. Locaserin (Belviq) suppresses the appetite and creates a sense of satiety that may be helpful for this patient. Phentermine (Adipex-P) needs to be used for a limited period of time (3 months or less). Qsymia is a combination of two drugs, phentermine and topiramate. Phentermine is a sympathomimetic agent that suppresses appetite and topiramate induces a sense of satiety.

A patient has been on a 1000-calorie diet with a daily exercise routine. In 2 months, the patient has lost 20 lb (9kg) toward a goal of 50 lb (23 kg) but is now discouraged that no weight has been lost in the last 2 weeks. What should the nurse tell the patient about this? a. Plateaus where no weight is lost normally occur during a weight-loss program. b. A weight considered by the body to most efficient for functioning has been reached. c. A return to former eating habits is the most common cause of not continuing to lose weight. d. A steady weight may be due to water gain from eating foods high in sodium.

Correct answer: a Rationale: Plateau periods during which no weight is lost are normal occurrences during weight reduction and may last for several days to several weeks but weight loss will resume if the prescribed weight reduction plan is continued. Weight loss may stop if former eating habits are resumed but this not the most common cause of plateaus.

A patient asks the nurse about taking phentermine and topiramate (Qsymia) for weight loss. To avoid side effects, it is important for the nurse to determine whether the patient has a history of a. glaucoma. b. hypertension. c. valvular heart disease. d. irritable bowel disease.

Correct answer: a Rationale: Qsymia is a combination of phentermine and topiramate. It must not be used in patients with glaucoma or hyperthyroidism.

Priority Decision: Following auscultation of the abdomen, what should the nurse's next action be? a. Lightly percuss over all four quadrants b. Have the patient empty his or her bladder c. Inspect perianal and anal areas for color, masses, rashes, and scars d. Perform deep palpation to delineate abdominal organs and masses

Correct answer: a Rationale: The abdomen should be assessed in the following sequence: inspection, auscultation, percussion, palpation. The patient should empty his or her bladder before assessment begins.

A patient with hepatitis A is in the acute phase. The nurse plans care for the patient based on the knowledge that a. pruritus is a common problem with jaundice in this phase. b. the patient is most likely to transmit the disease during this phase. c. gastrointestinal symptoms are not as severe in hepatitis A as they are in hepatitis B. d. extrahepatic manifestations of glomerulonephritis and polyarteritis are common in this phase. (Lewis 1042)

Correct answer: a Rationale: The acute phase of jaundice may be icteric or anicteric. Jaundice results when bilirubin diffuses into the tissues. Pruritus sometimes accompanies jaundice. Pruritus is the result of an accumulation of bile salts beneath the skin.

During the initial postoperative period following bariatric surgery, the nurse recognizes the importance of monitoring obese patients for respiratory insufficiency based on what knowledge? a. The body stores anesthetics in adipose tissue. b. Postoperative pain may cause a decreased respiratory rate. c. Intubation may be difficult because of extra chin skinfolds. d. The patient's head must remain flat for a minimum of 2 hours postprocedure.

Correct answer: a Rationale: The body stores anesthetics in adipose tissue, placing patients with excess adipose tissue at risk for re-sedation. As adipose cells release anesthetics back into the bloodstream, the patient may become sedated after surgery, increasing the risk of hypoventilation and resultant respiratory insufficiency. Difficult intubation does not cause respiratory insufficiency. Pain usually increases respiratory rate. The patient's head should be elevated after bariatric surgery to decrease abdominal pressure and facilitate respirations.

The nurse cares for a 34-year-old woman after bariatric surgery. The nurse determines that discharge teaching related to diet is successful if the patient makes which statement? a. "A high protein diet that is low in carbohydrates and fat will prevent diarrhea." b. "Food should be high in fiber to prevent constipation from the pain medication." c. "Three meals a day with no snacks between meals will provide optimal nutrition." d. "Fluid intake should be at least 2000 mL per day with meals to avoid dehydration."

Correct answer: a Rationale: The diet generally prescribed is high in protein and low in carbohydrates, fat, and roughage and consists of six small feedings daily. Fluids should not be ingested with the meal, and in some cases, fluids should be restricted to less than 1000 mL per day. Fluids and foods high in carbohydrate tend to promote diarrhea and symptoms of the dumping syndrome. Generally, calorically dense foods (foods high in fat) should be avoided to permit more nutritionally sound food to be consumed.

Checking for the return of the gag reflex and monitoring for LUQ pain, nausea and vomiting are necessary nursing actions after which diagnostic procedure? a. ERCP b. Colonoscopy c. Barium swallow d. Esophagogastroduodenoscopy (EGD)

Correct answer: a Rationale: The left upper quadrant (LUQ) pain and nausea and vomiting could occur from perforation. The return of gag reflex is essential to prevent aspiration after an ERCP. The gag reflex is also assessed with an EGD. These are not relevant assessments for the colonoscopy and barium swallow.

A 35-year-old man with a family history of adenomatous polyposis had a colonoscopy with removal of multiple polyps. Which signs and symptoms should the nurse teach the patient to report immediately? a. Fever and abdominal pain b. Flatulence and liquid stool c. Loudly audible bowel sounds d. Sleepiness and abdominal cramps

Correct answer: a Rationale: The patient should be taught to observe for signs of rectal bleeding and peritonitis. Fever, malaise, and abdominal pain and distention could indicate a perforated bowel with peritonitis.

The nurse teaches a 50-year-old woman who has a body mass index (BMI) of 39 kg/m2 about weight loss. Which dietary change would be appropriate for the nurse to recommend to this patient? a. Decrease fat intake and control portion size b. Increase vegetables and decrease fluid intake c. Increase protein intake and avoid carbohydrates d. Decrease complex carbohydrates and limit fiber

Correct answer: a Rationale: The safest dietary guideline for weight loss is to decrease caloric intake by maintaining a balance of nutrients and adequate hydration while controlling portion size and decreasing fat intake.

A 90-year-old healthy man is suffering from dysphagia. The nurse explains what age-related change of the GI tract is the most likely cause of his difficulty? a. Xerostomia b. Esophageal cancer c. Decreased taste buds d. Thinner abdominal wall

Correct answer: a Rationale: Xerostomia, decreased volume of saliva, leads to dry oral mucosa and dysphagia. Esophageal cancer is not an age-related change. Decreased taste buds and a thinner abdominal wall do not contribute to difficulty swallowing.

The nurse is caring for a 45-year-old woman with a herniated lumbar disc. The patient realizes that weight loss is necessary to lessen back strain. The patient is 5'6" tall and weighs 186 lb (84.5 kg) with a body mass index (BMI) of 28 kg/m2. The nurse explains to the patient that this measurement places her in which of the following weight categories? a. Normal weight b. Overweight c. Obese d. Severely obese

Correct answer: b Rationale: A normal BMI is 18.5 to 24.9 kg/m2, whereas a BMI of 25 to 29.9 kg/m2 is considered overweight. A BMI of 30.0-39.9 is considered obese, and a BMI of 40 or greater is severely obese.

A woman is 5 ft, 6 in (166 cm) tall and weighs 200 lb (90.9 kg) with a waist-to-hip ratio of 0.7. The nurse counsels the patient with the knowledge that the patient is at greatest risk for a. heart disease. b. osteoporosis. c. diabetes mellitus. d. endometrial cancer.

Correct answer: b Rationale: A patient who is obese (BMI of 32.2) but has a waist-to-hip ratio of less than 0.8, indicating gynoid obesity, has an increased risk for osteoporosis. The other conditions are risks associated with android obesity.

The patient with advanced cirrhosis asks why his abdomen is so swollen. The nurse's response is based on the knowledge that a. a lack of clotting factors promotes the collection of blood in the abdominal cavity. b. portal hypertension and hypoalbuminemia cause a fluid shift into the peritoneal space. c. decreased peristalsis in the GI tract contributes to gas formation and distention of the bowel. d. bile salts in the blood irritate the peritoneal membranes, causing edema and pocketing of fluid. (Lewis 1042)

Correct answer: b Rationale: Ascites is the accumulation of serous fluid in the peritoneal or abdominal cavity and is a common manifestation of cirrhosis. With portal hypertension, proteins shift from the blood vessels through the larger pores of the sinusoids (capillaries) into the lymph space. When the lymphatic system is unable to carry off the excess proteins and water, those substances leak through the liver capsule into the peritoneal cavity. Osmotic pressure of the proteins pulls additional fluid into the peritoneal cavity. A second mechanism of ascites formation is hypoalbuminemia, which results from the inability of the liver to synthesize albumin. Hypoalbuminemia results in decreased colloidal oncotic pressure. A third mechanism is hyperaldosteronism, which occurs when aldosterone is not metabolized by damaged hepatocytes. The increased level of aldosterone causes increases in sodium reabsorption by the renal tubules. Sodium retention and an increase in antidiuretic hormone levels cause additional water retention.

A patient has an elevated blood level of indirect (unconjugated) bilirubin. One cause of this finding is that a. the gallbladder is unable to contract to release stored bile. b. bilirubin is not being conjugated and excreted into the bile by the liver. c. the Kupffer cells in the liver are unable to remove bilirubin from the blood. d. there is an obstruction in the biliary tract preventing flow of bile into the small intestine.

Correct answer: b Rationale: Bilirubin is a pigment derived from the breakdown of hemoglobin and is insoluble in water. Bilirubin is bound to albumin for transport to the liver and is referred to as unconjugated. An indirect bilirubin determination is a measurement of unconjugated bilirubin, and the level may be elevated in hepatocellular and hemolytic conditions.

A patient with acute hepatitis B is being discharged in 2 days. In the discharge teaching plan the nurse should include instructions to a. avoid alcohol for the first 3 weeks. b. use a condom during sexual intercourse. c. have family members get an injection of immunoglobulin. d. follow a low-protein, moderate-carbohydrate, moderate-fat diet. (Lewis 1042)

Correct answer: b Rationale: Hepatitis B virus may be transmitted by mucosal exposure to infected blood, blood products, or other body fluids (e.g., semen, vaginal secretions, saliva). Hepatitis B is a sexually transmitted disease that is acquired through unprotected sex with an infected person. Condom use should be taught to patients to prevent transmission of hepatitis B.

Priority Decision: When medications are used in the treatment of obesity, what is most important for the nurse to teach the patient? a. Over-the-counter (OTC) diet aids are safer than other agents and con be useful in controlling appetite. b. Drugs should be used only as adjuncts to a diet and exercise program as treatment for a chronic condition. c. All drugs used for weight control are capable of altering central nervous system (CNS) function and should be used with caution. d. The primary effect of the medications is psychologic, controlling the urge to eat in response to stress or feelings of rejection.

Correct answer: b Rationale: Medications are used only as adjuncts to diet and exercise programs in the treatment of obesity. Drugs do not cure obesity; without changes in food intake and physical activity, weight gain will occur when the medications are discontinued. The medications used work in a variety of ways to control appetite but over-the-counter drugs are probably the least effective and most abused of these drugs.

A patient has been told that she has elevated liver enzymes caused by nonalcoholic fatty liver disease (NAFLD). The nursing teaching plan should include a. having genetic testing done. b. recommending a heart-healthy diet. c. the necessity to reduce weight rapidly. d. avoiding alcohol until liver enzymes return to normal. (Lewis 1042)

Correct answer: b Rationale: Nonalcoholic fatty liver disease (NAFLD) can progress to liver cirrhosis. There is no definitive treatment, and therapy is directed at reduction of risk factors, which include treatment of diabetes, reduction in body weight, and elimination of harmful medications. For patients who are overweight, weight reduction is important. Weight loss improves insulin sensitivity and reduces liver enzyme levels. No specific dietary therapy is recommended. However, a heart-healthy diet as recommended by the American Heart Association is appropriate.

An 80-year-old man states that, although be adds a lot of salt to his food, it still does not have much taste. The nurse's response is based on the knowledge that the older adult a. should not experience changes in taste. b. has a loss of taste buds, especially for sweet and salty. c. has some loss of taste but no difficulty chewing food. d. loses the sense of taste because the ability to smell is decreased.

Correct answer: b Rationale: Older adults have decreased numbers of taste buds and a decreased sense of smell. These age-related changes diminish the sense of taste (especially of salty and sweet substances).

Priority Decision: The nurse admitting a patient for bariatric surgery obtains the following information from the patient. Which finding should be brought to the surgeon's attention before proceeding with further patient preparation? a. History of hypertension b. History of untreated depression c. History of multiple attempts at weight loss d. History of sleep apnea treated with continuous positive airway pressure (CPAP)

Correct answer: b Rationale: Patients with histories of untreated depression or psychosis are not good candidates for surgery. All other historical information includes medical complications of severe obesity that would help to qualify the patient for the surgery.

In preparing a patient for a colonoscopy, the nurse explains that a. a signed permit is not necessary. b. sedation may be used during the procedure. c. only one cleansing enema is necessary for preparation. d. a light meal should be eaten the day before the procedure.

Correct answer: b Rationale: Sedation is induced during a colonoscopy. A signed consent form is necessary for a colonoscopy. A cathartic or enema is administered the night before the procedure, and more than one enema may be necessary. Patients may need to be kept on clear liquids 1 to 2 days before the procedure.

Which patient is at highest risk for complications of obesity? a. A 30-year-old woman who is 5 ft (151 cm) tall, weighs 140 lb (63 kg), and carries weight in her thighs. b. A 56-year-old woman with a BMI of 38 kg/m2, a waist measurement of 38 in (96 cm), and a hip measurement of 36 in (91 cm) c. A 42-year-old man with a waist measurement of 36 in (91 cm) and a hip measurement of 36 in (91 cm) who is 5 ft, 6 in (166 cm) tall and weighs 150 lb (68.2 kg) d. A 68-year-old man with a waist measurement of 38 in (96 cm) and a hip measurement of 42 in (76 cm) who is 5 ft, 11 in (179 cm) tall and weighs 200 lb (90.9 kg)

Correct answer: b Rationale: The 56-year-old woman has a body mass index (BMI) of 38 kg/m2 (obese, Class II) with a waist-to-hip ratio of 1.1 with android obesity and is more at risk (very high) than the other patients. The 30-year-old woman has the least risk with a BMI of 27.3 kg/m2 (overweight) and gynoid shape. The 42-year-old man has a BMI of 24.2 kg/m2 (normal weight) with one risk factor in the waist-to-hip ratio of 1.0 and the 68-year-old man has a BMI of 27.9 kg/m2 (overweight) with a waist-to-hip ratio of 0.9.

A 40-year-old severely obese female patient with type 2 diabetes wants to lose weight. After learning about the surgical procedures, she thinks a combination of restrictive and malabsorptive surgery would be best. Which procedure should the nurse teach her about? a. Lipectomy b. Roux-en-Y gastric bypass c. Adjustable gastric banding d. Vertical sleeve gastrectomy

Correct answer: b Rationale: The Roux-en-Y gastric bypass is a common combination of restrictive (limiting the size of the stomach) and malabsorptive (less food is absorbed) surgery. Lipectomy is used to remove unsightly flabby folds of adipose tissue. Adjustable gastric banding is the most common restrictive procedure. Vertical sleeve gastrectomy is a restrictive procedure that preserves stomach function.

The patient had a car accident and was "scared to death." The patient is now reporting constipation. What affecting the gastrointestinal (GI) tract does the nurse know could be contributing to the constipation? a. The patient is too nervous to eat or drink, so there is no stool. b. The sympathetic nervous system was activated, so the GI tract was slowed. c. The parasympathetic nervous system is now functioning to slow the GI tract. d. The circulation in the GI system has been increased, so less waste is removed.

Correct answer: b Rationale: The constipation is most likely related to the sympathetic nervous system activation from the stress related to the accident. SNS activation can decrease peristalsis. Even without oral intake for a short time, stool will be formed. The parasympathetic system stimulates peristalsis. The circulation to the GI system is decreased with stress.

As gastric contents move into the small intestine, the bowel is normally protected from the acidity of gastric contents by the a. inhibition of secretin release. b. release of bicarbonate by the pancreas. c. release of pancreatic digestive enzymes. d. release of gastrin by the duodenal mucosa.

Correct answer: b Rationale: The hormone secretin stimulates the pancreas to secrete fluid with a high concentration of bicarbonate. This alkaline secretion enters the duodenum and neutralizes acid in the chyme.

During an examination of the abdomen the nurse should a. position the patient in the supine position with the head of the bed flat and knees straight. b. listen in the epigastrium and all four quadrants for 2 minutes for bowel sounds. c. use the following order of techniques: inspection, palpation, percussion, auscultation. d. describe bowel sounds as absent if no sound is heard in the lower right quadrant after 2 minutes.

Correct answer: b Rationale: The nurse should listen in the epigastrium and all four quadrants for bowel sounds for at least 2 minutes. The patient should be in the supine position and should slightly flex the knees; the head of the bed should be raised slightly. During examination of the abdomen, the nurse auscultates before performing percussion and palpation because the latter procedures may alter the bowel sounds. Bowel sounds cannot be described as absent until no sound is heard for 5 minutes in each quadrant.

A patient is admitted to the hospital with left upper quadrant (LUQ) pain. What may be a possible source of the pain? a. Liver b. Pancreas c. Appendix d. Gallbladder

Correct answer: b Rationale: The pancreas is located in the left upper quadrant, the liver is in the right upper quadrant, the appendix is in the right lower quadrant, and the gallbladder is in the right upper quadrant.

The nurse has completed initial instruction with a patient regarding a weight loss program. The nurse determines that the teaching has been effective when the patient makes which statement? a. "I plan to lose 4 lb a week until I have lost the 60-pound goal." b. "I will keep a diary of weekly weights to illustrate my weight loss." c. "I will restrict my carbohydrate intake to less than 30 g/day to maximize weight loss." d."I should not exercise more than my program requires since increased activity increases the appetite."

Correct answer: b Rationale: The patient should monitor and record weight once per week. This prevents frustration at the normal variations in daily weights and may help the patient to maintain motivation to stay on the prescribed diet. Weight loss should occur at a rate of 1 to 2 lb/week. The diet should be well balanced rather than lacking in specific components that may cause an initial weight loss but is not usually sustainable. Exercise is a necessary component of any successful weight loss program.

A 62-year-old woman patient is scheduled for a percutaneous transhepatic cholangiography to restore biliary drainage. The nurse discusses the patient's health history and is most concerned if the patient makes which statement? a."I am allergic to bee stings." b. "My tongue swells when I eat shrimp." c. "I have had epigastric pain for 2 months." d. "I have a pacemaker because my heart rate was slow."

Correct answer: b Rationale: The percutaneous transhepatic cholangiography procedure will include the use of radiopaque contrast medium. Patients allergic to shellfish and iodine are also allergic to contrast medium. Having a pacemaker will not affect the patient during this procedure. It would be expected that the patient would have some epigastric pain given the patient's condition.

What is a normal finding on physical examination of the abdomen? a. Auscultation of bruits b. Observation of visible pulsations c. Percussion of liver dullness in the left midclavicular line d. Palpation of the spleen 1 to 2 cm below the left costal margin

Correct answer: b Rationale: The pulsation of the aorta in the epigastric area is a normal finding. Bruits indicate that blood flow is abnormal, the liver is percussed in the right midclavicular line, and a normal spleen cannot be palpated.

Which female patient is most likely to have metabolic syndrome? a. BP 128/78 mm Hg, triglycerides 160 mg/dL, fasting blood glucose 102 mg/dL b. BP 142/90 mm Hg, high-density lipoproteins 45 mg/dL, fasting blood glucose 130 mg/dL c. Waist circumference 36 in, triglycerides 162 mg/dL, high-density lipoproteins 55 mg/dL d. Waist circumference 32 in, high-density lipoproteins 38 mg/dL, fasting blood glucose 122 mg/dL

Correct answer: b Rationale: Three of the following five measures are needed for a woman to be diagnosed with metabolic syndrome: waist circumference >35 in, triglycerides >150 mg/dL, high-density lipoproteins <50 mg/dL, BP >130 mm Hg systolic or >85 mm Hg diastolic, fasting blood glucose >110 mg/dL. Although the other options have some abnormal measures, none has all three measures in the diagnostic ranges. The criteria for metabolic syndrome for both women and men are listed in Table 41-10.

Which statement about obesity is explained by genetics? a. Older obese patients have exacerbated changes of aging. b. Android body shape and weight gain are influenced by genetics. c. White Americans have a higher incidence of obesity than African Americans. d. Men have a harder time losing weight, as they have more muscle mass than women.

Correct answer: b Rationale: Twin studies and studies with adopted children have shown that body shape and weight gain are influenced by genetics but more research is needed. Older obese people do have exacerbated aging problems related to declines in physical function. African Americans and Hispanics have a higher incidence of obesity than whites. Women have a higher incidence of obesity and more difficulty losing weight than men because women have a higher percentage of metabolically less-active fat.

Inspection of an older patient's mouth reveals the presence of white, curd-like lesions on the patient's tongue. What is the most likely etiology for this abnormal assessment finding? a. Herpesvirus b. Candida albicans c. Vitamin deficiency d. Irritation from ill-fitting dentures

Correct answer: b Rationale: White, curd-like lesions surrounded by erythematous mucosa are associated with oral candidiasis. Herpesvirus causes benign vesicular lesions in the mouth. Vitamin deficiencies may cause a reddened, ulcerated, swollen tongue. Irritation from ill-fitting dentures will cause friable, edematous, painful, bleeding gingivae.

An 85-year-old woman seen in the primary care provider's office for a well check complains of difficulty swallowing. What common effect of aging should the nurse assess for as a possible cause? a. Anosmia b. Xerostomia c. Hypochlorhydria d. Salivary gland tumor

Correct answer: b Rationale: Xerostomia (decreased saliva production), or dry mouth, affects many older adults and may be associated with difficulty swallowing (dysphagia). Anosmia is loss of sense of smell. Hypochlorhydria, a decrease in stomach acid, does not affect swallowing. Salivary gland tumors are not common.

Which digestive substances are active or activated in the stomach (select all that apply)? a. Bile b. Pepsin c. Gastrin d. Maltase e. Secretin f. Amylase

Correct answer: b, c Rationale: Pepsinogen is changed to pepsin by acidity of the stomach, where it begins to break down proteins. Gastrin stimulates gastric acid secretion and motility and maintains lower esophageal sphincter tone. The stomach also secretes lipase for fat digestion.Bile is secreted by the liver and stored in the gallbladder for emulsifying fats. Maltase is secreted in the small intestine and converts maltose to glucose. Secretin is secreted y the duodenal mucosa and inhibits gastric motility and acid secretion. Amylase is secured in the small intestine and by the pancreas for carbohydrate digestion.

The nurse should recognize that the liver performs which functions (select all that apply) a. Bile storage b. Detoxification c. Protein metabolism d. Steroid metabolism e. Red blood cell (RBC) destruction

Correct answer: b, c, d Rationale: The liver performs multiple major functions that aid in the maintenance of homeostasis. These include metabolism of proteins and steroids as well as detoxification of drugs and metabolic waste products. The Kupffer cells of the liver participate in the breakdown of old RBCs. The liver produces bile, but storage occurs in the gall bladder.

The patient tells the nurse she had a history of abdominal pain, so she had a surgery to make an opening into the common bile duct to remove stones. The nurse knows that this surgery is called a a. colectomy b. cholecystectomy c. choledocholithotomy d. choledochojejunostomy

Correct answer: c Rationale: A choledocholithotomy is an opening into the common bile duct for the removal of stones. A colectomy is the removal of the colon. The cholecystectomy is the removal of the gallbladder. The choledochojejunostomy is an opening between the common bile duct and the jejunum.

In developing a weight reduction program with a 45-year-old female patient who weighs 197 lb, the nurse encourages the patient to set a weight loss goal of how many pounds in 4 weeks? a. 1-2 b. 3-5 c. 4-8 d. 5-10

Correct answer: c Rationale: A realistic weight loss goal for patients is 1 to 2 lb/wk, which prevents the patient from becoming frustrated at not meeting weight loss goals.

What is a normal finding during physical assessment of the mouth? a. A red, slick appearance of the tongue b. Uvular deviation to the side on saying "Ahh" c. A thin, white coating of the dorsum of the tongue d. Scattered red, smooth areas on the dorsum of the tongue

Correct answer: c Rationale: A thin white coating of the dorsum (top) of the tongue is normal. A red, slick appearance is characteristic of cobalamin deficiency and scattered red, smooth areas on the tongue are known as geographic tongue. The uvula should remain in the midline while the patient is saying "Ahh"

When assessing a patient's abdomen, what would be most appropriate for the nurse to do? a. Palpate the abdomen before auscultation. b. Percuss the abdomen before auscultation. c. Auscultate the abdomen before palpation. d. Perform deep palpation before light palpation.

Correct answer: c Rationale: During examination of the abdomen, auscultation is done before percussion and palpation because these latter procedures may alter the bowel sounds.

In developing an effective weight reduction plan for an overweight patient who states a willingness to try to lose weight, it is most important for the nurse to first assess which factor? a. The length of time the patient has been obese b. The patient's current level of physical activity c. The patient's social, emotional, and behavioral influences on obesity d. Anthropometric measurements, such as body mass index and skinfold thickness

Correct answer: c Rationale: Eating patterns are established early in life, and eating has many meanings for people. To establish a weight reduction plan that will be successful for the patient, the nurse should first explore the social, emotional, and behavioral influences on the patient's eating patterns. The duration of obesity, current physical activity level, and current anthropometric measurements are not as important for the weight reduction plan.

The health care team is assessing a male patient for acute pancreatitis after he presented to the emergency department with severe abdominal pain. Which laboratory value is the best diagnostic indicator of acute pancreatitis? a. Gastric pH b. Blood glucose c. Serum amylase d. Serum potassium

Correct answer: c Rationale: Elevated serum amylase levels indicate early pancreatic dysfunction and are used to diagnose acute pancreatitis. Serum lipase levels stay elevated longer than serum amylase in acute pancreatitis. Blood glucose, gastric pH, and potassium levels are not direct indicators of acute pancreatic dysfunction.

A patient who is scheduled for surgery with general anesthesia in 1 hour is observed with a moist, but empty water glass in his hand. Which assessment finding may indicate that the patient drank a glass of water? a. Flat abdomen without movement upon inspection b. Tenderness at left upper quadrant upon palpation c. Easily heard, loud gurgling in the right upper quadrant d. High-pitched, hollow sounds in the left upper quadrant

Correct answer: c Rationale: If the patient drank water on an empty stomach, gurgling can be assessed without a stethoscope or assessed with auscultation. High-pitched, hollow sounds are tympanic and indicate an empty cavity. A flat abdomen and tenderness do not indicate that the patient drank a glass of water.

What is the main underlying risk factor for metabolic syndrome? a. Age b. Heart disease c. Insulin resistance d. High cholesterol levels

Correct answer: c Rationale: Insulin resistance is the main underlying risk factor for metabolic syndrome. Aging is associated with metabolic syndrome. High cholesterol, hypertension, and increased clotting risk are characteristics of metabolic syndrome.

Priority Decision: During care of the severely obese patient, what is most important for the nurse to do? a. Avoid reference to the patient's weight to avoid embarrassing the patient. b. Emphasize to the patient how important it is to lose weight to maintain health. c. Plan for necessary modifications in equipment and nursing techniques before initiating care. d. Recognize that a full assessment of each body system might not be possible because of numerous layers of skinfolds.

Correct answer: c Rationale: Special considerations are needed for the care of the severely obese patient because most hospital units are not prepared with beds, chairs, BP cuffs, and other equipment that will need to be used with the very obese patient. Consideration of all aspects of care should be made before implementing care for the patient, including extra time and perhaps assistance for positioning, physical assessment, and transferring the patient.

When caring for the patient with heart failure, the nurse knows that which gastrointestinal process is most dependent on cardiac output and may affect the patient's nutritional status? a. Ingestion b. Digestion c. Absorption d. Elimination

Correct answer: c Rationale: Substances that interface with the absorptive surfaces of the GI tract (primarily in the small intestine) diffuse across the intestinal membranes into intestinal capillaries and are then carried to other parts of the body for use in energy production. The cardiac output provides the blood flow for this absorption of nutrients to occur.

How will an obstruction at the ampulla of Vater affect the digestion of all nutrients? a. Bile is responsible for emulsification of all nutrients and vitamins. b. Intestinal digestive enzymes are released through the ampulla of Vater. c. Both bile and pancreatic enzymes enter the duodenum at the ampulla of Vater. d. Gastric contents can ply pass to the duodenum when the ampulla of Vater is open.

Correct answer: c Rationale: The ampulla of Vater is the site where the pancreatic duct and common bile duct enter the duodenum and the opening and closing of the ampulla is controlled by the sphincter of Oddi. Because bile from the common bile duct is needed for emulsification of fat to promote digestion and pancreatic enzymes from the pancreas are needed for digestion of all nutrients, a blockage at this point would affect the digestion of all nutrients. Gastric contents pass into the duodenum through the pylorus or pyloric valve.

A patient's serum liver enzyme tests reveal an elevated aspartate aminotransferase (AST). The nurse recognizes what about the elevated AST? a. It eliminates infection as a cause of liver damage. b. It is diagnostic for liver inflammation and damage. c. Tissue damage in organs other than the liver may be identified. d. Nervous system symptoms related to hepatic encephalopathy may be the cause.

Correct answer: c Rationale: The aspartate aminotransferase (AST) level is elevated in liver disease but it is important to note that it is also elevated in damage to the heart and lungs and is not a specific test for liver function. Measurements of most of the transaminases involves nonspecific tests unless isoenzyme fractions are determined. Hepatic encephalopathy is related to elevated ammonia levels.

The nurse is assessing a 50-year-old woman admitted with a possible bowel obstruction. Which assessment finding would be expected in this patient? a. Tympany to abdominal percussion b. Aortic pulsation visible in epigastric region c. High-pitched sounds on abdominal auscultation d. Liver border palpable 1 cm below the right costal margin

Correct answer: c Rationale: The bowel sounds are more high pitched (rushes and tinkling) when the intestines are under tension, as in intestinal obstruction. Bowel sounds may also be diminished or absent with an intestinal obstruction. Normal findings include aortic pulsations on inspection and tympany with percussion, and the liver may be palpable 1 to 2 cm along the right costal margin.

Which statement best describes the etiology of obesity? a. Obesity primarily results from a genetic predisposition. b. Psychosocial factors can override the effects of genetics in the etiology of obesity. c. Obesity is the result of complex interactions between genetic and environmental factors. d. Genetic factors are more important than environmental factors in the etiology of obesity.

Correct answer: c Rationale: The cause of obesity involves significant genetic and biologic susceptibility factors that are highly influenced by environmental and psychosocial factors.

A patient receives atropine, an anticholinergic drug, in preparation for surgery. The nurse expects this drug to affect the GI tract by doing what? a. Increasing gastric emptying b. Relaxing pyloric and ileocecal sphincters c. Decreasing secretions and peristaltic action d. Stimulation the nervous system of the GI tract

Correct answer: c Rationale: The parasympathetic nervous system stimulates activity of the gastrointestinal (GI) tract, increasing motility and secretions and relaxing sphincters to promote movement of contents. A drug that blocks this activity decreases secretions and peristalsis, slows gastric emptying, and contracts sphincters. The enteric nervous system of the GI tract is modulated by sympathetic and parasympathetic influence.

Priority Decision: When caring for a patient who has had most of the stomach surgically removed, what is important for the nurse to teach the patient? a. Extra iron will need to be taken to prevent anemia. b. Avoid foods with lactose to prevent bloating and diarrhea. c. Lifelong supplementation of cobalamin (vitamin B12) will be needed. d. Because of the absence of digestive enzymes, protein malnutrition is likely.

Correct answer: c Rationale: The stomach secretes intrinsic factor, necessary for cobalamin (vitamin B12) absorption in the intestine. When part or all of the stomach is removed, cobalamin must be supplemented for life. The other options will not be a problem.

When the nurse is assessing the health perception-health maintenance pattern as related to GI function, an appropriate question to ask is a. "What is your usual bowel elimination pattern?" b. "What percentage of your income is spent on food?" c. "Have you traveled to a foreign country in the last year?" d. "Do you have diarrhea when you are under a lot of stress?"

Correct answer: c Rationale: When assessing gastrointestinal function in relation to the health perception-health management pattern, the nurse should ask the patient about recent foreign travel with possible exposure to hepatitis, parasitic infestation, or bacterial infection.

Which explanation about weight reduction should be included when teaching the obese patient and her obese husband? a. Weight gain is caused by psychologic factors. b. Daily weighing is recommended to monitor weight loss. c. Fat is not burned until the glycogen-water pool is depleted. d. Men lose weight less quickly than women because they have a higher percentage of metabolically less-active fat.

Correct answer: c Rationale: With reducing diets that severely restrict carbohydrates, the body's glycogen stores become depleted within a few days. The glycogen normally binds to water in fat cells and it is this water loss that causes weight loss in the first few days. Fat is not burned until the glycogen-water pool is depleted. Although psychosocial components (i.e., using food for comfort or reward and inability to buy high-nutritional quality food) may have an influence on weight gain, these factors along with lack of physical exercise, underestimation of portion size, and genetics contribute to weight gain. Weekly weighing is recommended as a more reliable indicator of weight loss because daily weighing shows frequent fluctuation from retained water (including urine) and elimination of feces. Men are able to lose weight more quickly than women because women have a higher percentage of metabolically less-active fat.

Which nursing actions are indicated for a liver biopsy (select all that apply)? a. Observe for white stools b. Monitor for rectal bleeding c. Monitor for internal bleeding d. Position to right side after test e. Ensure bowel preparation was done f. Check coagulation status before test

Correct answer: c, d, f Rationale: Because the liver is a vascular organ, vital signs are monitored to assess for internal bleeding. Prevention of bleeding is the reason for positioning on the right side for at least 2 hours and for splinting the puncture site. Again, because of the vasculature of the liver, coagulation status is checked before the biopsy is done. White stools occur with upper gastrointestinal (UGI) or barium swallow tests. No smoking is to be done after midnight before the study with an UGI. The bowel must be cleared before a lower GI or barium enema, a virtual colonoscopy, or a colonoscopy. Rectal bleeding may occur with a sigmoidoscopy or colonoscopy. A perforation may occur with an esophagogastroduodenoscopy (EGD), ERCP, or peritoneoscopy.

When teaching a patient about weight reduction diets, the nurse teaches the patient that an appropriate single serving of a food is a. a 6-inch bagel. b. 1 cup of chopped vegetables. c. a piece of cheese the size of three dice. d. a chicken breast the size of a deck of cards.

Correct answer: d Rationale: A chicken breast the size of a deck of cards is about 3 oz, a recommended portion size of meat. Other normal portions include a 3-inch bagel, 1/2 cup of chopped vegetables, and a piece of cheese the size of six dice.

Which patient is at highest risk for developing metabolic syndrome? a. A 62-year-old white man who has coronary artery disease with chronic stable angina b. A 54-year-old Hispanic woman who is sedentary and has nephrogenic diabetes insipidus c. A 27-year-old Asian American woman who has preeclampsia and gestational diabetes mellitus d. A 38-year-old Native American man who has diabetes mellitus and elevated hemoglobin A1C

Correct answer: d Rationale: African Americans, Hispanics, Native Americans, and Asians are at an increased risk for development of metabolic syndrome. Other risk factors include individuals who have diabetes that cannot maintain a normal glucose level, have hypertension, and secrete a large amount of insulin, or who have survived a heart attack and have hyperinsulinemia.

In the immediate postoperative period a nurse cares for a severely obese 72-year-old man who had surgery for repair of a lower leg fracture. Which assessment would be most important for the nurse to make? a. Cardiac rhythm b. Surgical dressing c. Postoperative pain d. Oxygen saturation

Correct answer: d Rationale: After surgery an older and/or severely obese patient should be closely monitored for oxygen desaturation. The body stores anesthetics in adipose tissue, placing patients with excess adipose tissue (e.g., obesity, older) at risk for resedation. As adipose cells release anesthetic back into the bloodstream, the patient may become sedated after surgery. This may depress the respiratory rate and result in a drop in oxygen saturation.

The nurse is performing a focused abdominal assessment of a patient who has been recently admitted. In order to palpate the patient's liver, where should the nurse palpate the patient's abdomen? a. Left lower quadrant b. Left upper quadrant c. Right lower quadrant d. Right upper quadrant

Correct answer: d Rationale: Although the left lobe of the liver is located in the left upper quadrant of the abdomen, the bulk of the liver is located in the right upper quadrant.

At the first visit to the clinic, the female patient with a BMI of 29 kg/m2 tells the nurse that she does not want to become obese. Which question used for assessing weight issues is the most important question for the nurse to ask? a. "What factors contributed to your current body weight?" b. "How is your overall health affected by your body weight?" c. "What is your history of gaining weight and losing weight?" d. "In what ways are you interested in managing your weight differently?"

Correct answer: d Rationale: Asking the patient about her desire to manage her weight in a different manner helps the nurse determine the patient's readiness for learning, degree of motivation, and willingness to change lifestyle habits. The nurse can help the patient set realistic goals. This question will also lead to discussing the patient's history of gaining and losing weight and factors that have contributed to the patient's current weight. The patient may be unaware of the overall health effects of her body weight, so this question is not helpful at this time.

After eating, a patient with an inflamed gallbladder experiences pain caused by contraction of the gallbladder. What is the mechanism responsible for this action? a. Production of bile by the liver b. Production of secretin by the duodenum c. Release of gastrin from the stomach antrum d. Production of cholecystokinin by the duodenum

Correct answer: d Rationale: Cholecystokinin is secreted by the duodenal mucosa when fats and amino acids enter the duodenum and stimulate the gallbladder to release bile to emulsify the fats for digestion. The bile is produced by the liver but stored in the gallbladder. Secretin is responsible for stimulating pancreatic bicarbonate secretion and gastrin increases gastric motility and acid secretion.

A severely obese patient has undergone Roux-en-Y gastric bypass surgery. In planning postoperative care, the nurse anticipates that the patient a. may have severe diarrhea early in the postoperative period. b. will not be allowed to ambulate for 1 to 2 days postoperatively. c. will require nasogastric suction until the incision heals. d. may have only liquids orally, and in very limited amounts, during the postoperative period.

Correct answer: d Rationale: During the immediate postoperative period, water and sugar-free clear liquids are given (30 mL every 2 hours while the patient is awake).

The obesity classification that is most often associated with cardiovascular health problems is a. primary obesity. b. secondary obesity. c. gynoid fat distribution. d. android fat distribution.

Correct answer: d Rationale: Individuals with fat located primarily in the abdominal area (i.e., whose body is apple-shaped) are at greater risk for obesity-related complications (e.g., heart disease) than are those whose fat is primarily located in the upper legs (i.e., whose body is pear-shaped). Individuals whose fat is distributed over the abdomen and upper body (i.e., neck, arms, and shoulders) are classified as having android obesity.

The nurse is reviewing the home medication list for a 44-year-old man admitted with suspected hepatic failure. Which medication could cause hepatotoxicity? a. Nitroglycerin b. Digoxin (Lanoxin) c. Ciprofloxacin (Cipro) d. Acetaminophen (Tylenol)

Correct answer: d Rationale: Many chemicals and drugs are potentially hepatotoxic (see Table 39-6) and result in significant patient harm unless monitored closely. For example, chronic high doses of acetaminophen and nonsteroidal antiinflammatory drugs (NSAIDs) may be hepatotoxic.

The nurse has completed initial instruction with a patient regarding a weight-loss program. Which patient comment indicates to the nurse that the teaching has been effective? a. "I will keep a diary of daily weight to illustrate my weight loss." b. "I plan to lose 4 lb a week until I have lost the 60 lb I want to lose." c. "I should not exercise more than what is required so I don't increase my appetite." d. "I plan to join a behavior modification group to help establish long-term behavior changes."

Correct answer: d Rationale: People who have undergone behavior therapy are more successful in maintaining weight losses over time because most programs deemphasize the diet, focus on how and when the person eats and education, and provide support from others. Weighing daily is not recommended and plateaus may not allow for consistent weight loss. A goal for weight loss must be set and 1 to 2 pounds a week is realistic. A more rapid loss often causes skin and underlying tissue to lose elasticity and become flabby folds of tissue. Exercising more often depresses appetite and exercise need not be limited.

A patient is admitted to the hospital with a diagnosis of diarrhea with dehydration. The nurse recognizes that increased peristalsis resulting in diarrhea can be related to a. sympathetic inhibition. b. mixing and propulsion. c. sympathetic stimulation. d. parasympathetic stimulation.

Correct answer: d Rationale: Peristalsis is increased by parasympathetic stimulation.

The best nutritional therapy plan for a person who is obese is a. the Zone diet. b. the Atkins diet. c. Sugar Busters. d. foods from the basic food groups.

Correct answer: d Rationale: Restricted food intake is a cornerstone for any weight loss or maintenance program. A good weight loss plan should include foods from the basic food groups.

The severely obese patient has elected to have the Roux-en-Y gastric bypass (RYGB) procedure. The nurse will know the patient understands the preoperative teaching when the patient makes which statement? a. "This surgery will preserve the function of my stomach." b. "This surgery will remove the fat cells from my abdomen." c. "This surgery can be modified whenever I need it to be changed." d. "This surgery decreases how much I can eat and how many calories I can absorb."

Correct answer: d Rationale: The RYGB decreases the size of the stomach to a gastric pouch and attaches it directly to the small intestine so food bypasses 90% of the stomach, the duodenum, and a small segment of the jejunum. The vertical sleeve gastrectomy removes 85% of the stomach, but preserves the function of the stomach. Lipectomy and liposuction remove fat tissue from the abdomen or other areas. Adjustable gastric banding can be modified or reversed at a later date.

Which patient has the highest morbidity risk? a. Male 6 ft. 1 in. tall, BMI 29 kg/m2 b. Female 5 ft. 6 in. tall, weight 150 lb. c. Male with waist circumference 46 in. d. Female 5 ft. 10 in. tall, obesity Class III

Correct answer: d Rationale: The patient in Class III obesity has the highest risk for disease because Class III denotes severe obesity or a BMI greater than 40 kg/m2. The patient with waist circumference 46 in. has a high risk for disease, but without the BMI or obesity class, a more precise determination cannot be made. The female who is 5 ft. 6 in. tall has a normal weight for her height. The male patient who is over 6 ft. tall is overweight, which increases his risk of disease, but a more precise determination cannot be made without the waist circumference.

What is a clinical manifestation of age-related changes in the GI system that the nurse may find in an older patient? a. Gastric hyperacidity b. Intolerance to fatty foods c. Yellowish tinge to the skin d. Reflux of gastric contents into the esophagus

Correct answer: d Rationale: There is decreased tone of the lower esophageal sphincter with again and regurgitation of gastric contents back into the esophagus occurs, causing heartburn and belching. There is a decrease in hydrochloric acid secretion with aging. Jaundice and intolerance to fatty foods are symptoms of liver or gallbladder disease and are not normal age-related findings.

What is a postoperative nursing intervention for the obese patient who has undergone bariatric surgery? a. Irrigating and repositioning the nasogastric (NG) tube as needed b. Delaying ambulation until the patient has enough strength to support self c. Keeping the patient positioned on the side to facilitate respiratory function d. Providing adequate support to the incision during coughing, deep breathing, and turning

Correct answer: d Rationale: Turning, coughing, and deep breathing are essential to prevent postoperative complications. Protecting the incision from strain is important since wound dehiscence is a problem for obese patients. If a nasogastric (NG) tube that is present following gastric surgery for severe obesity becomes blocked or needs repositioning, the health care provider should be notified. Ambulation is usually started on the evening of surgery and addition help will be needed to support the patient. Respiratory function is promoted by keeping the head of the bed elevated at an angle of 35 to 40 degrees.

A 68-year-old patient is in the office for a physical. She notes that she no longer has regular bowel movements. Which suggestion by the nurse would be most helpful to the patient? a. Take an additional laxative to stimulate defecation. b. Eat less acidic foods to enable the gastrointestinal system to increase peristalsis. c. Eat less food at each meal to prevent feces from backing up related to slowed peristalsis. d. Attempt defecation after breakfast because gastrocolic reflexes increase colon peristalsis at that time.

Correct answer: d Rationale: When food inters the stomach and duodenum, the gastrocolic and duodenocolic reflexes are initiated and are more active after the first daily meal. Additional laxatives or laxative abuse contribute to constipation in older adults. Decreasing food intake is not recommended, as many older adults have a decreased appetite. Fibre and fluids should be increased.

The ED nurse has inspected, auscultated, and palpated the abdomen with no obvious abnormalities, except pain. When the nurse palpates the abdomen for rebound tenderness, there is severe pain. The nurse should know that this could indicate what problem? a. Hepatic cirrhosis b. Hypersplenomegaly c. Gall bladder distention d. Peritoneal inflammation

Correct answer: d Rationale: When palpating for rebound tenderness, the problem area of the abdomen will produce pain and severe muscle spasm when there is peritoneal inflammation. Hepatic cirrhosis, hypersplenomegaly, and gall bladder distention do not manifest with rebound tenderness.

This bariatric surgical procedure involves creating a stoma and gastric pouch that is reversible, and no malabsorption occurs. What surgical procedure is this? a. Vertical gastric banding b. Biliopancreatic diversion c. Roux-en-Y gastric bypass d. Adjustable gastric banding

Correct answer: d Rationale: With adjustable gastric banding (AGB), the stomach size is limited by an inflatable band placed around the fundus of the stomach. The band is connected to a subcutaneous port and can be inflated or deflated to change the stoma size to meet the patient's needs as weight is lost. The procedure is performed laparoscopically and, if necessary, can be modified or reversed after the initial procedure.

Wich of the following criteria must be met for a diagnosis of metabolic syndrome (select all that apply)? a. Hypertension b. Elevated triglycerides c. Elevated plasma glucose d. Increased waist circumference e. Decreased low-density lipoproteins

Correct answers: a, b, c, d Rationale: Three of the following five criteria must be met for a diagnosis of metabolic syndrome: • Waist circumference of 40 inches or more in men and 35 inches or more in women • Triglyceride levels higher than 150 mg/dL, or need for drug treatment for elevated triglyceride levels • High-density lipoprotein (HDL) cholesterol levels lower than 40 mg/dL in men and lower than 50 mg/dL in women, or need for drug treatment for reduced HDL cholesterol levels • Blood pressure: 130 mm Hg or higher systolic or 85 mm Hg or higher diastolic, or need for drug treatment for hypertension • Fasting blood glucose level of 110 mg/dL or higher, or need for drug treatment for elevated glucose levels

Normally, which hormones and peptides affect appetite (select all that apply)? a. Leptin b. Insulin c. Ghrelin d. Peptide YY e. Neuropeptide Y f. Cholecystokinin

Correct answers: a, b, c, d, e, f Rationale: Normally ghrelin and neuropeptide Y stimulate appetite. Leptin suppresses appetite and hunger. Insulin decreases appetite. Peptide YY and cholecystokinin inhibit appetite by slowing gastric emptying and sending satiety signals to the hypothalamus.

A normal physical assessment finding of the GI system is/are (select all that apply) a. nonpalpable liver and spleen. b. borborygmi in upper right quadrant. c. tympany on percussion of the abdomen. d. liver edge 2 to 4 cm below the costal margin. e. finding of a firm, nodular edge on the rectal examination.

Correct answers: a, c Rationale: Normal assessment findings for the gastrointestinal system include a nonpalpable liver and spleen and generalized tympany on percussion. Normally, bowel sounds are high pitched and gurgling; loud gurgles indicate hyperperistalsis and are called borborygmi (stomach growling). If the patient has chronic obstructive pulmonary disease, large lungs, or a low-set diaphragm, the liver may be palpated 0.4 to 0.8 inch (1 to 2 cm) below the right costal margin. On palpation, the rectal wall should be soft and smooth and should have no nodules.

Priority Decision: The nurse is teaching a moderately obese woman interventions for the management of obesity. Initially, which strategies will support restricting dietary intake to below energy requirements (select all that apply)? a. Limit alcohol b. Rest when fatigued c. Determine portion sizes d. 1800- to 2200-calorie diet e. Attend Overeaters Anonymous

Correct answers: a, c Rationale: To restrict dietary intake so that it is below energy requirements, the moderately obese woman should limit or avoid alcohol intake because it increases caloric intake and has low nutritional value. Portion sizes have increased over the years and are larger than they should be. Teach the patient to determine portion sizes by weight or learn equivalencies such as that a serving of fruit is the size of a baseball. A progressive exercise program will increase energy requirements and a diet with an initial 800- to 1200-calorie limit would decrease calorie intake. Overeaters Anonymous would not restrict dietary intake below energy requirements, although it may offer support for the patient.

Health risks associated with obesity include (select all that apply) a. colorectal cancer. b. rheumatoid arthritis. c. polycystic ovary syndrome. d. nonalcoholic steatohepatitis. e. systemic lupus erythematosus.

Correct answers: a, c, d Rationale: Health risks associated with obesity include cardiovascular disease (related to increased low-density lipoprotein levels, increased triglyceride levels, and decreased high-density lipoprotein levels), hypertension, sleep apnea, obesity hypoventilation syndrome, reduced chest wall compliance, increased work of breathing, decreased total lung capacity and functional residual capacity, type 2 diabetes mellitus (i.e., hyperinsulinemia and insulin resistance), osteoarthritis, hyperuricemia, gout, gastroesophageal reflux disease, gallstones, nonalcoholic steatohepatitis, fatty liver and cirrhosis, cancer (mainly breast, endometrial, kidney, colorectal, pancreas, esophagus, and gallbladder), psychosocial problems (employment, education, and health care), low self-esteem, withdrawal from social interactions, and major depression.

Which teaching points are important when providing information to a patient with metabolic syndrome (select all that apply)? a. Stop smoking. b. Monitor weight daily. c. Increase level of activity. d. Decrease saturated fat intake. e. Reduce weight and maintain lower weight. f. Check blood glucose each morning prior to eating.

Correct answers: a, c, d, e Rationale: Patients with metabolic syndrome need to lower their risk factors by reducing and maintaining weight, increasing physical activity, establishing healthy diet habits, and smoking cessation. Some patients with metabolic syndrome are diabetic and would need to monitor glucose levels frequently. When monitoring weight reduction, it is recommended to check weight weekly, not daily.

A community health nurse is conducting an initial assessment of a new patient. Which assessments should the nurse include when screening the patient for metabolic syndrome (select all that apply)? a. Blood pressure b. Resting heart rate c. Physical endurance d. Waist circumference e. Fasting blood glucose

Correct answers: a, d, e Rationale: The diagnostic criteria for metabolic syndrome include elevated blood pressure, fasting blood glucose, waist circumference, triglycerides, and HDL cholesterol. Resting heart rate and physical endurance are not part of the diagnostic criteria.

What characteristics describe adjustable gastric banding (select all that apply)? a. 85% of the stomach is removed. b. Stomach restriction can be reversed. c. Eliminates hormones that stimulate hunger. d. Malabsorption of fat-soluble vitamins occurs. e. Inflatable band allows for modification of gastric stoma size. f. Stomach with a gastric pouch surgically anastomosed to the jejunum.

Correct answers: b, e Rationale: The adjustable gastric banding procedure is reversible and allows a change in gastric stoma size by inflation or deflation of the band around the funds of the stomach. The vertical sleeve gastrectomy removes 85% of the stomach and eliminates the hormones produced in the stomach that stimulate hunger. The biliopancreatic diversion is a maladaptive surgery that prevents absorption of nutrients, including fat-soluble vitamins. The Roux-en-Y gastric bypass reduces the stomach size with a gastric pouch anastomosed to the small intestine, so it is both restrictive and malabsorptive.

In preparing to care for the obese patient with cancer, what physiologic problems is this patient at a greater risk for having (select all that apply)? a. Tinnitus b. Fractures c. Sleep apnea d. Trousseau's sign e. Type 2 diabetes mellitus f. Gastroesophageal reflux disease (GERD)

Correct answers: c, e, f Rationale: Obese patients are at a higher risk for cancer, sleep apnea and sleep deprivation, type 2 diabetes mellitus, gastroesophageal reflux disease (GERD), nonalcoholic steatohepatits, osteoarthritis, and cardiovascular problems. The other options are not related to obesity.

A patient has received atropine before surgery and complains of dry mouth. Which action by the nurse is best? a. Check for skin tenting. b. Notify the health care provider. c. Ask the patient about any dizziness. d. Tell the patient dry mouth is an expected side effect.

D

A patient who takes a diuretic and a β-blocker to control blood pressure is scheduled for breast reconstruction surgery. Which patient information is most important to communicate to the health care provider before surgery? a. Hematocrit 36% b. Blood pressure 144/82 c. Pulse rate 58 beats/minute d. Serum potassium 3.2 mEq/L

D

As the nurse prepares a patient the morning of surgery, the patient refuses to remove a wedding ring, saying, "I have never taken it off since the day I was married." Which response by the nurse is best? a. Have the patient sign a release and leave the ring on. b. Tape the wedding ring securely to the patient's finger. c. Tell the patient that the hospital is not liable for loss of the ring. d. Suggest that the patient give the ring to a family member to keep.

D

The nurse interviews a patient scheduled to undergo general anesthesia for a hernia repair. Which information is most important to communicate to the surgeon and anesthesiologist before surgery? a. The patient drinks 3 or 4 cups of coffee every morning before going to work. b. The patient takes a baby aspirin daily but stopped taking aspirin 10 days ago. c. The patient drank 4 ounces of apple juice 3 hours before coming to the hospital. d. The patient's father died after receiving general anesthesia for abdominal surgery.

D

The nurse obtains a health history from a patient who is scheduled for elective hip surgery in 1 week. The patient reports use of garlic and ginkgo biloba. Which action by the nurse is most appropriate? a. Ascertain that there will be no interactions with anesthetic agents. b. Teach the patient that these products may be continued preoperatively. c. Advise the patient to stop the use of all herbs and supplements at this time. d. Discuss the herb and supplement use with the patient's health care provider.

D

The outpatient surgery nurse reviews the complete blood cell (CBC) count results for a patient who is scheduled for surgery in a few days. The results are white blood cell (WBC) count 10.2 × 103/µL; hemoglobin 15 g/dL; hematocrit 45%; platelets 150 × 103/µL. Which action should the nurse take? a. Call the surgeon and anesthesiologist immediately. b. Ask the patient about any symptoms of a recent infection. c. Discuss the possibility of blood transfusion with the patient. d. Send the patient to the holding area when the operating room calls.

D

A patient's recent medical history is indicative of diabetes insipidus. The nurse would perform patient teaching related to which diagnostic test? A) Thyroid scan B) Fasting glucose test C) Oral glucose tolerance D) Water deprivation test

D) A water deprivation test is used to diagnose the polyuria that accompanies diabetes insipidus. Glucose tests and thyroid tests are not directly related to the diagnosis of diabetes insipidus.

What is a priority nursing intervention in the care of a patient with a diagnosis of chronic venous insufficiency (CVI)? A) Application of topical antibiotics to venous ulcers B) Maintaining the patient's legs in a dependent position C) Administration of oral and/or subcutaneous anticoagulants D) Teaching the patient the correct use of compression stockings

D) CVI requires conscientious and consistent application of compression stockings. Anticoagulants are not necessarily indicated and antibiotics, if required, are typically oral or IV, not topical. The patient should avoid prolonged positioning with the limb in a dependent position.

The nurse should monitor for increases in which laboratory value for the patient as a result of being treated with dexamethasone (Decadron)? A) Sodium B) Calcium C) Potassium D) Blood glucose

D) Hyperglycemia or increased blood glucose level is an adverse effect of corticosteroid therapy. Sodium, calcium, and potassium levels are not directly affected by dexamethasone.

A patient with a severe pounding headache has been diagnosed with hypertension. However, the hypertension is not responding to traditional treatment. What should the nurse expect as the next step in management of this patient? A) Administration of β-blocker medications B) Abdominal palpation to search for a tumor C) Administration of potassium-sparing diuretics D) A 24-hour urine collection for fractionated metanephrines

D) Pheochromocytoma should be suspected when hypertension does not respond to traditional treatment. The 24-hour urine collection for fractionated metanephrines is simple and reliable with elevated values in 95% of people with pheochromocytoma. In a patient with pheochromocytoma preoperatively an α-adrenergic receptor blocker is used to reduce BP. Abdominal palpation is avoided to avoid a sudden release of catecholamines and severe hypertension. Potassium-sparing diuretics are not needed. Most likely they would be used for hyperaldosteronism, which is another cause of hypertension.

A patient reports having dry mouth and asks for some liquid to drink. The nurse reasons that this symptom can most likely be attributed to a common adverse effect of which of the following medications? A) Digoxin (Lanoxin) B) Cefotetan (Cefotan) C) Famotidine (Pepcid) D) Promethazine (Phenergan)

D) Promethazine (Phenergan) A common adverse effect of promethazine, an antihistamine antiemetic agent, is dry mouth; another is blurred vision.

The hypothalamus secretes releasing hormones and inhibiting hormones. What is the target tissue of these releasing hormones and inhibiting hormones? A) Pineal B) Adrenal cortex C) Posterior pituitary D) Anterior pituitary

D) The anterior pituitary is the target tissue of the releasing hormones (corticotropin releasing hormone, thyrotropin releasing hormone, growth hormone releasing factor, gonadotropin releasing hormone, prolactin releasing factor) and the inhibiting hormones (somatostatin, prolactin inhibiting factor). These hormones release or inhibit other hormones that affect the thyroid, adrenal cortex, pancreas, reproductive organs, and all body cells. The pineal gland is not directly affected by the releasing and inhibiting hormones from the hypothalamus. The posterior pituitary releases antidiuretic hormone (ADH) in response to plasma osmolality changes that is not directly affected by the hypothalamus hormones.

The patient reports tenderness when she touches her leg over a vein. The nurse assesses warmth and a palpable cord in the area. The nurse knows the patient needs treatment to prevent which sequelae? A) Pulmonary embolism Pulmonary hypertension Post-thrombotic syndrome Venous thromboembolism

D) The clinical manifestations are characteristic of a superficial vein thrombosis. If untreated, the clot may extend to deeper veins, and venous thromboembolism may occur. Pulmonary embolism, pulmonary hypertension, and post-thrombotic syndrome are the sequelae of venous thromboembolism.

A male patient was admitted for a possible ruptured aortic aneurysm, but had no back pain. Ten minutes later his assessment includes the following: sinus tachycardia at 138, BP palpable at 65 mm Hg, increasing waist circumference, and no urine output. How should the nurse interpret this assessment about the patient's aneurysm? A) Tamponade will soon occur. B) The renal arteries are involved. C) Perfusion to the legs is impaired. D) He is bleeding into the abdomen.

D) The lack of back pain indicates the patient is most likely exsanguinating into the abdominal space, and the bleeding is likely to continue without surgical repair. A blockade of the blood flow will not occur in the abdominal space as it would in the retroperitoneal space where surrounding anatomic structures may control the bleeding. The lack of urine output does not indicate renal artery involvement, but that the bleeding is occurring above the renal arteries, which decreases the blood flow to the kidneys. There is no assessment data indicating decreased perfusion to the legs.

The nurse is preparing to administer a scheduled dose of enoxaparin (Lovenox) 30 mg subcutaneously. What should the nurse do to administer this medication correctly? A) Remove the air bubble in the prefilled syringe. B) Aspirate before injection to prevent IV administration. C) Rub the injection site after administration to enhance absorption. D) Pinch the skin between the thumb and forefinger before inserting the needle.

D) The nurse should gather together or "bunch up" the skin between the thumb and the forefinger before inserting the needle into the subcutaneous tissue. The nurse should not remove the air bubble in the prefilled syringe, aspirate, nor rub the site after injection.

A 50-year-old female patient smokes, is getting a divorce, and is reporting eye problems. On assessment of this patient, the nurse notes exophthalmos. What other abnormal assessments should the nurse expect to find in this patient? A) Puffy face, decreased sweating, and dry hair B) Muscle aches and pains and slow movements C) Decreased appetite, increased thirst, and pallor D) Systolic hypertension and increased heart rate

D) The patient's manifestations point to Graves' disease or hyperthyroidism, which would also include systolic hypertension and increased heart rate and increased thirst. Puffy face, decreased sweating; dry, coarse hair; muscle aches and pains and slow movements; decreased appetite and pallor are all manifestations of hypothyroidism.

The patient with an adrenal hyperplasia is returning from surgery for an adrenalectomy. For what immediate postoperative risk should the nurse plan to monitor the patient? A) Vomiting B) Infection C) Thomboembolism D) Rapid BP changes

D) The risk of hemorrhage is increased with surgery on the adrenal glands as well as large amounts of hormones being released in the circulation, which may produce hypertension and cause fluid and electrolyte imbalances to occur for the first 24 to 48 hours after surgery. Vomiting, infection, and thromboembolism may occur postoperatively with any surgery.

The nurse is caring for a patient with a recent history of deep vein thrombosis (DVT). The patient now needs to undergo surgery for appendicitis. The nurse is reviewing the laboratory results for this patient before administering an ordered dose of vitamin K. The nurse determines that the medication is both safe to give and is most needed when the international normalized ratio (INR) is which result? A) 1.0 B) 1.2 C) 1.6 D) 2.2

D) Vitamin K is the antidote to warfarin (Coumadin), which the patient has most likely been taking before admission for treatment of DVT. Warfarin is an anticoagulant that impairs the ability of the blood to clot. Therefore it is necessary to give vitamin K before surgery to reduce the risk of hemorrhage. The largest value of the INR indicates the greatest impairment of clotting ability, making 2.2 the correct selection.

When administering a scheduled dose of pilocarpine, in which area should the nurse place the drops? A. Inner canthus B. Outer canthus C. Center of the eyeball D. Lower conjunctival sac

D. Ocular medications such as pilocarpine should be instilled into the lower conjunctival sac. Never apply eye drops directly to the cornea. Applying the drops to the inner canthus will cause them to be distributed systemically.

When performing teaching with a patient with glaucoma while administering a scheduled dose of pilocarpine, the nurse would include which statement? A. "Prolonged eye irritation is an expected adverse effect of this medication." B. "This medication will help to raise intraocular pressure to a near normal level." C. "This medication needs to be continued for at least 5 years after your initial diagnosis." D. "It is important not to do activities requiring visual acuity immediately after administration."

D. Pilocarpine causes blurred vision and difficulty in focusing, so it is important not to engage in any activities requiring visual acuity until the vision clears.

A patient with septic shock is receiving multiple medications. Which IV medication is most likely to cause a hearing loss? a. Dopamine (Intropin) b. Ampicillin (Principen) c. Aspirin (Bayer Aspirin) d. Vancomycin (Vancocin)

D. The IV medication in use that is most likely to cause a hearing loss is vancomycin (Vancocin) because it is an ototoxic medication. For that reason, serum drug levels are monitored to maintain therapeutic levels and reduce the risk of ototoxicity. Aspirin can also cause hearing loss, but it is not administered IV. Neither dopamine nor ampicillin is likely to cause hearing loss.

A patient with poor visual acuity is diagnosed with age-related macular degeneration (AMD). Which nursing intervention should be the nurse's priority? A. Teach about visual enhancement techniques. B. Teach nutritional strategies to improve vision. C. Assess coping strategies and support systems. D. Assess impact of vision on normal functioning.

D. The most important nursing intervention is to assess the patient's ability to function with the visual impairment. The nurse will use this information to plan nursing care including assessment of the patient's coping strategies and teaching about vision enhancement techniques and nutrition.

The patient needs, but does not want, a corneal transplant because of the difficulty with vision that may last for up to 12 months after the transplant. What can the nurse teach the patient about this? A. If the transplant is done soon after the donor dies, there will not be as much trouble recovering vision. B. The astigmatism the patient is experiencing may be corrected with glasses or rigid contact lenses. C. Increasing the amount of light and using a magnifier to read will be helpful if a transplant is not wanted. D. There are newer procedures where only the damaged cornea epithelial layer is replaced, and they have a faster recovery.

D. The new procedures are called Descemet's stripping endothelial keratoplasty (DSEK) and Descemet's membrane endothelial keratoplasty (DMEK). Corneal transplants should be done as soon as possible, but this does not affect the rate of visual recovery. Astigmatism is not experienced with corneal scars and opacities requiring a corneal transplant. Increasing light and magnification helps a person with cataracts to read.

Before administrating timolol (Timoptic) eye drops for treatment of glaucoma, the nurse would assess the patient for which contraindication for the use of this medicine? A. Sinusitis B. Migraine headaches C. Chronic urinary tract infection D. Chronic obstructive pulmonary disease

D. Timolol is a nonselective β-adrenergic blocker that could lead to bronchoconstriction and bronchospasm. For this reason, it should not be used in patients with COPD. Timolol may be used to treat migraine headaches, and it does not affect sinusitis or chronic urinary tract infections.

The nurse instructs a 66-year-old patient prescribed dipivefrin (Propine) eye drops to manage chronic open-angle glaucoma. Which statement, if made by the patient to the nurse, indicates that further teaching is needed? A. "The eye drops could cause a fast heart rate and high blood pressure." B. "I will need to take the eye drops twice a day for at least 2 to 3 months." C. "I may experience eye discomfort and redness from the use of these eye drops." D. "I will apply gentle pressure on the inside corner of my eye after each eye drop."

D. To avoid systemic reactions such as tachycardia and hypertension, the patient should apply punctual occlusion after instillation of the eye drops. Dipivefrin will control chronic open-angle glaucoma but will not cure the disease. Side effects associated with dipivefrin include ocular discomfort and redness, tachycardia, and hypertension.

The nurse is assessing an 86-year-old female who has just been transferred to the long-term care facility. Which assessment question will best allow the nurse to assess the woman for the presence of presbycusis? A. "Do you ever experience any ringing in your ears?" B. "Have you ever fallen down because you became dizzy?" C. "Do you ever have pain in your ears when you're chewing or swallowing?" D. "Have you noticed any change in your hearing in recent months and years?"

D. Presbycusis is an age-related change in auditory acuity. Ringing in the ears is termed tinnitus whereas dizziness and falls are related to balance and the function of the vestibular system. Presbycusis is not associated with pain during chewing and swallowing.

An 82-year-old patient is frustrated by her flabby belly and rigid hips. What should the nurse tell the patient about these frustrations? A. "You should go on a diet and exercise more to feel better about yourself." B. "Something must be wrong with you because you should not have these problems." C. "You have arthritis and need to go on nonsteroidal antiinflammatory drugs (NSAIDs)." D. "Decreased muscle mass and strength and increased hip rigidity are normal changes of aging."

D. "Decreased muscle mass and strength and increased hip rigidity are normal changes of aging." The musculoskeletal system's normal changes of aging include decreased muscle mass and strength; increased rigidity in the hips, neck, shoulders, back, and knees; decreased fine motor dexterity; and slowed reaction times. Going on a diet and exercising will help but not stop these changes. Telling the patient "Something must be wrong with you..." will not be helpful to the patient's frustrations.

A patient is being treated for second and third-degree burns over 30% of his body and is now ready for discharge. The nurse provides discharge instructions related to wound care. What statement indicates that the patient understands the instruction? A. "i can expect occasional periods of low-grade fever and can take tylenol every 4 hours" B. "I must wear my Jobst elastic garment all day and can only remove it when I'm going to bed" C. "I will need to take sponge baths at home to avoid exposing the wounds to unsterile bath water" D. "if any healed areas break open I should cover then with a sterile dressing and immediately report it"

D. "if any healed areas break open I should cover then with a sterile dressing and immediately report it"

Which assessment finding of the respiratory system does the nurse interpret as abnormal? A. Inspiratory chest expansion of 1 in B. Percussion resonance over the lung bases C. Symmetric chest expansion and contraction D. Bronchial breath sounds in the lower lung fields

D. Bronchial breath sounds in the lower lung fields Bronchial or bronchovesicular sounds heard in the peripheral lung fields are abnormal breath sounds.

After swallowing, a 73-year-old patient is coughing and has a wet voice. What changes of aging could be contributing to this abnormality? A. Decreased response to hypercapnia B. Decreased number of functional alveoli C. Increased calcification of costal cartilage D. Decreased respiratory defense mechanisms

D. Decreased respiratory defense mechanisms These manifestations are associated with aspiration, which more easily occur in the right lung as the right mainstem bronchus is shorter, wider, and straighter than the left mainstem bronchus. Aspiration occurs more easily in the older patient related to decreased respiratory defense mechanisms (e.g., decreases in immunity, ciliary function, cough force, sensation in pharynx). Changes of aging include a decreased response to hypercapnia, decreased number of functional alveoli, and increased calcification of costal cartilage, but these do not increase the risk of aspiration.

Which teaching helps to prevent botulism? A. Do not eat raw shellfish from contaminated water. B. Avoid consuming raw or undercooked eggs. C. Boil water for 10 minutes if unsure of the source. D. Discard any canned food with a swollen end.

D. Discard any canned food with a swollen end. A can's swollen ends may be caused by gases from C. botulinum and should be discarded. Improper food canning is a major cause of botulism. The other options are not related to botulism.

The nurse is reinforcing health teaching about osteoporosis with a 72-year-old patient admitted to the hospital. In reviewing this disorder, what should the nurse explain to the patient? A. With a family history of osteoporosis, there is no way to prevent or slow bone resorption. B. Continuous, low-dose corticosteroid treatment is effective in stopping the course of osteoporosis. C. Estrogen therapy must be maintained to prevent rapid progression of the osteoporosis. D. Even with a family history of osteoporosis, the calcium loss from bones can be slowed by increased calcium intake and exercise.

D. Even with a family history of osteoporosis, the calcium loss from bones can be slowed by increased calcium intake and exercise. The rate of progression of osteoporosis can be slowed if the patient takes calcium supplements and/or foods high in calcium and engages in regular weight-bearing exercise. Corticosteroids interfere with bone metabolism. Estrogen therapy is no longer used to prevent osteoporosis because of the associated increased risk of heart disease and breast and uterine cancer.

What should the nurse inspect when assessing a patient with shortness of breath for evidence of long-standing hypoxemia? A. Chest excursion B. Spinal curvatures C. Respiratory pattern D. Fingernails and their base

D. Fingernails and their base Clubbing, a sign of long-standing hypoxemia, is evidenced by an increase in the angle between the base of the nail and the fingernail to 180 degrees or more, usually accompanied by an increase in the depth, bulk, and sponginess of the end of the finger.

When planning care for a patient with a C5 spinal cord injury, which nursing diagnosis is the highest priority? A. Impaired tissue integrity due to paralysis B. Impaired urinary elimination due to quadriplegia C. Ineffective coping due to the extent of trauma D. Ineffective airway clearance due to high cervical spinal cord injury

D. Ineffective airway clearance due to high cervical spinal cord injury Maintaining a patent airway is the most important goal for a patient with a high cervical fracture. Although all of these options are appropriate nursing diagnoses for a patient with a spinal cord injury, respiratory needs are always the highest priority. Remember the ABCs.

A patient is admitted to the intensive care unit (ICU) with a C7 spinal cord injury and diagnosed with Brown-Séquard syndrome. What would you most likely find on physical examination? A. Upper extremity weakness only B. Complete motor and sensory loss below C7 C. Loss of position sense and vibration in both lower extremities D. Ipsilateral motor loss and contralateral sensory loss below C7

D. Ipsilateral motor loss and contralateral sensory loss below C7 Brown-Séquard syndrome is a result of damage to one half of the spinal cord. This syndrome is characterized by a loss of motor function and position and vibratory sense, as well as vasomotor paralysis on the same side (ipsilateral) as the lesion. The opposite (contralateral) side has loss of pain and temperature sensation below the level of the lesion.

A patient is admitted to the hospital with a C4 spinal cord injury after a motorcycle collision. The patient's blood pressure is 83/49 mm Hg and pulse is 39 beats/minute. He remains orally intubated. What is the cause of this pathophysiologic response? A. Increased vasomotor tone after the injury B. A temporary loss of sensation and flaccid paralysis below the level of injury C. Loss of parasympathetic nervous system innervation resulting in vasoconstriction D. Loss of sympathetic nervous system innervation resulting in peripheral vasodilation

D. Loss of sympathetic nervous system innervation resulting in peripheral vasodilation eurogenic shock results from loss of vasomotor tone caused by injury, and it is characterized by hypotension and bradycardia. Loss of sympathetic nervous system innervation causes peripheral vasodilation, venous pooling, and a decreased cardiac output. These effects usually are associated with a cervical or high thoracic injury (T6 or higher).

The nurse cares for a 58-year-old woman with breast cancer who is admitted for severe back pain related to a compression fracture. The patient's laboratory values include serum potassium of 4.5 mEq/L, serum sodium of 144 mEq/L, and serum calcium of 14.3 mg/dL. Which signs and symptoms will the nurse expect the patient to exhibit? A. Anxiety, irregular pulse, and weakness B. Muscle stiffness, dysphagia, and dyspnea C. Hyperactive reflexes, tremors, and seizures D. Nausea, vomiting, and altered mental status

D. Nausea, vomiting, and altered mental status Breast cancer can metastasize to the bone. Vertebrae are a common site. Pathologic fractures at the site of metastasis are common because of a weakening of the involved bone. High serum calcium levels result as calcium is released from damaged bones. Normal serum calcium is between 8.6 to 10.2 mg/dL. Clinical manifestations of hypercalcemia include nausea, vomiting, and altered mental status (e.g., lethargy, decreased memory, confusion, personality changes, psychosis, stupor, coma). Other manifestations include weakness, depressed reflexes, anorexia, bone pain, fractures, polyuria, dehydration, and nephrolithiasis. Manifestations of hypomagnesemia include hyperactive reflexes, tremors, and seizures. Symptoms of hyperkalemia include anxiety, irregular pulse, and weakness. Symptoms of hypocalcemia include muscle stiffness, dysphagia, and dyspnea.

During routine assessment of a patient with Guillain-Barré syndrome, you find the patient is short of breath. What is causing the patient's respiratory distress? A. Elevated protein levels in the cerebrospinal fluid (CSF) B. Immobility resulting from ascending paralysis C. Degeneration of motor neurons in the brainstem and spinal cord D. Paralysis ascending to the nerves that stimulate the thoracic area

D. Paralysis ascending to the nerves that stimulate the thoracic area Guillain-Barré syndrome is characterized by ascending, symmetric paralysis that usually affects cranial nerves and the peripheral nervous system. The most serious complication of this syndrome is respiratory failure, which occurs as the paralysis progresses to the nerves that innervate the thoracic area.

Which patient should be assigned to the experienced registered nurse on a neurologic floor? A. Patient with trigeminal neuralgia reporting facial pain rated at 10 B. Patient with Bell's palsy with unilateral facial droop C. Patient after surgical removal of a spinal cord tumor who is scheduled for discharge tomorrow D. Patient with traumatic injury to the cervical spinal cord who was admitted today from the emergency department

D. Patient with traumatic injury to the cervical spinal cord who was admitted today from the emergency department The patient with the cervical spinal cord injury is potentially the most unstable and needs an experienced, professional nurse.

A 67-year-old patient hospitalized with osteomyelitis has an order for bed rest with bathroom privileges with the affected foot elevated on two pillows. The nurse would place highest priority on which intervention? A. Ambulate the patient to the bathroom every 2 hours. B. Ask the patient about preferred activities to relieve boredom. C. Allow the patient to dangle legs at the bedside every 2 to 4 hours. D. Perform frequent position changes and range-of-motion exercises.

D. Perform frequent position changes and range-of-motion exercises. The patient is at risk for atelectasis of the lungs and for contractures because of prescribed bed rest. For this reason, the nurse should place the priority on changing the patient's position frequently to promote lung expansion and performing range-of-motion (ROM) exercises to prevent contractures. Assisting the patient to the bathroom will keep the patient safe as the patient is in pain, but it may not be needed every 2 hours. Providing activities to relieve boredom will assist the patient to cope with the bed rest, and dangling the legs every 2 to 4 hours may be too painful. The priority is position changes and ROM exercises.

A patient with a recent history of a dry cough has had a chest x-ray that revealed the presence of nodules. In an effort to determine whether the nodules are malignant or benign, what is the primary care provider likely to order? A. Thoracentesis B. Pulmonary angiogram C. CT scan of the patient's chest D. Positron emission tomography (PET)

D. Positron emission tomography (PET) PET is used to distinguish benign and malignant pulmonary nodules. Because malignant lung cells have an increased uptake of glucose, the PET scan (which uses an IV radioactive glucose preparation) can demonstrate increased uptake of glucose in malignant lung cells. This differentiation cannot be made using CT, a pulmonary angiogram, or thoracentesis.

When the patient is experiencing metabolic acidosis secondary to type 1 diabetes mellitus, what physiologic response should the nurse expect to assess in the patient? A. Vomiting B. Increased urination C. Decreased heart rate D. Rapid respiratory rate

D. Rapid respiratory rate When a patient with type 1 diabetes has hyperglycemia and ketonemia causing metabolic acidosis, the physiologic response is to increase the respiratory rate and tidal volume to blow off the excess CO2. Vomiting and increased urination may occur with hyperglycemia, but not as physiologic responses to metabolic acidosis. The heart rate will increase.

When the patient is diagnosed with muscular dystrophy, what information should the nurse include in the teaching about this disorder? A. Prolonged bed rest will be used to decrease fatigue. B. An orthotic jacket will limit mobility and may contribute to deformity. C. Continuous positive airway pressure will be used to facilitate sleeping. D. Remain active to prevent skin breakdown and respiratory complications.

D. Remain active to prevent skin breakdown and respiratory complications. With muscular dystrophy, it is important for the patient to remain active for as long as possible. Prolonged bed rest should be avoided because immobility leads to further muscle wasting. An orthotic jacket may be used to provide stability and prevent further deformity. Continuous positive airway pressure (CPAP) is used as respiratory function decreases, before mechanical ventilation is needed to sustain respiratory function.

What is the primary treatment for a primary neoplasm on the spine? A. High-dose oral corticosteroids B. Methylprednisolone IV C. Chemotherapy D. Surgery

D. Surgery Treatment for most spinal cord tumors is surgical removal. Because autodestruction does not occur, recovery without residual problems is possible after the physical compression is relieved.

A 54-year-old patient is about to have a bone scan. In teaching the patient about this procedure, the nurse should include what information? A. Two additional follow-up scans will be required. B. There will be only mild pain associated with the procedure. C. The procedure takes approximately 15 to 30 minutes to complete. D. The patient will be asked to drink increased fluids after the procedure.

D. The patient will be asked to drink increased fluids after the procedure. Patients are asked to drink increased fluids after a bone scan to aid in excretion of the radioisotope, if not contraindicated by another condition. No follow-up scans and no pain are associated with bone scans that take 1 hour of lying supine.

You are caring for a patient admitted 1 week earlier with an acute spinal cord injury. Which assessment finding alerts you to the presence of autonomic dysreflexia? A. Tachycardia B. Hypotension C. Hot, dry skin D. Throbbing headache

D. Throbbing headache Autonomic dysreflexia is related to reflex stimulation of the sympathetic nervous system, which is reflected by hypertension, bradycardia, throbbing headache, and diaphoresis.

What is a predominant causative trigger for the painful episodes in trigeminal neuralgia? A. Staphylococcus aureus cellulitis B. Misaligned bite of molar teeth C. Direct trauma to transmandibular joint D. Touching along the lower jaw

D. Touching along the lower jaw Touch and tickle predominate as causative triggers. Others include chewing, tooth brushing, hot or cold blast of air on the face, washing the face, yawning, or talking. The other options are not significant causative factors.

The patient arrives in the emergency department from a motor vehicle accident, during which the car ran into a tree. The patient was not wearing a seat belt, and the windshield is shattered. What action is most important for you to do? A. Determine if the patient lost consciousness. B. Assess the Glasgow Coma Scale (GCS) score. C. Obtain a set of vital signs. D. Use a logroll technique when moving the patient.

D. Use a logroll technique when moving the patient. When the head hits the windshield with enough force to shatter it, you must assume neck or cervical spine trauma occurred and you need to maintain spinal precautions. This includes moving the patient in alignment as a unit or using a logroll technique during transfers. The other options are important and are done after spinal precautions are applied.

What is the most common cause of Guillain-Barré syndrome? A. Parasite infestation causing demyelinization. B. Brain neuron damage from plaques. C. Systemic sepsis from a bacterial infection. D. Viral infection or immunization.

D. Viral infection or immunization. The syndrome is often preceded by immune system stimulation by a viral infection, trauma, surgery, viral immunization, or human immunodeficiency virus (HIV). The other options are not related to Guillain-Barré syndrome.

A patient is admitted to the burn center with burns of his head and neck, chest and back, and left arm and hand following an explosion and a fire in his garage. On admission to the unit, the nurse auscultates wheezes through the lung fields. On reassessment, the nurse notes that the wheezes are gone and the breath sounds are greatly diminished. Which of the following actions is most appropriate for the nurse to take next? A. place the patient in a high Fowler's position B. encourage the patient to cough and auscultate the lungs again C. document the results and continue to monitor the patient's progress D. anticipate the need for endotracheal intubation and notify the physician

D. anticipate the need for endotracheal intubation and notify the physician

A patient with a respiratory condition asks "how does air get into my lungs?". The nurse bases her answer on her knowledge that air moves into the lungs because of: A. contraction of the accessory abdominal muscles. B. increased carbon dioxide and decreased oxygen in the blood. C. stimulation of the respiratory muscles by the chemoreceptors. D. decrease in intrathoracic pressure relative to pressure at the airway.

D. decrease in intrathoracic pressure relative to pressure at the airway. During inspiration, the diaphragm contracts, increasing intrathoracic volume and pushing the abdominal contents downward. At the same time, the external intercostal muscles and scalene muscles contract, increasing the lateral and anteroposterior dimension of the chest. This causes the size of the thoracic cavity to increase and intrathoracic pressure to decrease, which enables air to enter the lungs.

In reviewing medication instructions with a patient being discharged on antihypertensive medications, which statement would be most appropriate for the nurse to make when discussing guanethidine (Ismelin)? A: "A fast heart rate is a side effect to watch for while taking guanethidine." B: "Stop the drug and notify your doctor if you experience any nausea or vomiting." C: "Because this drug may affect the lungs in large doses, it may also help your breathing." D: "Make position changes slowly, especially when rising from lying down to a standing position."

D: "Make position changes slowly, especially when rising from lying down to a standing position." Guanethidine is a peripheral-acting α-adrenergic antagonist and can cause marked orthostatic hypotension. For this reason, the patient should be instructed to rise slowly, especially when moving from a recumbent to a standing position. Support stockings may also be helpful. Tachycardia or lung effects are not evident with guanethidine.

The health care provider orders lactulose for a patient with hepatic encephalopathy. The nurse will monitor for effectiveness of this medication for this patient by assessing what? Relief of constipation Relief of abdominal pain Decreased liver enzymes Decreased ammonia levels

Decreased ammonia levels Hepatic encephalopathy is a complication of liver disease and is associated with elevated serum ammonia levels. Lactulose traps ammonia in the intestinal tract. Its laxative effect then expels the ammonia from the colon, resulting in decreased serum ammonia levels and correction of hepatic encephalopathy.

A 19-year-old patient reports to the clinic nurse the following symptoms: a ring-like itchy rash on the upper leg, low-grade fever, nausea, and joint pain for the past 3 weeks. What question is important for the nurse to ask the patient?

Have you had a tick bite recently?" Symptoms are consistent with Lyme disease caused by the organism Borrelia burgdorferi, which is transmitted by a tick bite.

The condition of a patient who has cirrhosis of the liver has deteriorated. Which diagnostic study would help determine if the patient has developed liver cancer? Serum α-fetoprotein level Ventilation/perfusion scan Hepatic structure ultrasound Abdominal girth measurement

Hepatic structure ultrasound Hepatic structure ultrasound, CT, and MRI are used to screen and diagnose liver cancer. Serum α-fetoprotein level may be elevated with liver cancer or other liver problems. Ventilation/perfusion scans do not diagnose liver cancer. Abdominal girth measurement would not differentiate between cirrhosis and liver cancer.

A patient who has hepatitis B surface antigen (HBsAg) in the serum is being discharged with pain medication after knee surgery. Which medication order should the nurse question because it is most likely to cause hepatic complications? Tramadol (Ultram) Hydromorphone (Dilaudid) Oxycodone with aspirin (Percodan) Hydrocodone with acetaminophen (Vicodin)

Hydrocodone with acetaminophen (Vicodin) The analgesic with acetaminophen should be questioned because this patient is a chronic carrier of hepatitis B and is likely to have impaired liver function. Acetaminophen is not suitable for this patient because it is converted to a toxic metabolite in the liver after absorption, increasing the risk of hepatocellular damage.

The nurse teaches a 50-year-old woman with chronic kidney disease several interventions to reduce pruritus associated with dry skin and uremia. Which statement, if made by the patient to the nurse, indicates further teaching is required?

I can rub my skin instead of scratching. Any activity that causes vasodilation, such as rubbing or bathing and showering in hot water should be avoided as vasodilation leads to increased itching. Menthol in skin products provides a sensation that may distract the patient from the sensation of itchiness. Applying lotion right after bathing helps retain moisture in the skin.

The nurse determines that a patient with a diagnosis of which disorder is most at risk for spreading the disease?

Impetigo on the face

When planning care for a patient with cirrhosis, the nurse will give highest priority to which nursing diagnosis? Impaired skin integrity related to edema, ascites, and pruritus Imbalanced nutrition: less than body requirements related to anorexia Excess fluid volume related to portal hypertension and hyperaldosteronism Ineffective breathing pattern related to pressure on diaphragm and reduced lung volume

Ineffective breathing pattern related to pressure on diaphragm and reduced lung volume Although all of these nursing diagnoses are appropriate and important in the care of a patient with cirrhosis, airway and breathing are always the highest priorities.

The patient with sudden pain in the left upper quadrant radiating to the back and vomiting was diagnosed with acute pancreatitis. What intervention(s) should the nurse expect to include in the patient's plan of care? Immediately start enteral feeding to prevent malnutrition. Insert an NG and maintain NPO status to allow pancreas to rest. Initiate early prophylactic antibiotic therapy to prevent infection. Administer acetaminophen (Tylenol) every 4 hours for pain relief.

Insert an NG and maintain NPO status to allow pancreas to rest. Correct Initial treatment with acute pancreatitis will include an NG tube if there is vomiting and being NPO to decrease pancreatic enzyme stimulation and allow the pancreas to rest and heal. Fluid will be administered to treat or prevent shock. The pain will be treated with IV morphine because of the NPO status. Enteral feedings will only be used for the patient with severe acute pancreatitis in whom oral intake is not resumed. Antibiotic therapy is only needed with acute necrotizing pancreatitis and signs of infection.

The nurse is caring for a 55-year-old man patient with acute pancreatitis resulting from gallstones. Which clinical manifestation would the nurse expect the patient to exhibit? Hematochezia Left upper abdominal pain Ascites and peripheral edema Temperature over 102o F (38.9o C)

Left upper abdominal pain Abdominal pain (usually in the left upper quadrant) is the predominant manifestation of acute pancreatitis. Other manifestations of acute pancreatitis include nausea and vomiting, low-grade fever, leukocytosis, hypotension, tachycardia, and jaundice. Abdominal tenderness with muscle guarding is common. Bowel sounds may be decreased or absent. Ileus may occur and causes marked abdominal distention. Areas of cyanosis or greenish to yellow-brown discoloration of the abdominal wall may occur. Other areas of ecchymoses are the flanks (Grey Turner's spots or sign, a bluish flank discoloration) and the periumbilical area (Cullen's sign, a bluish periumbilical discoloration).

patient with type 2 diabetes and cirrhosis asks the nurse if it would be okay to take silymarin (milk thistle) to help minimize liver damage. The nurse responds based on what knowledge? Milk thistle may affect liver enzymes and thus alter drug metabolism. Milk thistle is generally safe in recommended doses for up to 10 years. There is unclear scientific evidence for the use of milk thistle in treating cirrhosis. Milk thistle may elevate the serum glucose levels and is thus contraindicated in diabetes.

Milk thistle may affect liver enzymes and thus alter drug metabolism. There is good scientific evidence that there is no real benefit from using milk thistle to protect the liver cells from toxic damage in the treatment of cirrhosis. Milk thistle does affect liver enzymes and thus could alter drug metabolism. Therefore patients will need to be monitored for drug interactions. It is noted to be safe for up to 6 years, not 10 years, and it may lower, not elevate, blood glucose levels.

A frail 74-year-old man with recent severe weight loss is instructed to eat a high-protein, high-calorie diet at home. If the man likes all of the items below, which would be the most appropriate for the nurse to suggest?

Oatmeal, butter, and cream Oatmeal, butter, and cream are examples of food items that would be appropriate to include for a patient on a high-protein, high-calorie diet.

A patient who is unable to swallow because of progressive amyotrophic lateral sclerosis is prescribed enteral nutrition through a newly placed gastrostomy tube. Which task is appropriate for the nurse to delegate to unlicensed assistive personnel (UAP)?

Position the patient with a 45-degree head of bed elevation Rationale: Unlicensed assistive personnel (UAP) may position the patient receiving enteral feedings with the head of bed elevated. A licensed practical nurse/licensed vocational nurse (LPN/LVN) or an RN could perform the other activities. Irrigate the tube between feedings. • Provide wound care at the gastrostomy site. • Administer prescribed liquid medications through the tube.

5. An African American woman with a history of breast cancer has panhypopituitarism from radiation therapy for primary pituitary tumors. Which medications should the nurse teach her about needing for the rest of her life (select all that apply)? a. Cortisol b. Vasopressin c. Sex hormones d. Levothyroxine (Synthyroid) e. Growth hormone (somatropin [Omnitrope]) f. Dopamine agonists (bromocriptine [Parlodel])

5. a, b, d, e. With panhypopituitarism, lifetime hormone replacement is needed for cortisol, vasopressin, thyroid, and GH. Sex hormones will not be replaced because of the patient's history of breast cancer. Dopamine agonists will not be used because they reduce secretion of GH, which has already been achieved with the radiation.

5. Which nutrients are essential for red blood cell production (select all that apply)? a. Iron d. Vitamin D b. Folic acid c. Vitamin C e. Vitamin B12 f. Carbohydrates

5. a, b, e. Although all of the listed nutrients are helpful, iron, folic acid, and cobalamin (vitamin B12) are essential for erythropoiesis.

5. During assessment of a patient with dementia, the nurse determines that the condition is potentially reversible when finding out what about the patient? a. Has long-standing abuse of alcohol b. Has a history of Parkinson's disease c. Recently developed symptoms of hypothyroidism d. Was infected with human immunodeficiency virus (HIV) 10 years ago

5. c. Hypothyroidism can cause dementia but it is a treatable condition if it has not been long standing. The other conditions are causes of irreversible dementia.

The patient works on a computer 8 hours each day. What kind of repetitive strain injury would be expected in this patient? a. Meniscus injury b. Rotator cuff injury c. Radial-ulnar fracture d. Carpal tunnel syndrome

5. d. Carpal tunnel syndrome would be expected related to the continuous wrist movements. Injuries of the menisci, which are fibrocartilage in the knee, are common with athletes. Radial-ulnar fractures are seen with great force such as a car accident or a fall. Rotator cuff injuries occur with sudden adduction forces applied to the cuff while the arm is held in abduction. They are commonly seen with repetitive overhead motions.

6. Number the components of normal hemostasis in the order of occurrence, beginning with 1 for the first component and ending with 4 for the last component. a. Lysis of clot b. Vascular response c. Plasma clotting factors d. Platelet plug formation

6. a. 4; b. 1; c. 3; d. 2

6. The patient is diagnosed with syndrome of inappropriate antidiuretic hormone (SIADH). What manifestation should the nurse expect to find? a. Decreased body weight b. Decreased urinary output c. Increased plasma osmolality d. Increased serum sodium levels

6. b. With increased antidiuretic hormone (ADH), the permeability of the renal distal tubules is increased, so water is reabsorbed into circulation. Decreased output of concentrated urine with increased urine osmolality and specific gravity occur. In addition, fluid retention with weight gain, serum hypoosmolality, dilutional hyponatremia, and hypochloremia occur.

6. The athlete comes to the clinic with bursitis. What does the nurse know happens to the tissue to cause pain when bursitis occurs? a. Tearing of a ligament b. Stretching of muscle and fascia sheath c. Inflammation of synovial membrane sac at friction sites d. Incomplete separation of articular surfaces of joint caused by ligament injury

6. c. Bursitis is inflammation of synovial membrane sac at friction sites. Tearing of a ligament is a sprain. Stretching of muscle and fascia sheath is a strain. Incomplete separation of articular surfaces of joints caused by ligament injury is subluxation.

6. The husband of a patient is complaining that his wife's memory has been decreasing lately. When asked for examples of her memory loss, the husband says that she is forgetting the neighbors' names and forgot their granddaughter's birthday. What kind of loss does the nurse recognize this to be? a. Delirium b. Memory loss in AD c. Normal forgetfulness d. Memory loss in mild cognitive impairment

6. d. In mild cognitive impairment people frequently forget people's names and begin to forget important events. Delirium changes usually occur abruptly. In Alzheimer's disease the patient may not remember knowing a person and loses the sense of time and which day it is. Normal forgetfulness includes momentarily forgetting names and occasionally forgetting to run an errand.

7. Which component of normal hemostasis involves the processes of protein C and protein S and plasminogen? a. Lysis of clot b. Vascular response c. Plasma clotting factors d. Platelet plug formation

7. a. Protein C and protein S are examples of anticoagulants that are involved in the lysis of clots. Fibrinolysis is also achieved by thrombin-activating conversion of plasminogen to plasmin, which attacks fibrin or fibrinogen and splits it into smaller elements known as fibrin split products (FSPs) or fibrin degradation products (FDPs).

7. During care of the patient with SIADH, what should the nurse do? a. Monitor neurologic status at least every 2 hours. b. Teach the patient receiving treatment with diuretics to restrict sodium intake. c. Keep the head of the bed elevated to prevent antidiuretic hormone (ADH) release. d. Notify the health care provider if the patient's blood pressure decreases more than 20 mm Hg from baseline.

7. a. The patient with syndrome of inappropriate antidiuretic hormone (SIADH) has marked dilutional hyponatremia and should be monitored for decreased neurologic function and seizures every 2 hours. Sodium intake is supplemented because of the hyponatremia and sodium loss caused by diuretics. ADH release is reduced by keeping the head of the bed flat to increase left atrial filling pressure. A reduction in blood pressure (BP) indicates a reduction in total fluid volume and is an expected outcome of treatment.

7. Application of RICE (rest, ice, compression, and elevation) is indicated for initial management of which type of injury? a. Muscle spasms b. Sprains and strains c. Repetitive strain injury d. Dislocations and subluxations

7. b. Application of cold, compression, and elevation are indicated to prevent edema resulting from sprain and some strain injuries. Muscle spasms are usually treated with heat applications and massage and repetitive strain injuries require cessation of the precipitating activity and physical therapy. Dislocations or subluxations require immediate reduction and immobilization to prevent vascular impairment and bone cell death.

7. The wife of a patient who is manifesting deterioration in memory asks the nurse whether her husband has AD. The nurse explains that a diagnosis of AD is usually made when what happens? a. A urine test indicates elevated levels of isoprostanes b. All other possible causes of dementia have been eliminated c. Blood analysis reveals increased amounts of β-amyloid protein d. A computed tomography (CT) scan of the brain indicates brain atrophy

7. b. The only definitive diagnosis of AD can be made on examination of brain tissue during an autopsy but a clinical diagnosis is made when all other possible causes of dementia have been eliminated. Patients with AD may have β-amyloid proteins in the blood, brain atrophy, or isoprostanes in the urine but these findings are not exclusive to those with AD.

What should be included in the management during the first 48 hours after an acute soft tissue injury of the ankle (select all that apply)? a. Use of elastic wrap b. Initial immobilization and rest c. Elevation of ankle above the heart d. Alternating the use of heat and cold e. Administration of antiinflammatory drugs

8. a, b, c, e. Consider the principle of RICE. Rest: movement should be restricted. Ice: cold should be used to promote vasoconstriction and to reduce edema. C: compression helps to decrease swelling. E: elevate the extremity above the level of the heart. Mild nonsteroidal antiinflammatory drugs (NSAIDs) may be needed to manage pain. Warm, moist compresses may be used after 48 hours for 20 to 30 minutes at a time to reduce swelling and provide comfort.

8. A patient who was in a car accident had abdominal trauma. Which organs may be damaged and contribute to altered function of the hematologic system (select all that apply)? a. Liver b. Spleen c. Stomach d. Gallbladder e. Lymph nodes

8. a, b, e. The abdominal organs that are primarily involved in hematologic function are the liver, spleen, and lymph nodes. The liver filters the blood, produces procoagulants, and stores iron. The spleen removes old and defective erythrocytes and filters iron for reuse. The lymph nodes filter pathogens and foreign particles from lymphatic circulation.

8. A patient with SIADH is treated with water restriction. What does the patient experience when the nurse determines that treatment has been effective? a. Increased urine output, decreased serum sodium, and increased urine specific gravity b. Increased urine output, increased serum sodium, and decreased urine specific gravity c. Decreased urine output, increased serum sodium, and decreased urine specific gravity d. Decreased urine output, decreased serum sodium, and increased urine specific gravity

8. b. The patient with SIADH has water retention with hyponatremia, decreased urine output, and concentrated urine with high specific gravity. Improvement in the patient's condition is reflected by increased urine output, normalization of serum sodium, and more water in the urine, thus decreasing the specific gravity.

9. The patient had a fracture. At 3 weeks to 6 months there is clinical union, and this is the first stage of healing that is sufficient to prevent movement of the fracture site when the bones are gently stressed. How is this stage of fracture healing documented? a. Ossification b. Remodeling c. Consolidation d. Callus formation

9. a. Ossification is the stage of fracture healing when there is clinical union and enough strength to prevent movement at the fracture site. Remodeling is the normal function of the bone. Consolidation is when the distance between bone fragments eventually closes and radiologic union first occurs. The callus formation stage appears by the end of the second week of injury when minerals and new bone matrix are deposited in the osteoid that is produced in the granulation tissue stage.

9. What is one focus of collaborative care of patients with AD? a. Replacement of deficient acetylcholine in the brain b. Drug therapy for cognitive problems and undesirable behaviors c. The use of memory-enhancing techniques to delay disease progression d. Prevention of other chronic diseases that hasten the progression of AD

9. b. Because there is no cure for AD, collaborative management is aimed at controlling the decline in cognition, controlling the undesirable manifestations that the patient may exhibit, and providing support for the family caregiver. Anticholinesterase agents help to increase acetylcholine (ACh) in the brain but a variety of other drugs are also used to control behavior. Memoryenhancing techniques have little or no effect in patients with AD, especially as the disease progresses. Patients with AD have limited ability to communicate health symptoms and problems, leading to a lack of professional attention for acute and other chronic illnesses.

The patient with diabetes insipidus is brought to the emergency department with confusion and dehydration after excretion of a large volume of urine today even though several liters of fluid were drunk. What is a diagnostic test that the nurse should expect to be done to help make a diagnosis? a. Blood glucose b. Serum sodium level c. Urine specific gravity d. Computed tomography (CT) of the head

9. c. Patients with diabetes insipidus (DI) excrete large amounts of urine with a specific gravity of less than 1.005. Blood glucose would be tested to diagnose diabetes mellitus. The serum sodium level is expected to be low with DI but is not diagnostic. To diagnose central DI a water deprivation test is required. Then a CT of the head may be done to determine the cause. Nephrogenic DI is differentiated from central DI with determination of the level of ADH after an analog of ADH is given.

A patient arrives at the ambulatory surgery center for a scheduled laparoscopy procedure in outpatient surgery. Which information is of most concern to the nurse? a. The patient is planning to drive home after surgery. b. The patient had a sip of water 4 hours before arriving. c. The patient's insurance does not cover outpatient surgery. d. The patient has not had surgery using general anesthesia before.

A

A patient scheduled for an elective hysterectomy tells the nurse, "I am afraid that I will die in surgery like my mother did!" Which response by the nurse is most appropriate? a. "Tell me more about what happened to your mother." b. "You will receive medications to reduce your anxiety." c. "You should talk to the doctor again about the surgery." d. "Surgical techniques have improved a lot in recent years."

A

A patient who is scheduled for a therapeutic abortion tells the nurse, "Having an abortion is not right." Which functional health pattern should the nurse further assess? a. Value-belief b. Cognitive-perceptual c. Sexuality-reproductive d. Coping-stress tolerance

A

The surgical unit nurse has just received a patient with a history of smoking from the postanesthesia care unit. Which action is most important at this time? a. Auscultate for adventitious breath sounds. b. Obtain the patient's blood pressure and temperature. c. Remind the patient about harmful effects of smoking. d. Ask the health care provider about prescribing a nicotine patch.

A

Which statement by a patient scheduled for surgery is most important to report to the health care provider? a. "I had a heart valve replacement last year." b. "I had bacterial pneumonia 3 months ago." c. "I have knee pain whenever I walk or jog." d. "I have a strong family history of breast cancer."

A

The nurse is teaching about skin cancer prevention at the community center. Which individual is most at risk for developing skin cancer?

A 55-year-old woman with fair skin and red hair who has a family history of skin cancer Risk factors for skin cancer include having fair skin (with red hair) and a family history of skin cancer. Allergies, acrochordons (skin tags), psoriasis, type 2 diabetes mellitus, and chronic kidney disease are not risk factors associated with the development of skin cancer.

The nurse is caring for a group of patients. Which patient is at highest risk for pancreatic cancer? A 38-year-old Hispanic female who is obese and has hyperinsulinemia A 23-year-old who has cystic fibrosis-related pancreatic enzyme insufficiency A 72-year-old African American male who has smoked cigarettes for 50 years A 19-year-old who has a 5-year history of uncontrolled type 1 diabetes mellitus

A 72-year-old African American male who has smoked cigarettes for 50 years Risk factors for pancreatic cancer include chronic pancreatitis, diabetes mellitus, age, cigarette smoking, family history of pancreatic cancer, high-fat diet, and exposure to chemicals such as benzidine. African Americans have a higher incidence of pancreatic cancer than whites. The most firmly established environmental risk factor is cigarette smoking. Smokers are two or three times more likely to develop pancreatic cancer as compared with nonsmokers. The risk is related to duration and number of cigarettes smoked.

An older patient was admitted with a fractured hip after being found on the floor of her home. She was extremely malnourished and started on parenteral nutrition (PN) 3 days ago. Which assessment finding would be of most concern to the nurse?

Serum phosphate level of 1.9 mg/dL Refeeding syndrome can occur if a malnourished patient is started on aggressive nutritional support. Hypophosphatemia (serum phosphate level less than 2.4 mg/dL) is the hallmark of refeeding syndrome and could result in cardiac dysrhythmias, respiratory arrest, and neurologic problems. An increase in the blood glucose level is expected during the first few days after PN is started. The goal is to maintain a glucose range of 110 to 150 mg/dL. An elevated white blood cell count (greater than 11,000/µL) could indicate an infection. Normal serum potassium levels are between 3.5 and 5.0 mEq/L.

The nurse is evaluating the nutritional status of a 55-year-old man who is undergoing radiation treatment for oropharyngeal cancer. Which laboratory test would be the best indicator to determine if the patient has protein-calorie malnutrition?

Serum prealbumin In the absence of an inflammatory condition, the best indicator of protein-calorie malnutrition (PCM) is prealbumin; prealbumin is a protein synthesized by the liver and indicates recent or current nutritional status. Decreased albumin and transferrin levels are other indicators that protein is deficient. C-reactive protein (CRP) is elevated during inflammation and is used to determine if prealbumin, albumin, and transferrin are decreased related to protein deficiency or an inflammatory process. Other indicators of protein deficiency include elevated serum potassium levels, low red blood cell counts and hemoglobin levels, decreased total lymphocyte count, elevated liver enzyme levels (ALT), and decreased levels of both fat-soluble and water-soluble vitamins.

The patient with a history of lung cancer and hepatitis C has developed liver failure and is considering liver transplantation. After the comprehensive evaluation, the nurse knows that which factor discovered may be a contraindication for liver transplantation? Has completed a college education Has been able to stop smoking cigarettes Has well-controlled type 1 diabetes mellitus The chest x-ray showed another lung cancer lesion.

The chest x-ray showed another lung cancer lesion. Correct Contraindications for liver transplant include severe extrahepatic disease, advanced hepatocellular carcinoma or other cancer, ongoing drug and/or alcohol abuse, and the inability to comprehend or comply with the rigorous post-transplant course.

A 67-year-old woman admitted with heart failure is also diagnosed with herpes zoster (shingles) and draining vesicles. Which action, if observed by the nurse, would require additional teaching for that individual?

The dietitian wears a mask when entering the patient's room Rationale: Herpes zoster (shingles) is spread by contact with fluid draining from the vesicles (not by coughing, sneezing, or casual contact). Shingles is not contagious before the vesicles appear or after the vesicles have crusted over. The risk of a person with shingles spreading the virus is low if the rash is covered. Wearing a mask would not prevent the spread of infection. Until the rash develops crusts, the patient should not have contact with an immune compromised person (e.g., a person taking prednisone). Frequent hand washing helps to prevent the spread of varicella zoster virus.

The patient with cirrhosis has an increased abdominal girth from ascites. The nurse should know that this fluid gathers in the abdomen for which reasons (select all that apply)? There is decreased colloid oncotic pressure from the liver's inability to synthesize albumin. Hyperaldosteronism related to damaged hepatocytes increases sodium and fluid retention. Portal hypertension pushes proteins from the blood vessels, causing leaking into the peritoneal cavity. Osmoreceptors in the hypothalamus stimulate thirst, which causes the stimulation to take in fluids orally. Overactivity of the enlarged spleen results in increased removal of blood cells from the circulation, which decreases the vascular pressure.

There is decreased colloid oncotic pressure from the liver's inability to synthesize albumin. Correct Hyperaldosteronism related to damaged hepatocytes increases sodium and fluid retention. Correct Portal hypertension pushes proteins from the blood vessels, causing leaking into the peritoneal cavity. Correct The ascites related to cirrhosis are caused by decreased colloid oncotic pressure from the lack of albumin from liver inability to synthesize it and the portal hypertension that shifts the protein from the blood vessels to the peritoneal cavity, and hyperaldosteronism which increases sodium and fluid retention. The intake of fluids orally and the removal of blood cells by the spleen do not directly contribute to ascites.

When caring for a patient with liver disease, the nurse recognizes the need to prevent bleeding resulting from altered clotting factors and rupture of varices. Which nursing interventions would be appropriate to achieve this outcome (select all that apply)? Use smallest gauge needle possible when giving injections or drawing blood. Teach patient to avoid straining at stool, vigorous blowing of nose, and coughing. Advise patient to use soft-bristle toothbrush and avoid ingestion of irritating food. Apply gentle pressure for the shortest possible time period after performing venipuncture. Instruct patient to avoid aspirin and NSAIDs to prevent hemorrhage when varices are present.

Use smallest gauge needle possible when giving injections or drawing blood. Teach patient to avoid straining at stool, vigorous blowing of nose, and coughing. Advise patient to use soft-bristle toothbrush and avoid ingestion of irritating food. Instruct patient to avoid aspirin and NSAIDs to prevent hemorrhage when varices are present. Using the smallest gauge needle for injections will minimize the risk of bleeding into the tissues. Avoiding straining, nose blowing, and coughing will reduce the risk of hemorrhage at these sites. The use of a soft-bristle toothbrush and avoidance of irritating food will reduce injury to highly vascular mucous membranes. The nurse should apply gentle but prolonged pressure to venipuncture sites to minimize the risk of bleeding. Aspirin and NSAIDs should not be used in patients with liver disease because they interfere with platelet aggregation, thus increasing the risk for bleeding.

When caring for a patient with a biliary obstruction, the nurse will anticipate administering which vitamin supplements (select all that apply)? Vitamin A Vitamin D Vitamin E Vitamin K Vitamin B

Vitamin A Correct Vitamin D Correct Vitamin E Correct Vitamin K Correct Biliary obstruction prevents bile from entering the small intestine and thus prevents the absorption of fat-soluble vitamins. Vitamins A, D, E, and K are all fat-soluble and thus would need to be supplemented in a patient with biliary obstruction.

The nurse cares for a 41-year-old male patient admitted for uncontrolled seizures who is also diagnosed with impetigo on the face and neck. Which action is appropriate for the nurse to take?

Wash hands for 1 to 2 minutes when leaving the room. Impetigo is a bacterial skin infection with group A β-hemolytic streptococci or staphylococci. Meticulous hygiene (including hand washing) is essential to prevent the spread of infection. A particulate mask or a gown would not be necessary to prevent the spread of impetigo. Gloves would not be needed to make a delivery to the room.

When caring for the patient with a traumatic brain injury (TBI), the nurse knows that damage to which endocrine gland can affect the hormone secretion from some of the other endocrine glands?

With a TBI, the anterior pituitary is likely to be damaged. The anterior pituitary gland secrets tropic hormones that control the secretion of hormones by other endocrine glands (the thyroid, adrenal cortex, and reproductive organs). The parathyroids secrete parathyroid hormone that regulates serum calcium level by acting on bone, the kidneys, and indirectly the gastrointestinal tract. The pineal gland secretes melatonin that helps regulate circadian rhythm and reproduction. The thyroid glands secrete thyroxine (T4), triiodothyronine (T3) that regulates the cell processes of cell growth and tissue differentiation, and calcitonin that affects bone tissue to regulate serum calcium and phosphorus levels.

Inflammation and infection of the eye a. are caused by irritants and microorganisms. b. have a higher incidence in sexually active patients. c. are chronic problems that result in a loss of vision. d. are frequently treated with cold compresses and antibiotics.

a Rationale: Inflammation or infection of the eye is caused by external irritants or microorganisms. The nurse teaches the patient appropriate interventions related to the specific disorder. Common interventions include the application of warm, moist compresses and administration of antibiotics.

In planning care for a patient with metastatic liver cancer, the nurse should include interventions that a. focus primarily on symptomatic and comfort measures. b. reassure the patient that chemotherapy offers a good prognosis. c. promote the patient's confidence that surgical excision of the tumor will be successful. d. provide information necessary for the patient to make decisions regarding liver transplantation. (Lewis 1042)

a Rationale: Nursing intervention for a patient with liver cancer focuses on keeping the patient as comfortable as possible. The prognosis for patients with liver cancer is poor. The cancer grows rapidly, and death may occur within 4 to 7 months as a result of hepatic encephalopathy or massive blood loss from gastrointestinal (GI) bleeding.

The patient who has a conductive hearing loss a. hears better in a noisy environment. b. hears sound but does not understand speech. c. often speaks loudly because his or her own voice seems low. d. experiences clearer sound with a hearing aid if the loss is less than 30 dB.

a Rationale: The patient with conductive hearing loss often speaks softly because hearing his or her own voice (which is conducted by bone) seems loud. This patient hears better in a noisy environment. The first step is to identify and treat the cause if possible. If correction of the cause is not possible, a hearing aid may help if the loss is greater than 40 to 50 dB.

Number the following physiologic occurrences in the order they occur in the formation of urine. Begin with 1 for the fist occurrence and number through 6 for the last occurrence in the formation of urine. ____ a. Blood is filtered in the glomerulus. ____ b. Reabsorption of water in the loop of Henle. ____ c. Reabsorption of electrolytes, glucose, amino acids, and small proteins in the tubules. ____ d. Acid-base regulation with conservation of bicarbonate (HCO3-) and secretion of excess H+ in the distal tubule. ____ e. Active reabsorption of chloride (Cl-) ions and passive reabsorption of sodium (Na+) ions in the ascending loop of Henle. ____ f. Ultrafiltrate flows from Bowman's capsule and passess down the tubules without blood cells, platelets, or large plasma proteins.

a, 1; b, 4; c, 3; d, 6; e, 5; f, 2. Rationale: Blood is filtered in the glomerulus and the ultrafiltrate flows from the Bowman's capsule to the tubules for reabsorption of essential materials and secretion of the nonessential ones. In the proximal convoluted tubule, most electrolytes, glucose, amino acids, and small proteins are reabsorbed. Water is conserved in the loop of Henle with chloride and sodium reabsorbed in the ascending loop. The distal convoluted tubules complete final water balance and acid-base balance.

Prevention of AKI is important because of the high mortality rate. Which patients are at increased risk for AKI (select all that apply)? a. An 86-year-old woman scheduled for a cardiac catheterization b. A 48-year-old man with multiple injuries from a motor vehicle accident c. A 32-year-old woman following a C-section delivery for abruptio placentae d. A 64-year-old woman with chronic heart failure admitted with bloody stools e. A 58-year-old man with prostate cancer undergoing preoperative workup for prostatectomy

a, b, c, d, e. High-risk patients include those exposed to nephrotoxic agents and advanced age (a), massive trauma (b), prolonged hypovolemia or hypotension (possibly b and c), obstetric complications (c), cardiac failure (d), preexisting chronic kidney disease, extensive burns, or sepsis. Patients with prostate cancer may have obstruction of the outflow tract, which increases risk of postrenal AKI (e).

What are the characteristics of neutrophils (select all that apply)? a. Also known as "segs" b. Band is immature cell c. First WBC at injury site d. Arises from megakaryocyte e. Increased in individuals with allergies f. 50% to 70% of white blood cells (WBCs)

a, b, c, f. These characteristics are evident with neutrophils. Platelets arise from megakaryocytes and are stored in the spleen. Eosinophils are increased in individuals with allergies and make up 2% to 4% of WBCs.

Which drugs will be used to treat the patient with CKD for mineral and bone disorder (select all that apply)? a. Cinacalcet (Sensipar) b. Sevelamer (Renagel) c. IV glucose and insulin d. Calcium acetate (PhosLo) e. IV 10% calcium gluconate

a, b, d. Cinacalcet (Sensipar), a calcimimetic agent to control secondary hyperparathyroidism; sevelamer (Renagel), a noncalcium phosphate binder; and calcium acetate (PhosLo), a calcium-based phosphate binder are used to treat mineral and bone disorder in CKD. IV glucose and insulin and IV 10% calcium gluconate along with sodium polystyrene sulfonate (Kayexalate) are used to treat the hyperkalemia of CKD.

The patient with CKD is receiving dialysis, and the nurse observes excoriations on the patient's skin. What pathophysiologic changes in CKD can contribute to this finding (select all that apply)? a. Dry skin b. Sensory neuropathy c. Vascular calcifications d. Calcium-phosphate skin deposits e. Uremic crystallization from high BUN

a, b, d. Pruritus is common in patients receiving dialysis. It causes scratching from dry skin, sensory neuropathy, and calcium-phosphate deposition in the skin. Vascular calcifications contribute to cardiovascular disease, not to itching skin. Uremic frost rarely occurs without BUN levels greater than 200 mg/dL, which should not occur in a patient on dialysis; urea crystallizes on the skin and also causes pruritis.

What are common diagnostic studies done for a patient with severe renal colic (select all that apply)? a. CT scan b. Urinalysis c. Cystoscopy d. Ureteroscopy e. Abdominal ultrasound

a, b, e. Testing would include urinalysis to see crystals and look for red blood cells. Abdominal ultrasound and CT scan may also be done.

What accurately describes prostate cancer detection and/or treatment (select all that apply)? a. The symptoms of pelvic or perineal pain, fatigue, and malaise may be present. b. Palpation of the prostate reveals hard and asymmetric enlargement with areas of induration or nodules. c. Orchiectomy is a treatment option for all patients with prostatic cancer except those with stage IV tumors. d. The preferred hormonal therapy for treatment of prostate cancer includes estrogen and androgen receptor blockers. e. Early detection of cancer of the prostate is increased with annual rectal examinations and serum prostatic acid phosphatase (PAP) measurements. f. An annual prostate examination is recommended starting at age 45 for African American men because of the increased mortality rate from prostatic cancer in this population

a, b, f. Pelvic or perineal pain, fatigue, malaise, and a hard asymmetric prostate may be present with prostate cancer. Annual prostate examination is recommended starting at a younger age for African American men because of increased diagnosis and mortality from prostate cancer in this ethnic group. An orchiectomy may be done with prostatectomy or for metastatic stages of prostate cancer. Hormonal treatment includes androgen deprivation therapy, luteinizing hormone-releasing hormone agonists, and androgen receptor blockers. Early detection of prostate cancer is best detected with annual rectal exams and serum PSA. Elevated prostatic acid phosphatase (PAP) will be seen with metastasis, not a new diagnosis.

The nurse performs a detailed assessment of the abdomen of a patient with a possible bowel obstruction, knowing that a manifestation of an obstruction in the large intestine is (select all that apply) a. a largely distended abdomen. b. Diarrhea that is loose or liquid. c. persistent, colicky abdominal pain. d. profuse vomiting that relieves abdominal pain.

a, c

The right atrium myocytes secrete atrial natriuretic peptide (ANP)when there is increased plasma volume. What actions does ANP take to produce a large volume of dilute urine (select all that apply)? a. Inhibits renin b. Increases ADH c. Inhibits angiotensin II action d. Decreases sodium excretion e. Increases aldosterone secretion

a, c Rationale: Atrial natriuretic peptide (ANP) responds to increased atrial distention by increasing sodium excretion and inhibiting renin, ADH, and angiotensin action. Aldosterone secretion is also suppressed. ANP also causes afferent arteriole relaxation that increases the glomerular filtration rate (GFR).

Age-related changes in the auditory system commonly include (select all that apply) a. drier cerumen. b. tinnitus in both ears. c. auditory nerve degeneration. d. atrophy of the tympanic membrane. e. greater ability to hear high-pitched sounds.

a, c, d Rationale: Age-related changes include increased production of drier cerumen, atrophic changes of tympanic membrane, and neuron degeneration in auditory nerve and central pathways.

3. Which cells are classified as granulocytes (select all that apply)? a. Basophil b. Monocyte c. Eosinophil d. Neutrophil e. Lymphocyte

a, c, d. Basophils, eosinophils, and neutrophils are the granulocytic leukocytes. Lymphocytes are the agranular leukocytes that form the basis of the cellular and humoral immune responses. Monocytes are agranulocytes that are potent phagocytic cells.

23. What emergency considerations must be included with facial fractures (select all that apply)? a. Airway patency b. Oral examination c. Cervical spine injury d. Cranial nerve assessment e. Immobilization of the jaw

a, c. Airway patency and cervical spinal cord injury are the emergency considerations with facial fractures. Oral examination and cranial nerve assessment will be done after the patient is stabilized. Immobilization of the jaw is done surgically for a mandibular fracture.

What should be included in the postoperative teaching of the patient who has undergone cataract surgery (select all that apply)? a. Eye discomfort is often relieved with mild analgesics. b. A decline in visual acuity is common for the first week. c. Stay on bed rest and limit activity for the first few days. d. Notify surgeon if an increase in redness or drainage occurs. e. Nighttime eye shielding and activity restrictions are essential to prevent eyestrain.

a, d Rationale: After cataract surgery, the nurse should teach the patient and caregiver about the following topics before discharge: topical antibiotics; topical corticosteroids or other antiinflammatory agents; mild analgesia, if necessary; eye shield if used (usually worn overnight and removed during the first postoperative visit); and activity restrictions as preferred by the patient's surgeon (activities that increase the intraocular pressure, such as bending or stooping, coughing, or lifting, may be restricted). Complications that should be reported include intense pain (which may indicate hemorrhage), infection, increased intraocular pressure, increased or purulent drainage, increased redness, and any decrease in visual acuity.

Before injecting fluorescein for angiography, it is important for the nurse to (select all that apply) a. obtain an emesis basin. b. ask if the patient is fatigued. c. administer a topical anesthetic. d. inform patient that skin may turn yellow. e. assess for allergies to iodine-based contrast media.

a, d Rationale: Fluorescein dye sometimes causes nausea or vomiting, and the dye may cause a transient yellow-orange discoloration of urine and skin. Extravasation of the dye causes tissue toxicity. Systemic allergic reactions to the dye are rare, but the nurse should be familiar with emergency equipment and procedures.

Which patient behaviors would the nurse promote for healthy eyes and ears (select all that apply)? a. Wearing protective sunglasses when bicycling b. Supplemental intake of B vitamins and magnesium c. Playing amplified music at 75% of maximum volume d. Patient notifying the health care provider of tinnitus while on antibiotics e. A woman avoiding pregnancy for 4 weeks after receiving measles, mumps, rubella (MMR) immunization

a, d Rationale: Wearing sunglasses may contribute to the prevention of cataract development and age-related macular degeneration. Protective eyewear during sports activities reduces the risk of eye injuries. Antibiotics, salicylates, diuretics, and antineoplastic drugs are commonly associated with ototoxicity.

What manifestations of cognitive impairment are primarily characteristic of delirium (select all that apply)? a. Reduced awareness b. Impaired judgments c. Words difficult to find d. Sleep/wake cycle reversed e. Distorted thinking and perception f. Insidious onset with prolonged duration

a, d, e. Manifestations of delirium include cognitive impairment with reduced awareness, reversed sleep/wake cycle, and distorted thinking and perception. The other options are characteristic of dementia.

Nursing management of the patient with acute pancreatitis includes (select all that apply) a. checking for signs of hypocalcemia. b. providing a diet low in carbohydrates. c. giving insulin based on a sliding scale. d. observing stools for signs of steatorrhea. e. monitoring for infection, particularly respiratory tract infection. (Lewis 1042)

a, e Rationale: During the acute phase, it is important to monitor vital signs. Hemodynamic stability may be compromised by hypotension, fever, and tachypnea. Intravenous fluids are ordered, and the response to therapy is monitored. Fluid and electrolyte balances are closely monitored. Frequent vomiting, along with gastric suction, may result in decreased levels of chloride, sodium, and potassium. Because hypocalcemia can occur in acute pancreatitis, the nurse should observe for symptoms of tetany, such as jerking, irritability, and muscular twitching. Numbness or tingling around the lips and in the fingers is an early indicator of hypocalcemia. The patient should be assessed for Chvostek's sign or Trousseau's sign. A patient with acute pancreatitis should be observed for fever and other manifestations of infection. Respiratory infections are common because the retroperitoneal fluid raises the diaphragm, which causes the patient to take shallow, guarded abdominal breaths.

Which important functions of regulation of water balance and acid-base balance occur in the distal convoluted tubules of the nephron (select all that apply)? a. Secretion of H+ into filtrate b. Reabsorption of water without ADH c. Reabsorption of Na+ in exchange for K+ d. Reabsorption of glucose and amino acids e. Reabsorption of water under ADH influence f. Reabsorption of Ca+2 under parathormone influence

a,c,e,f Rationale: The distal tubules regulate water and acid-base balance by reabsorption of water under antidiuretic hormone (ADH) influence, secreting H and reabsorbing bicarbonate, reabsorption of Na+ in exchange for K+, and reabsorption of Ca+2 with the influence of parathormone. The reabsorption of water without ADH occurs in the proximal convoluted tubule and the descending loop of Henle. The reabsorption of glucose and amino acids occurs in the proximal convoluted tubule. Active reabsorption of Cl- and passive reabsorption of Na+ occurs in the ascending loop of Henle.

Always assess the patient with an ophthalmic problem for a. visual acuity. b. pupillary reactions. c. intraocular pressure. d. confrontation visual fields.

a. Rationale: Always assess and record the patient's visual acuity for medical and legal reasons.

Number the following in the order of the phases of exchange in PD. Begin with 1 and end with 3. ____ a. Drain ____ b. Dwell ____ c. Inflow

a. 3; b. 2; c. 1

Number in sequence the following ascending pathologic changes that occur in the urinary tract in the presence of a bladder outlet obstruction. a. Hydronephrosis b. Reflux of urine into ureter c. Bladder detrusor muscle hypertrophy d. Ureteral dilation e. Renal atrophy f. Vesicoureteral reflux g. Large residual urine in bladder h. Chronic pyelonephritis

a. 6; b. 3; c. 1; d. 4; e. 8; f. 5; g. 2; h. 7

3. A patient with acromegaly is treated with a transsphenoidal hypophysectomy. What should the nurse do postoperatively? a. Ensure that any clear nasal drainage is tested for glucose. b. Maintain the patient flat in bed to prevent cerebrospinal fluid (CSF) leakage. c. Assist the patient with toothbrushing every 4 hours to keep the surgical area clean. d. Encourage deep breathing, coughing, and turning to prevent respiratory complications.

a. A transsphenoidal hypophysectomy involves entry into the sella turcica through an incision in the upper lip and gingiva into the floor of the nose and the sphenoid sinuses. Postoperative clear nasal drainage with glucose content indicates cerebrospinal fluid (CSF) leakage from an open connection to the brain, putting the patient at risk for meningitis. After surgery, the patient is positioned with the head elevated to avoid pressure on the sella turcica. Coughing and straining are avoided to prevent increased intracranial pressure and CSF leakage. Although mouth care is required every 4 hours, toothbrushing should not be performed because injury to the suture line may occur.

Which urine specific gravity value would indicate to the nurse that the patient is receiving excessive IV fluid therapy? a. 1.002 b. 1.010 c. 1.025 d. 1.030

a. A urine specific gravity of 1.002 is low, indicating dilute urine and the excretion of excess fluid. Fluid overload, diuretics, or lack of ADH can cause dilute urine. Normal urine specific gravity is 1.003 to 1.030. A high urine specific gravity indicates concentrated urine that would be seen in dehydration.

Which classification of urinary tract infection (UTI) is described as infection of the renal parenchyma, renal pelvis, and ureters? a. Upper UTI b. Lower UTI c. Complicated UTI d. Uncomplicated UTI

a. An upper urinary tract infection (UTI) affects the renal parenchyma, renal pelvis, and ureters. A lower UTI is an infection of the bladder and/or urethra. A complicated UTI exists in the presence of obstruction, stones, or preexisting diseases. An uncomplicated UTI occurs in an otherwise normal urinary tract.

A patient is seeking medical intervention for erectile dysfunction. Why should he be thoroughly evaluated? a. Treatment of erectile dysfunction is based on the cause of the problem. b. Psychologic counseling can reverse the problem in 80% to 90% of the cases. c. New invasive and experimental techniques currently used have unknown risks. d. Most treatments for erectile dysfunction are contraindicated in patients with systemic diseases.

a. Before treatment for erectile dysfunction is initiated, the cause must be determined so that appropriate treatment can be planned. Only a small percentage of erectile dysfunction is caused by psychologic factors. In the case of the 80% to 90% of erectile dysfunction that is of physiologic causes, interventions are directed at correcting or eliminating the cause or restoring function by medical means. New invasive or experimental treatments are not widely used and should be limited to research centers and patients with systemic diseases can be treated medically if the cause cannot be eliminated.

When obtaining a nursing history from a patient with cancer of the urinary system, what does the nurse recognize as a risk factor associated with both kidney cancer and bladder cancer? a. Smoking b. Family history of cancer c. Chronic use of phenacetin d. Chronic, recurrent nephrolithiasis

a. Both cancer of the kidney and cancer of the bladder are associated with smoking. A family history of renal cancer is a risk factor for kidney cancer and cancer of the bladder has been associated with the use of phenacetin-containing analgesics and recurrent upper UTIs.

Besides being mixed with struvite or oxalate stones, what characteristic is associated with calcium phosphate calculi? a. Associated with alkaline urine b. Genetic autosomal recessive defect c. Three times as common in women as in men d. Defective gastrointestinal (GI) and kidney absorption

a. Calcium phosphate calculi are typically mixed with struvite or oxalate stones and related to alkaline urine. Cystine calculi are associated with a genetic autosomal recessive defect and defective GI and kidney absorption of cystine. Struvite calculi are three to four times more common in women than in men.

Priority Decision: A patient on a medical unit has a potassium level of 6.8 mEq/L. What is the priority action that the nurse should take? a. Place the patient on a cardiac monitor. b. Check the patient's blood pressure (BP). c. Instruct the patient to avoid high-potassium foods. d. Call the lab and request a redraw of the lab to verify results.

a. Dysrhythmias may occur with an elevated potassium level and are potentially lethal. Monitor the rhythm while contacting the physician or calling the rapid response team. Vital signs should be checked. Depending on the patient's history and cause of increased potassium, instruct the patient about dietary sources of potassium; however, this would not help at this point. The nurse may want to recheck the value but until then the heart rhythm needs to be monitored.

In which clinical situation would the increased release of erythropoietin be expected? a. Hypoxemia b. Hypotension c. Hyperkalemia d. Fluid overload

a. Erythropoietin is released when the oxygen tension of the renal blood supply is low and stimulates production of red blood cells in the bone marrow. Hypotension causes activation of the renin-angiotensin-aldosterone system, as well as release of ADH. Hyperkalemia stimulates the release of aldosterone from the adrenal cortex and fluid overload does not directly stimulate factors affecting the erythropoietin release by the kidney.

Which complication of chronic kidney disease is treated with erythropoietin (EPO)? a. Anemia b. Hypertension c. Hyperkalemia d. Mineral and bone disorder

a. Erythropoietin is used to treat anemia, as it stimulates the bone marrow to produce red blood cells.

What is the effect of finasteride (Proscar) in the treatment of BPH? a. A reduction in the size of the prostate gland b. Relaxation of the smooth muscle of the urethra c. Increased bladder tone that promotes bladder emptying d. Relaxation of the bladder detrusor muscle promoting urine flow

a. Finasteride results in suppression of androgen formation by inhibiting the formation of the testosterone metabolite dihydroxytestosterone, the principal prostatic androgen, and results in a decrease in the size of the prostate gland. α-Adrenergic blockers are used to cause smooth muscle relaxation in the prostate that improves urine flow. Drugs affecting bladder tone are not indicated.

Priority Decision: During the immediate postoperative care of a recipient of a kidney transplant, what should the nurse expect to do? a. Regulate fluid intake hourly based on urine output. b. Monitor urine-tinged drainage on abdominal dressing. c. Medicate the patient frequently for incisional flank pain. d. Remove the urinary catheter to evaluate the ureteral implant.

a. Fluid and electrolyte balance is critical in the transplant recipient patient, especially because diuresis often begins soon after surgery. Fluid replacement is adjusted hourly based on kidney function and urine output. Urine-tinged drainage on the abdominal dressing may indicate leakage from the ureter implanted into the bladder and the health care provider should be notified. The donor patient may have a flank or laparoscopic incision(s) where the kidney was removed. The recipient has an abdominal incision where the kidney was placed in the iliac fossa. The urinary catheter is usually used for 2 to 3 days to monitor urine output and kidney function.

A patient received a kidney transplant last month. Because of the effects of immunosuppressive drugs and CKD, what complication of transplantation should the nurse be assessing the patient for to decrease the risk of mortality? a. Infection b. Rejection c. Malignancy d. Cardiovascular disease

a. Infection is a significant cause of morbidity and mortality after transplantation because the surgery, the immunosuppressive drugs, and the effects of CKD all suppress the body's normal defense mechanisms, thus increasing the risk of infection. The nurse must assess the patient as well as use aseptic technique to prevent infections. Rejection may occur but for other reasons. Malignancy occurrence increases later due to immunosuppressive therapy. Cardiovascular disease is the leading cause of death after renal transplantation but this would not be expected to cause death within the first month after transplantation.

Metabolic acidosis occurs in the oliguric phase of AKI as a result of impairment of a. ammonia synthesis. b. excretion of sodium. c. excretion of bicarbonate. d. conservation of potassium.

a. Metabolic acidosis occurs in AKI because the kidneys cannot synthesize ammonia or excrete acid products of metabolism, resulting in an increased acid load. Sodium is lost in urine because the kidneys cannot conserve sodium. Impaired excretion of potassium results in hyperkalemia. Bicarbonate is normally generated and reabsorbed by the functioning kidney to maintain acid/base balance.

The nurse plans care for the patient with APSGN based on what knowledge? a. Most patients with APSGN recover completely or rapidly improve with conservative management. b. Chronic glomerulonephritis leading to renal failure is a common sequela to acute glomerulonephritis. c. Pulmonary hemorrhage may occur as a result of antibodies also attacking the alveolar basement membrane. d. A large percentage of patients with APSGN develop rapidly progressive glomerulonephritis, resulting in kidney failure.

a. Most patients recover completely from acute poststreptococcal glomerulonephritis (APSGN) with supportive treatment. Chronic glomerulonephritis that progresses insidiously over years and rapidly progressive glomerulonephritis that results in renal failure within weeks or months occur in only a few patients with APSGN. In Goodpasture syndrome, antibodies are present against both the GBM and the alveolar basement membrane of the lungs and dysfunction of both renal and pulmonary are present.

The patient has a thoracic spinal cord lesion and incontinence that occurs equally during the day and night. What type of incontinence is this patient experiencing? a. Reflex incontinence b. Overflow incontinence c. Functional incontinence d. Incontinence after trauma

a. Reflex incontinence occurs with no warning, equally during the day and night, and with spinal cord lesions above S2. Overflow incontinence is when the pressure of urine in the overfull bladder overcomes sphincter control and is caused by bladder or urethral outlet obstruction. Functional incontinence is loss of urine resulting from cognitive, functional, or environmental factors. Incontinence after trauma or surgery occurs when fistulas have occurred or after a prostatectomy.

A patient with an obstruction of the renal artery causing renal ischemia exhibits hypertension. What is one factor that may contribute to the hypertension? a. Increased renin release b. Increased ADH secretion c. Decreased aldosterone secretion d. Increased synthesis and release of prostaglandins

a. Renin is released in response to decreased arterial blood pressure (BP), renal ischemia, decreased extracellular fluid (ECF), decreased serum Na+ concentration, and increased urinary Na+ concentration. It is the catalyst of the reninangiotensin- aldosterone system, which raises stimulated. ADH is secreted by the posterior pituitary in response to serum hyperosmolality and low blood volume. Aldosterone is secreted only after stimulation by angiotensin II. Kidney prostaglandins lower BP by causing vasodilation.

The physician documented that the patient has urinary retention. How should the nurse explain this when the nursing student asks what it is? a. Inability to void b. No urine formation c. Large amount of urine output d. Increased incidence of urination

a. Retention is the inability to void. Anuria is no urine formation. Polyuria is a large amount of urine output over time. Frequency is increased incidence of urination.

14. Using light pressure with the index and middle fingers, the nurse cannot palpate any of the patient's superficial lymph nodes. How should the nurse respond to this assessment? a. Record this finding as normal. b. Reassess the lymph nodes using deeper pressure. c. Ask the patient about any history of radiation therapy. d. Notify the health care provider that x-rays of the nodes will be necessary.

a. Superficial lymph nodes are evaluated by light palpation but they are not normally palpable. It may be normal to find small (<1.0 cm), mobile, firm, nontender nodes. Deep lymph nodes are detected radiographically.

Which characteristic is more likely with acute pyelonephritis than with a lower UTI? a. Fever b. Dysuria c. Urgency d. Frequency

a. Systemic manifestations of fever and chills with leukocytosis and nausea and vomiting are more common in pyelonephritis than in a lower UTI. Dysuria, frequency, and urgency can be present with both.

Which urinary diversion is a continent diversion created by formation of an ileal pouch with a stoma for catheterization? a. Kock pouch b. Ileal conduit c. Orthotopic neobladder d. Cutaneous ureterostomy

a. The Kock pouch is a continent diversion created by formation of an ileal pouch with an external stoma requiring catheterization. Ileal conduit is the most common incontinent diversion using a stoma of resected ileum with implanted ureters. Orthotopic neobladder is a new bladder from a reshaped segment of intestine in the anatomic position of the bladder with urine discharged through the urethra. A cutaneous ureterostomy diverts the ureter from the bladder to the abdominal skin but there is frequent scarring and strictures of the ureters, so ileal conduits are used more often.

What is the most likely reason that the BUN would be increased in a patient? a. Has impaired renal function b. Has not eaten enough protein c. Has decreased urea in the urine d. May have nonrenal tissue destruction

a. The blood urea nitrogen (BUN) is increased in patients with renal problems. It may also be increased when there is rapid or extensive tissue damage from other causes. Low protein intake may cause a low BUN.

A 47-year-old patient who is experiencing andropause has decided to try the testosterone gel Testim. What should the nurse teach the patient and his wife about this gel? a. Wash the hands with soap and water after applying it. b. His wife should apply it to help him feel better about using it. c. Do not wear clothing over the area until it has been absorbed. d. The gel may be taken buccally if it is not effective on the abdomen.

a. The gel may spread the testosterone to others if it is not washed off of his hands after application. If his wife applies the gel, she should wear gloves to prevent absorption of the testosterone and its effects on her body. Clothing over the area until it has dried is recommended. The gel is only topical; a buccal testosterone tablet is called Striant.

19. What preoperative instruction should the nurse give to the patient scheduled for a subtotal thyroidectomy? a. How to support the head with the hands when turning in bed b. Coughing should be avoided to prevent pressure on the incision c. Head and neck will need to remain immobile until the incision heals d. Any tingling around the lips or in the fingers after surgery is expected and temporary

a. To prevent strain on the suture line postoperatively, the patient's head must be manually supported while turning and moving in bed but range-of-motion exercises for the head and neck are also taught preoperatively to be gradually implemented after surgery. There is no contraindication for coughing and deep breathing and these should be carried out postoperatively. Tingling around the lips or fingers is a sign of hypocalcemia, which may occur if the parathyroid glands are inadvertently removed during surgery. This sign should be reported immediately.

A man with end-stage kidney disease is scheduled for hemodialysis following healing of an arteriovenous fistula (AVF). What should the nurse explain to him that will occur during dialysis? a. He will be able to visit, read, sleep, or watch TV while reclining in a chair. b. He will be placed on a cardiac monitor to detect any adverse effects that might occur. c. The dialyzer will remove and hold part of his blood for 20 to 30 minutes to remove the waste products. d. A large catheter with two lumens will be inserted into the fistula to send blood to and return it from the dialyzer.

a. While patients are undergoing hemodialysis, they can perform quiet activities that do not require the limb that has the vascular access. Blood pressure is monitored frequently and the dialyzer monitors dialysis function but cardiac monitoring is not usually indicated. The hemodialysis machine continuously circulates both the blood and the dialysate past the semipermeable membrane in the machine. Graft and fistula access involve the insertion of two needles into the site: one to remove blood from and the other to return blood to the dialyzer.

The nurse explains to the patient undergoing ostomy surgery that the procedure that maintains the most normal functioning of the bowel is a. a sigmoid colostomy b. a transverse colostomy c. a descending colostomy d. an ascending colostomy

a. a sigmoid colostomy

The appropriate collaborative therapy for the patient with acute diarrhea caused by a viral infection is to a. increase fluid intake b. administer an antibiotic. c. administer antimotility drugs d. quarantine the patient to prevent spread of the virus.

a. increase fluid intake

The nurse determines that the goals of dietary teaching have been met when the patient with celiac disease selects from the menu a. scrambled eggs and sausage b. buckwheat pancakes with syrup c. oatmeal, skim milk, and orange juice d. yogurt, strawberries, and rye toast with butter.

a. scrambled eggs and sausage

A patient with osteomyelitis is treated with surgical debridement with implantation of antibiotic beads. When the patient asks why the beads are used, the nurse answers (select all that apply) a."The beads are used to directly deliver antibiotics to the site of the infection." b."There are no effective oral or IV antibiotics to treat most cases of bone infection." c."This is the safest method of delivering long-term antibiotic therapy for a bone infection." d."The beads are an adjunct to debridement and oral and IV antibiotics for deep infections." e."The ischemia and bone death that occur with osteomyelitis are impenetrable to IV antibiotics."

a."The beads are used to directly deliver antibiotics to the site of the infection." d."The beads are an adjunct to debridement and oral and IV antibiotics for deep infections." Treatment of chronic osteomyelitis includes surgical removal of the poorly vascularized tissue and dead bone and the extended use of IV and oral antibiotics. Antibiotic-impregnated polymethylmethacrylate bead chains may be implanted during surgery to aid in combating the infection.

The increased risk for falls in the older adult is most likely due to a.changes in balance. b.decrease in bone mass. c.loss of ligament elasticity. d.erosion of articular cartilage.

a.changes in balance Aging can cause changes in a person's sense of balance, making the person unsteady, and proprioception may be altered. The risk for falls also increases in older adults partly because of a loss of strength.

A patient with tendonitis asks what the tendon does. The nurse's response is based on the knowledge that tendons a.connect bone to muscle. b.provide strength to muscle. c.lubricate joints with synovial fluid. d.relieve friction between moving parts.

a.connect bone to muscle Tendons are composed of dense, fibrous connective tissue that contains bundles of closely packed collagen fibers arranged in the same plane for additional strength. They connect the muscle sheath to adjacent bone.

While performing passive range of motion for a patient, the nurse puts the ankle joint through the movements of (select all that apply) a.flexion and extension. b.inversion and eversion. c.pronation and supination d.flexion, extension, abduction, and adduction. e.pronation, supination, rotation, and circumduction.

a.flexion and extension. b.inversion and eversion. Common movements that occur at the ankle include inversion, eversion, flexion, and extension.

A normal assessment finding of the musculoskeletal system is a.no deformity or crepitation. b.muscle and bone strength of 4. c.ulnar deviation and subluxation. d.angulation of bone toward midline.

a.no deformity or crepitation Normal physical assessment findings of the musculoskeletal system include normal spinal curvatures; no muscle atrophy or asymmetry; no joint swelling, deformity, or crepitation; no tenderness on palpation of muscles and joints; full range of motion of all joints without pain or laxity; and muscle strength score of 5.

Before discharge from the same-day surgery unit, instruct the patient who has had a surgical correction of bilateral hallux valgus to a.rest frequently with the feet elevated. b.soak the feet in warm water several times a day. c.expect the feet to be numb for the next few days. d.expect continued pain in the feet, since this is not uncommon.

a.rest frequently with the feet elevated. After surgical correction of bilateral hallux valgus, the feet should be elevated with the heel off the bed to help reduce discomfort and prevent edema.

A common site for the lesions associated with atopic dermatitis is the

antecubital space

A mother and her two children have been diagnosed with pediculosis corporis at a health care center. An appropriate measure in treating this condition is:

applying pyrethrins to the body

Important patient teaching after a chemical peel includes:

avoidance of sun exposure

Presbyopia occurs in older individuals because a. the eyeball elongates. b. the lens becomes inflexible. c. the corneal curvature becomes irregular. d. light rays are focusing in front of the retina.

b Rationale: Presbyopia is the loss of accommodation in association with age. As the eye ages, the lens becomes larger, firmer, and less elastic.

Teaching in relation to home management after a laparoscopic cholecystectomy should include a. keeping the bandages on the puncture sites for 48 hours. b. reporting any bile-colored drainage or pus from any incision. c. using over-the-counter antiemetics if nausea and vomiting occur. d. emptying and measuring the contents of the bile bag from the T tube every day. (Lewis 1042)

b Rationale: The following discharge instructions are taught to the patient and caregiver after a laparoscopic cholecystectomy: First, remove the bandages on the puncture site the day after surgery and shower. Second, notify the surgeon if any of the following signs and symptoms occur: redness, swelling, bile-colored drainage or pus from any incision; and severe abdominal pain, nausea, vomiting, fever, or chills. Third, gradually resume normal activities. Fourth, return to work within 1 week of surgery. Fifth, resume a usual diet, but a low-fat diet is usually better tolerated for several weeks after surgery.

Which therapies for BPH are done on an outpatient basis (select all that apply)? a. Intraprostatic urethral stents b. Transurethral needle ablation (TUNA) c. Transurethral incision of prostate (TUIP) d. Transurethral microwave therapy (TUMT) e. Visual laser ablation of the prostate (VLAP)

b, c, d. TUNA, TUIP, and TUMT are currently done on an outpatient basis or in a health care provider's office.

Instruct the patient who is newly fitted with bilateral hearing aids to (select all that apply) a. replace the batteries monthly. b. clean the ear molds weekly or as needed. c. clean ears with cotton-tipped applicators daily. d. disconnect or remove the batteries when not in use. e. initially restrict usage to quiet listening in the home.

b, d, e Rationale: Initially, use of the hearing aid should be restricted to quiet situations in the home. As adjustment to the increase in sounds and background noise occurs, the patient can progress to using the hearing aid in situations in which several people are talking simultaneously. Next, use can be expanded to the outdoors and then shopping malls or grocery stores. When the hearing aid is not being worn, it should be should be disconnected or removed when not in use. Battery life averages 1 week. Ear molds should be cleaned weekly or as needed.

Which strategies would best assist the nurse in communicating with a patient who has a hearing loss (select all that apply)? a. Overenunciate speech. b. Speak normally and slowly. c. Exaggerate facial expressions. d. Raise the voice to a higher pitch. e. Write out names or difficult words.

b, e Rationale: Speak normally and slowly directly into the patient's better ear. Do not exaggerate facial expressions. Do not overenunciate. Use simple sentences; rephrase sentences; use different words. Write out names or difficult words. Avoid shouting.

The male patient is Jewish, has a history of gout, and has been diagnosed with renal calculi. Which treatment will be used with this patient (select all that apply)? a. Reduce dietary oxalate b. Administer allopurinol c. Administer α-penicillamine d. Administer thiazide diuretics e. Reduce animal protein intake f. Reduce intake of milk products

b, e. This patient is most likely to have uric acid calculi, which have a high incidence in Jewish men, and gout is a predisposing factor. The treatment will include allopurinol and reducing animal protein intake to reduce purine, as uric acid is a waste product from purine metabolism. Reducing oxalate and using thiazide diuretics help to treat calcium oxalate calculi. Administration of α-penicillamine and tiopronin prevent cystine crystallization for cystine calculi. Reducing intake of milk products to reduce calcium intake may be used with calcium calculi.

In a patient who has a hemorrhage in the posterior cavity of the eye, the nurse knows that blood is accumulating a. in the aqueous humor. b. between the lens and the retina. c. between the cornea and the lens. d. in the space between the iris and the lens.

b. Rationale: The posterior chamber lies between the anterior surface of the lens and the posterior surface of the iris. The posterior cavity lies in the large space behind the lens and in front of the retina.

What is an appropriate nursing intervention for the patient with hyperparathyroidism? a. Pad side rails as a seizure precaution. b. Increase fluid intake to 3000 to 4000 mL daily. c. Maintain bed rest to prevent pathologic fractures. d. Monitor the patient for Trousseau's and Chvostek's signs.

b. A high fluid intake is indicated in hyperparathyroidism to dilute the hypercalcemia and flush the kidneys so that calcium stone formation is reduced. Seizures are not associated with hyperparathyroidism. Impending tetany of hypoparathyroidism after parathyroidectomy can be noted with Trousseau's and Chvostek's signs. The patient with hyperparathyroidism is at risk for pathologic fractures resulting from decreased bone density but mobility is encouraged to promote bone calcification.

Priority Decision: Following electrohydraulic lithotripsy for treatment of renal calculi, the patient has a nursing diagnosis of risk for infection related to the introduction of bacteria following manipulation of the urinary tract. What is the most appropriate nursing intervention for this patient? a. Monitor for hematuria. b. Encourage fluid intake of 3 L/day. c. Apply moist heat to the flank area. d. Strain all urine through gauze or a special strainer.

b. A high fluid intake maintains dilute urine, which decreases bacterial concentration in addition to washing stone fragments and expected blood through the urinary system following lithotripsy. High urine output also prevents supersaturation of minerals. Moist heat to the flank may be helpful to relieve muscle spasms during renal colic and all urine should be strained in patients with renal stones to collect and identify stone composition but these are not related to infection.

During assessment of the patient who has a nephrectomy, what should the nurse expect to find? a. Shallow, slow respirations b. Clear breath sounds in all lung fields c. Decreased breath sounds in the lower left lobe d. Decreased breath sounds in the right and left lower lobes

b. A nephrectomy incision is usually in the flank, just below the diaphragm or in the abdominal area. Although the patient is reluctant to breathe deeply because of incisional pain, the lungs should be clear. Decreased sounds and shallow respirations are abnormal and would require intervention.

What is the primary way that a nurse will evaluate the patency of an AVF? a. Palpate for pulses distal to the graft site. b. Auscultate for the presence of a bruit at the site. c. Evaluate the color and temperature of the extremity. d. Assess for the presence of numbness and tingling distal to the site.

b. A patent arteriovenous fistula (AVF) creates turbulent blood flow that can be assessed by listening for a bruit or palpated for a thrill as the blood passes through the graft. Assessment of neurovascular status in the extremity distal to the graft site is important to determine that the graft does not impair circulation to the extremity but the neurovascular status does not indicate whether the graft is open.

19. If a patient with blood type O Rh+ is given AB Rh- blood, what would the nurse expect to happen? a. The patient's Rh factor will react with the RBCs of the donor blood. b. The anti-A and anti-B antibodies in the patient's blood will hemolyze the donor blood. c. The anti-A and anti-B antibodies in the donor blood will hemolyze the patient's blood. d. No adverse reaction is expected because the patient has no antibodies against the donor blood.

b. A patient with type O Rh+ blood has no A or B antigens on the RBC but does have anti-A and anti-B antibodies in the blood and has an Rh antigen. Type AB Rh− blood has both A and B antigens on the RBC but no Rh antigen and no anti-A or anti-B antibodies. If the type AB Rh− blood is given to the patient with type O Rh+ blood, the antibodies in the patient's blood will react with the antigens in the donor blood, causing hemolysis of the donor cells. There will be no Rh reaction because the donor blood has no Rh antigen.

In a patient with AKI, which laboratory urinalysis result indicates tubular damage? a. Hematuria b. Specific gravity fixed at 1.010 c. Urine sodium of 12 mEq/L (12 mmol/L) d. Osmolality of 1000 mOsm/kg (1000 mmol/kg)

b. A urine specific gravity that is consistently 1.010 and a urine osmolality of about 300 mOsm/kg is the same specific gravity and osmolality as plasma. This indicates that tubules are damaged and unable to concentrate urine. Hematuria is more common with postrenal damage. Tubular damage is associated with a high sodium concentration (greater than 40 mEq/L).

34. During discharge teaching for the patient with Addison's disease, which statement by the patient indicates that the nurse needs to do additional teaching? a. "I should always call the doctor if I develop vomiting or diarrhea." b. "If my weight goes down, my dosage of steroid is probably too high." c. "I should double or triple my steroid dose if I undergo rigorous physical exercise." d. "I need to carry an emergency kit with injectable hydrocortisone in case I can't take my medication by mouth."

b. A weight reduction in the patient with Addison's disease may indicate a fluid loss and a dose of replacement therapy that is too low rather than too high. Because vomiting and diarrhea are early signs of crisis and because fluid and electrolytes must be replaced, patients should notify their health care provider if these symptoms occur. Patients with Addison's disease are taught to take two to three times their usual dose of steroids if they become ill, have teeth extracted, or engage in rigorous physical activity and should always have injectablehydrocortisone available if oral doses cannot be taken.

The nurse provides discharge teaching to a patient following a TURP and determines that the patient understands the instructions when he makes which statement? a. "I should use daily enemas to avoid straining until healing is complete." b. "I should avoid heavy lifting, climbing, and driving until my follow-up visit." c. "At least I don't have to worry about developing cancer of the prostate now." d. "Every day I should drink 10 to 12 glasses of liquids such as coffee, tea, or soft drinks."

b. Activities that increase intraabdominal pressure should be avoided until the surgeon approves these activities at a follow-up visit. Stool softeners and high-fiber diets may be used to promote bowel elimination but enemas should not be used because they increase intraabdominal pressure and may initiate bleeding. Because TURP does not remove the entire prostate gland, the patient needs annual prostatic examinations to screen for cancer of the prostate. Fluid intake should be high but caffeine and alcohol should not be used because they have a diuretic effect and increase bladder distention.

In providing care for the patient with adult-onset polycystic kidney disease, what should the nurse do? a. Help the patient to cope with the rapid progression of the disease b. Suggest genetic counseling resources for the children of the patient c. Expect the patient to have polyuria and poor concentration ability of the kidneys d. Implement measures for the patient's deafness and blindness in addition to the renal problems

b. Adult-onset polycystic kidney disease is an inherited autosomal dominant disorder that often manifests after the patient has children but the children should receive genetic counseling regarding their life choices. The disease progresses slowly, eventually causing progressive renal failure. Hereditary medullary cystic disease causes poor concentration ability of the kidneys and classic Alport syndrome is a hereditary nephritis that is associated with deafness and deformities of the optic lens.

In which type of dialysis does the patient dialyze during sleep and leave the fluid in the abdomen during the day? a. Long nocturnal hemodialysis b. Automated peritoneal dialysis (APD) c. Continuous venovenous hemofiltration (CVVH) d. Continuous ambulatory peritoneal dialysis (CAPD)

b. Automated peritoneal dialysis (APD) is the type of dialysis in which the patient dialyzes during sleep and leaves the fluid in the abdomen during the day. Long nocturnal hemodialysis occurs while the patient is sleeping and is done up to six times per week. Continuous venovenous hemofiltration (CVVH) is a type of continuous renal replacement therapy used to treat AKI. Continuous ambulatory peritoneal dialysis (CAPD) is dialysis that is done with exchanges of 1.5 to 3 L of dialysate at least four times daily.

A urinalysis of a urine specimen that is not processed within 1 hour may result in erroneous measurement of a. glucose. b. bacteria. c. specific gravity. d. white blood cells.

b. Bacteria in warm urine specimens multiply rapidly and false or unreliable bacterial counts may occur with urine that has been sitting for periods of time. Glucose, specific gravity, and WBCs do not change in urine specimens but pH becomes more alkaline, RBCs are hemolyzed, and casts may disentigrate.

Before undergoing a TURP, what should the patient be taught? a. Some degree of urinary incontinence is likely to occur. b. This surgery results in some degree of retrograde ejaculation. c. Erectile dysfunction is a common complication of this prostate surgery. d. An indwelling catheter will be used to maintain urinary output until healing is complete.

b. Because of injury to the internal urinary sphincter, there is usually some degree of retrograde ejaculation following most transurethral surgeries, especially following TURP. The semen is ejaculated into the bladder and is eliminated with the next voiding. Urinary incontinence, erectile dysfunction, and continued catheterization are uncommon following TURP.

A teaching plan developed by the nurse for the patient with a new ileal conduit includes instructions to do what? a. Clean the skin around the stoma with alcohol every day. b. Use a wick to keep the skin dry during appliance changes. c. Use sterile supplies and technique during care of the stoma. d. Change the appliance every day and wash it with soap and warm water.

b. Because the stoma continuously drains urine, a wick formed of a rolled-up 4 × 4 gauze or a tampon is held against the stoma to absorb the urine while the skin is cleaned and a new appliance is attached. The skin is cleaned with warm water only because soap and other agents cause drying and irritation and clean, not sterile, technique is used. The appliance should be left in place for as long as possible before it loosens and allows leakage onto the skin, perhaps up to 14 days.

Priority Decision: Following a renal biopsy, what is the nurse's priority? a. Offer warm sitz baths to relieve discomfort. b. Test urine for microscopic bleeding with a dipstick. c. Expect the patient to experience burning on urination. d. Monitor the patient for symptoms of a urinary infection.

b. Bleeding from the kidney following a biopsy is the most serious complication of the procedure and urine must be examined for both gross and microscopic blood, in addition to vital signs and hematocrit levels being monitored. Following a cystoscopy the patient may have burning with urination and warm sitz baths may be used. Urinary infections are a complication of any procedure requiring instrumentation of the bladder.

Which urinalysis results most likely indicate a urinary tract infection (UTI)? a. Yellow; protein 6 mg/dL; pH 6.8; 102/mL bacteria b. Cloudy, yellow; WBC >5/hpf; pH 8.2; numerous casts c. Cloudy, brown; ammonia odor; specific gravity 1.030; RBC 3/hpf d. Clear; colorless; glucose: trace; ketones: trace; osmolality 500 mOsm/kg (500 mmol/kg)

b. Cloudiness in a fresh urine specimen, WBC count above 5 per high-power field (hpf), and the presence of casts are all indicative of urinary tract infection (UTI). The pH is usually elevated because bacteria in urine split the urea alkaline ammonia. Cloudy, brown urine usually indicates hematuria or the presence of bile. Colorless urine is usually very dilute. Option a is characteristic of normal urine.

A patient with AKI has a serum potassium level of 6.7 mEq/L (6.7 mmol/L) and the following arterial blood gas results: pH 7.28, PaCO2 30 mm Hg, PaO2 86 mm Hg, HCO3 − 18 mEq/L (18 mmol/L). The nurse recognizes that treatment of the acid-base problem with sodium bicarbonate would cause a decrease in which value? a. pH b. Potassium level c. Bicarbonate level d. Carbon dioxide level

b. During acidosis, potassium moves out of the cell in exchange for H+ ions, increasing the serum potassium level. Correction of the acidosis with sodium bicarbonate will help to shift the potassium back into the cells. A decrease in pH and the bicarbonate and PaCO2 levels would indicate worsening acidosis.

9. Laboratory test results indicate increased fibrin split products (FSPs). An appropriate nursing action is to monitor the patient for a. fever. b. bleeding. c. faintness. d. thrombotic episodes

b. During fibrinolysis by plasmin, the fibrin clot is split into smaller molecules known as FSPs or FDPs. Increased FSPs impair platelet aggregation, reduce prothrombin, and prevent fibrin stabilization and lead to bleeding.

The family caregiver for a patient with AD expresses an inability to make decisions, concentrate, or sleep. The nurse determines what about the caregiver? a. The caregiver is also developing signs of AD. b. The caregiver is manifesting symptoms of caregiver role strain. c. The caregiver needs a period of respite from care of the patient. d. The caregiver should ask other family members to participate in the patient's care.

b. Family caregiver role strain is characterized by such symptoms of stress as the inability to sleep, make decisions, or concentrate. It is frequently seen in family members who are responsible for the care of the patient with AD. Assessment of the caregiver may reveal a need for assistance to increase coping skills, effectively use community resources, or maintain social relationships. Eventually the demands on a caregiver exceed the resources and the person with AD may be placed in an institutional setting.

The male patient is admitted with a diagnosis of benign prostatic hyperplasia (BPH). What urination characteristics should the nurse expect to assess in this patient? a. Oliguria b. Hesitancy c. Hematuria d. Pneumaturia

b. Hesitancy is difficulty starting the urine stream and is common with benign prostatic hyperplasia (BPH). Oliguria is scanty urine formation and output. Hematuria is blood in the urine. Pneumaturia is urine containing gas, as is caused by a fistula between the bowel and bladder.

What is the most serious electrolyte disorder associated with kidney disease? a. Hypocalcemia b. Hyperkalemia c. Hyponatremia d. Hypermagnesemia

b. Hyperkalemia can lead to life-threatening dysrhythmias. Hypocalcemia leads to an accelerated rate of bone remodeling and potentially to tetany. Hyponatremia may lead to confusion. Elevated sodium levels lead to edema, hypertension, and heart failure. Hypermagnesemia may decrease reflexes, mental status, and blood pressure.

Priority Decision: A dehydrated patient is in the Injury stage of the RIFLE staging of AKI. What would the nurse first anticipate in the treatment of this patient? a. Assess daily weight b. IV administration of fluid and furosemide (Lasix) c. IV administration of insulin and sodium bicarbonate d. Urinalysis to check for sediment, osmolality, sodium, and specific gravity

b. Injury is the stage of RIFLE classification when urine output is less than 0.5 mL/kg/hr for 12 hours, the serum creatinine is increased times two or the glomerular filtration rate (GFR) is decreased by 50%. This stage may be reversible by treating the cause or, in this patient, the dehydration by administering IV fluid and a low dose of a loop diuretic, furosemide (Lasix). Assessing the daily weight will be done to monitor fluid changes but it is not the first treatment the nurse should anticipate. IV administration of insulin and sodium bicarbonate would be used for hyperkalemia. Checking the urinalysis will help to determine if the AKI has a prerenal, intrarenal, or postrenal cause by what is seen in the urine but with this patient's dehydration, it is thought to be prerenal to begin treatment.

The patient with CKD asks why she is receiving nifedipine (Procardia) and furosemide (Lasix). The nurse understands that these drugs are being used to treat the patient's a. anemia. b. hypertension. c. hyperkalemia. d. mineral and bone disorder.

b. Nifedipine (Procardia) is a calcium channel blocker and furosemide (Lasix) is a loop diuretic. Both are used to treat hypertension.

A patient has a right ureteral catheter placed following a lithotripsy for a stone in the ureter. In caring for the patient after the procedure, what is an appropriate nursing action? a. Milk or strip the catheter every 2 hours. b. Measure ureteral urinary drainage every 1 to 2 hours. c. Irrigate the catheter with 30-mL sterile saline every 4 hours. d. Encourage ambulation to promote urinary peristaltic action.

b. Output from ureteral catheters must be monitored every 1 to 2 hours because an obstruction will cause overdistention of the renal pelvis and renal damage. The renal pelvis has a capacity of only 3 to 5 mL and if irrigation is ordered, no more than 5 mL of sterile saline is used. The patient with a ureteral catheter is usually kept on bed rest until specific orders for ambulation are given. Suprapubic tubes may be milked to prevent obstruction of the catheter by sediment and clots.

To prevent the most common serious complication of PD, what is important for the nurse to do? a. Infuse the dialysate slowly. b. Use strict aseptic technique in the dialysis procedures. c. Have the patient empty the bowel before the inflow phase. d. Reposition the patient frequently and promote deep breathing.

b. Peritonitis is a common complication of peritoneal dialysis (PD) and may require catheter removal and termination of dialysis. Infection occurs from contamination of the dialysate or tubing or from progression of exit-site or tunnel infections and strict sterile technique must be used by health professionals as well as the patient to prevent contamination. Too-rapid infusion may cause shoulder pain and pain may be caused if the catheter tip touches the bowel. Difficulty breathing, atelectasis, and pneumonia may occur from pressure of the fluid on the diaphragm, which may be prevented by elevating the head of the bed and promoting repositioning and deep breathing.

In replying to a patient's questions about the seriousness of her chronic kidney disease (CKD), the nurse knows that the stage of CKD is based on what? a. Total daily urine output b. Glomerular filtration rate c. Degree of altered mental status d. Serum creatinine and urea levels

b. Stages of chronic kidney disease are based on the GFR. No specific markers of urinary output, mental status, or azotemia classify the degree of chronic kidney disease (CKD).

The female patient with a UTI also has renal calculi. The nurse knows that these are most likely which type of stone? a. Cystine b. Struvite c. Uric acid d. Calcium phosphate

b. Struvite calculi are most common in women and always occur with UTIs. They are also usually large staghorn type.

A female patient with a UTI has a nursing diagnosis of risk for infection related to lack of knowledge regarding prevention of recurrence. What should the nurse include in the teaching plan instructions for this patient? a. Empty the bladder at least 4 times a day. b. Drink at least 2 quarts of water every day. c. Wait to urinate until the urge is very intense. d. Clean the urinary meatus with an antiinfective agent after voiding.

b. The bladder should be emptied at least every 3 to 4hours. Fluid intake should be increased to about 2000 mL/ day without caffeine, alcohol, citrus juices, and chocolate drinks, because they are potential bladder irritants. Cleaning the urinary meatus with an antiinfective agent after voiding will irritate the meatus but the perineal area should be wiped from front to back after urination and defecation to prevent fecal contamination of the meatus.

Which infection is asymptomatic in the male patient at first and then progresses to cystitis, frequent urination, burning on voiding, and epididymitis? a. Urosepsis b. Renal tuberculosis c. Urethral diverticula d. Goodpasture syndrome

b. The manifestations of renal tuberculosis are described. Urosepsis is when the UTI has spread systemically. Urethral diverticula are localized outpouching of the urethra and occur more often in women. Goodpasture syndrome manifests with flu-like symptoms with pulmonary symptoms that include cough, shortness of breath, and pulmonary insufficiency and renal manifestations that include hematuria, weakness, pallor, anemia, and renal failure.

Priority Decision: A patient with continuous bladder irrigation following a prostatectomy tells the nurse that he has bladder spasms and leaking of urine around the catheter. What should the nurse do first? a. Slow the rate of the irrigation. b. Assess the patency of the catheter. c. Encourage the patient to try to urinate around the catheter. d. Administer a belladonna and opium (B&O) suppository as prescribed.

b. The nurse should first check for the presence of clots obstructing the catheter or tubing and then may administer a belladonna and opium (B&O) suppository if one is ordered. The patient should not try to void around the catheter because this will increase the spasms. The flow rate of the irrigation fluid may be decreased if orders permit because fast-flowing, cold fluid may also contribute to spasms.

With which diagnosis will the patient benefit from being taught to do self-catheterization? a. Renal trauma b. Urethral stricture c. Renal artery stenosis d. Accelerated nephrosclerosis

b. The patient with urethral stricture will benefit from being taught to dilate the urethra by self-catheterization every few days. Renal trauma is treated related to the severity of the injury with bed rest, fluids, and analgesia. Renal artery stenosis includes control of hypertension with possible surgical revascularization. Accelerated nephrosclerosis is associated with malignant hypertension that must be aggressively treated as well as monitoring kidney function.

Which treatment for BPH uses a low-wave radiofrequency to precisely destroy prostate tissue? a. Laser prostatectomy b. Transurethral needle ablation (TUNA) c. Transurethral microwave thermotherapy (TUMT) d. Transurethral electrovaporization of prostate (TUVP)

b. The transurethral needle ablation (TUNA) uses lowwave radiofrequency to heat the prostate, causing necrosis. Laser prostatectomy uses a laser beam. Transurethral microwave thermotherapy (TUMT) uses microwave radiating heat to produce coagulative necrosis of the prostate and is not used for men with rectal problems. Transurethral electrovaporization of prostate (TUVP) uses electrosurgical vaporization and desiccation to destroy prostate tissue.

A 68-year-old man with a history of heart failure resulting from hypertension has AKI as a result of the effects of nephrotoxic diuretics. Currently his serum potassium is 6.2 mEq/L (6.2 mmol/L) with cardiac changes, his BUN is 108 mg/dL (38.6 mmol/L), his serum creatinine is 4.1 mg/dL (362 mmol/L), and his serum HCO3 − is 14 mEq/L (14 mmol/L). He is somnolent and disoriented. Which treatment should the nurse expect to be used for him? a. Loop diuretics b. Renal replacement therapy c. Insulin and sodium bicarbonate d. Sodium polystyrene sulfonate (Kayexalate)

b. This patient has at least three of the six common indications for renal replacement therapy (RRT), including (1) high potassium level, (2) metabolic acidosis, and (3) changed mental status. The other indications are (4) volume overload, resulting in compromised cardiac status (this patient has a history of hypertension), (5) BUN greater than 120 mg/dL, and (6) pericarditis, pericardial effusion, or cardiac tamponade. Although the other treatments may be used, they will not be as effective as RRT for this older patient. Loop diuretics and increased fluid are used if the patient is dehydrated. Insulin and sodium bicarbonate can be used to temporarily drive the potassium into the cells. Sodium polystyrene sulfonate (Kayexalate) is used to actually decrease the amount of potassium in the body.

Which volume of urine in the bladder would cause discomfort and require urinary catheterization? a. 250 mL b. 500 mL c. 1200 mL d. 1500 mL

b. When the amount of urine in the bladder has reached 1200 mL, the person would need relief and probably catheterization. The bladder capacity ranges from 600 to1000 mL. When there is 250 mL of urine in the bladder, the person will usually feel the urge to urinate and 400 to 600 mL will be uncomfortable.

The nurse would increase the comfort a the patient with appendicitis by a. having the patient lie prone. b. flexing the patient's right knee. c. sitting the patient upright in a char. d. turn the patient onto his or her left side.

b. flexing the patient's right knee.

In contrast to diverticulitis, the patient with diverticulosis a. has no rectal bleeding b. often has no symptoms c. has localized cramping pain d. frequently develops peritonitis

b. often has no symptoms

Serum tumor markers that may be elevated on diagnosis of testicular cancer and used to monitor the response to therapy include a. tumor necrosis factor (TNF) and C-reactive protein (CRP). b. α-fetoprotein (AFP) and human chorionic gonadotropin (hCG). c. prostate-specific antigen (PSA) and prostate acid phosphatase (PAP). d. carcinoembryonic antigen (CEA) and antinuclear antibody (ANA).

b. α-Fetoprotein (AFP) and human chorionic gonadotropin (hCG) are glycoproteins that may be elevated in testicular cancer. If they are elevated before surgical treatment, the levels are noted, and if response to therapy is positive, the levels will decrease. Lactate dehydrogenase (LDH) may also be elevated. Tumor necrosis factor (TNF) is a normal cytokine responsible for tumor surveillance and destruction. C-reactive protein (CRP) is found in inflammatory conditions and widespread malignancies. PSA and PAP are used for screening of prostatic cancer. Carcinoembryonic antigen (CEA) is a tumor marker for cancers of the GI system. Antinuclear antibody (ANA) is found most frequently in autoimmune disorders.

A patient has been diagnosed with osteosarcoma of the humerus. He shows an understanding of his treatment options when he states a."I accept that I have to lose my arm with surgery." b."The chemotherapy before surgery will shrink the tumor." c."This tumor is related to the melanoma I had 3 years ago." d."I'm glad they can take out the cancer with such a small scar."

b."The chemotherapy before surgery will shrink the tumor." A patient with osteosarcoma usually has preoperative chemotherapy to decrease tumor size before surgery. As a result, limb-salvage procedures, including a wide surgical resection of the tumor, are being used more often. Osteosarcoma is a primary bone tumor that is extremely aggressive and rapidly metastasizes to distant sites.

You are teaching a patient with osteopenia. What is important to include in the teaching plan? a.Lose weight. b.Stop smoking. c.Eat a high-protein diet. d.Start swimming for exercise

b.Stop smoking. Patients with osteopenia should be instructed to quit smoking in order to decrease loss of bone mass.

In caring for a patient after a spinal fusion, the nurse would immediately report to the physician which patient symptom? a.The patient experiences a single episode of emesis. b.The patient is unable to move the lower extremities. c. The patient is nauseated and has not voided in 4 hours. d. The patient complains of pain at the bone graft donor site.

b.The patient is unable to move the lower extremities. After spinal fusion surgery, the nurse should frequently monitor peripheral neurologic signs. Movement of the arms and legs and assessment of sensation should be unchanged in comparison with the preoperative status. These assessments are usually repeated every 2 to 4 hours during the first 48 hours after surgery, and findings are compared with those of the preoperative assessment. Paresthesias, such as numbness and tingling sensation, may not be relieved immediately after surgery. The nurse should document any new muscle weakness or paresthesias and report this to the surgeon immediately.

A patient is scheduled for an electromyogram (EMG). The nurse explains that this diagnostic test involves a.incision or puncture of the joint capsule. b.insertion of small needles into certain muscles. c.administration of a radioisotope before the procedure. d.placement of skin electrodes to record muscle activity.

b.insertion of small needles into certain muscles Electromyography (EMG) is an evaluation of electrical potential associated with skeletal muscle contraction. Small-gauge needles are inserted into certain muscles and attached to leads that record electrical activity of muscle. Results provide information related to lower motor neuron dysfunction and primary muscle disease

What should be included in the nursing plan for a patient who needs to administer antibiotic eardrops? a. Cool the drops so that they decrease swelling in the canal. b. Avoid placing a cotton wick to assist in administering the drops. c. Be careful to avoid touching the tip of the dropper bottle to the ear. d. Keep the head tilted 5 to 7 minutes after administration of the drops.

c Rationale: Hands should be washed before and after administration of otic drops (eardrops). The drops should be administered at room temperature, because cold drops can cause vertigo and heated drops can burn the tympanum. To prevent contamination of the entire bottle of drops, the tip of the dropper should not touch the ear during administration. The ear is positioned so that the drops can run into the canal. This position should be maintained for 2 minutes after eardrop administration to allow the drops to become instilled. Drops can be placed onto a wick of cotton that is placed in the canal. Instruct the patient not to push the cotton farther into the ear. Material saturated with drainage should be carefully handled and discarded.

A patient with pancreatic cancer is admitted to the hospital for evaluation of possible treatment options. The patient asks the nurse to explain the Whipple procedure that the surgeon has described. The explanation includes the information that a Whipple procedure involves a. creating a bypass around the obstruction caused by the tumor by joining the gallbladder to the jejunum. b. resection of the entire pancreas and the distal portion of the stomach, with anastomosis of the common bile duct and the stomach into the duodenum. c. removal of part of the pancreas, part of the stomach, the duodenum, and the gallbladder, with joining of the pancreatic duct, the common bile duct, and the stomach into the jejunum. d. radical removal of the pancreas, the duodenum, and the spleen, and attachment of the stomach to the jejunum, which requires oral supplementation of pancreatic digestive enzymes and insulin replacement therapy. (Lewis 1042)

c Rationale: The classic operation for pancreatic cancer is a radical pancreaticoduodenectomy, or Whipple procedure. This entails resection of the proximal pancreas (i.e., proximal pancreatectomy), the adjoining duodenum (i.e., duodenectomy), the distal portion of the stomach (i.e., partial gastrectomy), and the distal segment of the common bile duct. The pancreatic duct, common bile duct, and stomach are anastomosed to the jejunum.

Which drugs are used to treat overflow incontinence (select all that apply)? a. Baclofen (Lioresal) b. Anticholinergic drugs c. α-Adrenergic blockers d. 5α-reductase inhibitors e. Bethanechol (Urecholine)

c, d, e. α-Adrenergic blockers block the stimulation of the smooth muscle of the bladder, 5α reductase inhibitors decrease outlet resistance, and bethanechol enhances bladder contractions. Baclofen or diazepam is used to relax the external sphincter for reflex incontinence. Anticholinergics are used to relax bladder tone and increase sphincter tone with urge incontinence.

An 83-year-old female patient was found lying on the bathroom floor. She said she fell 2 days ago and has not been able to take her heart medicine or eat or drink anything since then. What conditions could be causing prerenal AKI in this patient (select all that apply)? a. Anaphylaxis b. Renal calculi c. Hypovolemia d. Nephrotoxic drugs e. Decreased cardiac output

c, e. Because the patient has had nothing to eat or drink for 2 days, she is probably dehydrated and hypovolemic. Decreased cardiac output (CO) is most likely because she is older and takes heart medicine, which is probably for heart failure or hypertension. Both hypovolemia and decreased CO cause prerenal AKI. Anaphylaxis is also a cause of prerenal AKI but is not likely in this situation. Nephrotoxic drugs would contribute to intrarenal causes of AKI and renal calculi would be a postrenal cause of AKI.

The patient with chronic kidney disease is considering whether to use peritoneal dialysis (PD) or hemodialysis (HD). What are advantages of PD when compared to HD (select all that apply)? a. Less protein loss b. Rapid fluid removal c. Less cardiovascular stress d. Decreased hyperlipidemia e. Requires fewer dietary restrictions

c, e. Peritoneal dialysis is less stressful for the cardiovascular system and requires fewer dietary restrictions. Peritoneal dialysis actually contributes to more protein loss and increased hyperlipidemia. The fluid and creatinine removal are slower with peritoneal dialysis than hemodialysis.

A patient with metastatic colorectal cancer is scheduled for both chemotherapy and radiation therapy. Patient teaching regarding these therapies for this patient would include an explanation that a. Chemotherapy can be used to cure colorectal cancer. b radiation is routinely used as adjuvant therapy following surgery. c. both chemotherapy and radiation can be used a palliative treatments. d. the patient should expect few if any side effects from chemotherapeutic agents.

c.

Question patients using eyedrops to treat their glaucoma about a. use of corrective lenses. b. their usual sleep pattern. c. a history of heart or lung disease. d. sensitivity to opioids or depressants.

c. Rationale: Assess whether the patient is taking -adrenergic blockers, because these drugs can be potentiated by the β-adrenergic blockers used to treat glaucoma. Many preparations for colds contain a form of epinephrine (i.e., pseudoephedrine) that can cause the pupils to dilate, and antihistamines or decongestants can cause ocular dryness.

During an assessment of hearing, the nurse would expect to find normal finding of a. absent cone of light. b. bluish purple tympanic membrane. c. midline tone heard equally in both ears. d. fluid level at hairline in the tympanum.

c. Rationale: Normal findings of physical assessment of the auditory system include symmetry of the ears in location and shape; nontenderness and no lesions of auricles and tragus; clearness of the canal; pearl-gray color of tympanic membrane with landmarks and light reflex intact; ability of the patient to hear low whisper at a distance of 30 cm; better Rinne test results for air conduction than for bone conduction (AC > BC); and no lateralization according to the Weber test results.

Which diagnostic study would include assessing for iodine sensitivity, teaching the patient to take a cathartic the night before the procedure, and telling the patient that a salty taste may occur during the procedure? a. Cystometrogram b. Renal arteriogram c. Intravenous pyelogram (IVP) d. Kidneys, ureters, bladder (KUB)

c. A cathartic the evening before the procedure and sensitivity to iodine are important for both intravenous pyelogram (IVP) and renal arteriogram but the salty taste is only a possibility with IVP. The cystometrogram involves filling the bladder with water or saline to measure tone and stability. The kidneys, ureters, and bladder (KUB) is an x-ray that may have bowel preparation.

On assessment of the patient with a renal calculus passing down the ureter, what should the nurse expect the patient to report? a. A history of chronic UTIs b. Dull, costovertebral flank pain c. Severe, colicky back pain radiating to the groin d. A feeling of bladder fullness with urgency and frequency

c. A classic sign of the passage of a calculus down the ureter is intense, colicky back pain that may radiate into the testicles, labia, or groin and may be accompanied by mild shock with cool, moist skin. Many patients with renal stones do not have a history of chronic UTIs. Stones obstructing a calyx or at the ureteropelvic junction may produce dull costovertebral flank pain and large bladder stones may cause bladder fullness and lower obstructive symptoms.

A patient on hemodialysis develops a thrombus of a subcutaneous arteriovenous (AV) graft, requiring its removal. While waiting for a replacement graft or fistula, the patient is most likely to have what done for treatment? a. Peritoneal dialysis b. Peripheral vascular access using radial artery c. Silastic catheter tunneled subcutaneously to the jugular vein d. Peripherally inserted central catheter (PICC) line inserted into subclavian vein

c. A more permanent, soft, flexible Silastic double-lumen catheter is used for long-term access when other forms of vascular access have failed. These catheters are tunneled subcutaneously and have Dacron cuffs that prevent infection from tracking along the catheter.

Priority Decision: When caring for a patient following a radical prostatectomy with a perineal approach, what is the priority nursing intervention the nurse should use to prevent complications? a. Use chemotherapeutic agents to prevent metastasis. b. Administer sildenafil (Viagra) as needed for erectile dysfunction. c. Provide wound care after each bowel movement to prevent infection. d. Insert a smaller indwelling urinary catheter to prevent urinary retention.

c. A prostatectomy performed with a perineal approach has a high risk for infection because of the proximity of the wound to the anus, so wound care is the priority. Chemotherapy is usually not the first choice of drug therapy following surgery, nor is sildenafil. The catheter size would not be changed but the catheter would be removed. Urinary incontinence is a bigger problem than retention.

What is included in nursing care that applies to the management of all urinary catheters in hospitalized patients? a. Measuring urine output every 1 to 2 hours to ensure patency b. Turning the patient frequently from side to side to promote drainage c. Using strict sterile technique during irrigation and obtaining culture specimens d. Daily cleaning of the catheter insertion site with soap and water and application of lotion

c. All urinary catheters in hospitalized patients pose a very high risk for infection, especially antibiotic-resistant, health care-associated infections, and scrupulous aseptic technique is essential in the insertion and maintenance of all catheters. Routine irrigations are not performed. Turning the patient to promote drainage is recommended only for suprapubic catheters. Cleaning the insertion site with soap and water should be performed for urethral and suprapubic catheters but lotion or powder should be avoided and site care for other catheters may require special interventions.

17. A patient is being treated with chemotherapy. The nurse revises the patient's care plan based on which result? a. WBC count 4000/μL b. RBC count 3.8 × 106/μL c. Platelets 50,000/μL d. Hematocrit (Hct) 39%

c. Any platelet count <150,000/μL is considered thrombocytopenia and could place the patient at risk for bleeding, necessitating special consideration in nursing care. Chemotherapy may cause bone marrow suppression and a depletion of all blood cells. The other factors are all within normal range.

What is the most common cause of acute pyelonephritis resulting from an ascending infection from the lower urinary tract? a. The kidney is scarred and fibrotic. b. The organism is resistant to antibiotics. c. There is a preexisting abnormality of the urinary tract. d. The patient does not take all of the antibiotics for treatment of a UTI.

c. Ascending infections from the bladder to the kidney are prevented by the normal anatomy and physiology of the urinary tract unless a preexisting condition, such as vesicoureteral reflux or lower urinary tract dysfunction (bladder tumors, prostatic hyperplasia, strictures, or stones), is present. Resistance to antibiotics and failure to take a full prescription of antibiotics for a UTI usually result in relapse or reinfection of the lower urinary tract.

What can patients at risk for renal lithiasis do to prevent the stones in many cases? a. Lead an active lifestyle b. Limit protein and acidic foods in the diet c. Drink enough fluids to produce dilute urine d. Take prophylactic antibiotics to control UTIs

c. Because crystallization of stone constituents can precipitate and unite to form a stone when in supersaturated concentrations, one of the best ways to prevent stones of any type is by drinking adequate fluids to keep the urine dilute and flowing (e.g., an output of about 2 L of urine a day). Sedentary lifestyle is a risk factor for renal stones but exercise also causes fluid loss and a need for additional fluids. Protein foods high in purine should be restricted only for the small percentage of patients with uric acid stones and although UTIs contribute to stone formation, prophylactic antibiotics are not indicated.

On admission to the ambulatory surgical center, a patient with BPH informs the nurse that he is going to have a laser treatment of his enlarged prostate. The nurse plans patient teaching with the knowledge that the patient will need to know what? a. The effects of general anesthesia b. The possibility of short-term incontinence c. Home management of an indwelling catheter d. Monitoring for postoperative urinary retention

c. Because of edema, urinary retention, and delayed sloughing of tissue that occurs with a laser prostatectomy, the patient will have postprocedure catheterization for up to 7 days. The procedure is done under local anesthetic, and incontinence or urinary retention is not usually a problem with laser prostatectomy.

Following a TURP, a patient has continuous bladder irrigation. Four hours after surgery, the catheter is draining thick, bright red clots and tissue. What should the nurse do? a. Release the traction on the catheter. b. Manually irrigate the catheter until the drainage is clear. c. Increase the rate of the irrigation and take the patient's vital signs. d. Clamp the drainage tube and notify the patient's health care provider.

c. Bleeding and blood clots from the bladder are expected after prostatectomy and continuous irrigation is used to keep clots from obstructing the urinary tract. The rate of the irrigation may be titrated to keep the clots from forming, if ordered, but the nurse should also check the vital signs because hemorrhage is the most common complication of prostatectomy. The traction on the catheter applies pressure to the operative site to control bleeding and should be relieved only if specific orders are given. The catheter does not need to be manually irrigated unless there are signs that the catheter is obstructed and clamping the drainage tube is contraindicated because it would cause distention of the bladder.

Which type of urinary tract calculi are the most common and frequently obstruct the ureter? a. Cystine b. Uric acid c. Calcium oxalate d. Calcium phosphate

c. Calcium oxalate calculi are most common and small enough to get trapped in the ureter.

When taking a nursing history from a patient with BPH, the nurse would expect the patient to report a. nocturia, dysuria, and bladder spasms. b. urinary frequency, hematuria, and perineal pain. c. urinary hesitancy, postvoid dribbling, and weak urinary stream. d. urinary urgency with a forceful urinary stream and cloudy urine.

c. Classic symptoms of uncomplicated BPH are those associated with irritative symptoms, including nocturia, frequency, urgency, dysuria, bladder pain, and incontinence associated with inflammation or infection. Urinary obstruction symptoms include diminished caliber and force of the urinary stream, hesitancy, difficulty initiating voiding, intermittent urination, dribbling at the end of urination, and a feeling of incomplete bladder emptying because of urinary retention.

A patient with AKI is a candidate for continuous renal replacement therapy (CRRT). What is the most common indication for use of CRRT? a. Azotemia b. Pericarditis c. Fluid overload d. Hyperkalemia

c. Continuous renal replacement therapy (CRRT) is indicated for the patient with AKI as an alternative or adjunct to hemodialysis to slowly remove solutes and fluid in the hemodynamically unstable patient. It is especially useful for treatment of fluid overload, but hemodialysis is indicated for treatment of hyperkalemia, pericarditis, or other serious effects of uremia.

What does the dialysate for PD routinely contain? a. Calcium in a lower concentration than in the blood b. Sodium in a higher concentration than in the blood c. Dextrose in a higher concentration than in the blood d. Electrolytes in an equal concentration to that of the blood

c. Dextrose or icodextrin or amino acid is added to dialysate fluid to create an osmotic gradient across the membrane to remove excess fluid from the blood. The dialysate fluid has no potassium so that potassium will diffuse into the dialysate from the blood. Dialysate also usually contains higher calcium to promote its movement into the blood. Dialysate sodium is usually less than or equal to that of blood to prevent sodium and fluid retention.

The mother of an 8-year-old girl has brought her child to the clinic because she is wetting the bed at night. What terminology should the nurse use when documenting this situation? a. Ascites b. Dysuria c. Enuresis d. Urgency

c. Enuresis is involuntary urination at night. Ascites is excess fluid in the intraperitoneal cavity. Dysuria is painful urination. Urgency is the feeling of needing to void immediately.

Which statement accurately describes glomerular filtration rate (GFR)? a. The primary function of GFR is to excrete nitrogenous waste products. b. Decreased permeability in the glomerulus causes loss of proteins into the urine. c. The GFR is primarily dependent on adequate blood flow and adequate hydrostatic pressure. d. The GFR is decreased with prostaglandins cause vasodilation and increased renal blood flow.

c. GFR is primarily dependent on adequate blood flow and hydrostatic pressure. The glomerulus filters the blood. The GFR is the amount of blood filtered each minute by the glomeruli, which determines the concentration of urea in the blood. Increased permeability in the glomerulus causes loss of proteins in the urine. The prostaglandins increase the GFR with increased renal blood flow.

Glomerulonephritis is characterized by glomerular damage caused by a. growth of microorganisms in the glomeruli. b. release of bacterial substances toxic to the glomeruli. c. accumulation of immune complexes in the glomeruli. d. hemolysis of red blood cells circulating in the glomeruli.

c. Glomerulonephritis is not an infection but rather an antibody-induced injury to the glomerulus, where either autoantibodies against the glomerular basement membrane (GBM) directly damage the tissue or antibodies reacting with nonglomerular antigens are randomly deposited as immune complexes along the GBM. Prior infection by bacteria or viruses may stimulate the antibody production but is not present or active at the time of glomerular damage.

18. What describes hypospadias? a. Scrotal lymphedema b. Undescended testicle c. Ventral urinary meatus d. Inflammation of the prepuce

c. Hypospadias is the urethral meatus located on the ventral surface of the penis. Scrotal lymphedema is called a hydrocele. An undescended testicle is cryptorchidism. Inflammation of the prepuce or foreskin is called phimosis.

The newly admitted patient has moderate AD. What does the nurse know this patient will need help with? a. Eating b. Walking c. Dressing d. Self-care activities

c. In the moderate stage of AD, the patient may need help with getting dressed. In the severe stage, patients will be unable to dress or feed themselves and are usually incontinent.

The patient with CKD is brought to the emergency department with Kussmaul respirations. What does the nurse know about CKD that could cause this patient's Kussmaul respirations? a. Uremic pleuritis is occurring. b. There is decreased pulmonary macrophage activity. c. They are caused by respiratory compensation for metabolic acidosis. d. Pulmonary edema from heart failure and fluid overload is occurring.

c. Kussmaul respirations occur with severe metabolic acidosis when the respiratory system is attempting to compensate by removing carbon dioxide with exhalations. Uremic pleuritis would cause a pleural friction rub. Decreased pulmonary macrophage activity increases the risk of pulmonary infection. Dyspnea would occur with pulmonary edema.

A 55-year-old man with a history of prostate cancer in his family asks the nurse what he can do to decrease the risk of prostate cancer. What should the nurse teach him about prostate cancer risks? a. Nothing can decrease the risk because prostate cancer is primarily a disease of aging. b. Treatment of any enlargement of the prostate gland will help to prevent prostate cancer. c. Substituting fresh fruits and vegetables for high-fat foods in the diet may lower the risk of prostate cancer. d. Using a natural herb, saw palmetto, has been found to be an effective protection against prostate cancer.

c. Most prostate cancers (about 75%) are considered sporadic. About the only modifiable risk factor for prostate cancer is its association with a diet high in red and processed meat and high-fat dairy products along with a low intake of vegetables and fruits. Age, ethnicity, and family history are risk factors for prostate cancer but are not modifiable. Simple enlargement or hyperplasia of the prostate is not a risk factor for prostate cancer.

11. A patient with diabetes insipidus is treated with nasal desmopressin acetate (DDAVP). The nurse determines that the drug is not having an adequate therapeutic effect when the patient experiences a. headache and weight gain. b. nasal irritation and nausea. c. a urine specific gravity of 1.002. d. an oral intake greater than urinary output.

c. Normal urine specific gravity is 1.005 to 1.025 and urine with a specific gravity of 1.002 is very dilute, indicating that there continues to be excessive loss of water and that treatment of diabetes insipidus is inadequate. Headache, weight gain, and oral intake greater than urinary output are signs of volume excess that occur with overmedication. Nasal irritation and nausea may also indicate overdosage.

To assist the patient with stress incontinence, what is the best thing the nurse should teach the patient to do? a. Void every 2 hours to prevent leakage. b. Use absorptive perineal pads to contain urine. c. Perform pelvic floor muscle exercises 40 to 50 times per day. d. Increase intraabdominal pressure during voiding to empty the bladder completely.

c. Pelvic floor exercises (Kegel exercises) increase the tone of the urethral sphincters and should be done in sets of 10 or more contractions four to five times a day (total of 40 to 50 per day). Frequent bladder emptying is recommended for patients with urge incontinence and an increase in pressure on the bladder is recommended for patients with overflow incontinence. Absorptive perineal pads should be only a temporary measure because longterm use discourages continence and can lead to skin problems.

What is a factor that contributes to an increased incidence of urinary tract infections in aging women? a. Length of the urethra b. Larger capacity of bladder c. Relaxation of pelvic floor and bladder muscles d. Tight muscular support at the urinary sphincter

c. Relaxation of female urethra, bladder, vagina, and pelvic floor muscles may contribute to stress and urge incontinence and urinary tract infections. The short urethra of women allows easier ascension and colonization of bacteria in the bladder than occurs in men and the urethra does not lengthen with age. The bladder capacity of men and women is the same but decreases with aging.

For a patient with CKD the nurse identifies a nursing diagnosis of risk for injury: fracture related to alterations in calcium and phosphorus metabolism. What is the pathologic process directly related to the increased risk for fractures? a. Loss of aluminum through the impaired kidneys b. Deposition of calcium phosphate in soft tissues of the body c. Impaired vitamin D activation resulting in decreased GI absorption of calcium d. Increased release of parathyroid hormone in response to decreased calcium levels

c. The calcium-phosphorus imbalances that occur in CKD result in hypocalcemia, from a deficiency of active vitamin D and increased phosphorus levels. This leads to an increased rate of bone remodeling with a weakened bone matrix. Aluminum accumulation is also believed to contribute to the osteomalacia. Osteitis fibrosa involves replacement of calcium in the bone with fibrous tissue and is primarily a result of elevated levels of parathyroid hormone resulting from hypocalcemia.

What results in the edema associated with nephrotic syndrome? a. Hypercoagulability b. Hyperalbuminemia c. Decreased plasma oncotic pressure d. Decreased glomerular filtration rate

c. The massive proteinuria that results from increased glomerular membrane permeability in nephrotic syndrome leaves the blood without adequate proteins (hypoalbuminemia) to create an oncotic colloidal pressure to hold fluid in the vessels. Without oncotic pressure, fluid moves into the interstitium, causing severe edema. Hypercoagulability occurs in nephrotic syndrome but is not a factor in edema formation and glomerular filtration rate (GFR) is not necessarily affected in nephrotic syndrome.

During the nursing assessment of the patient with renal insufficiency, the nurse asks the patient specifically about a history of a. angina. b. asthma. c. hypertension. d. rheumatoid arthritis.

c. The most common causes of CKD in the United States are diabetes mellitus and hypertension. The nurse should obtain information on long-term health problems that are related to kidney disease. The other disorders are not closely associated with renal disease.

Which test is most specific for renal function? a. Renal scan b. Serum creatinine c. Creatinine clearance d. Blood urea nitrogen (BUN)

c. The rate at which creatinine is cleared from the blood and eliminated in the urine approximates the GFR and is the most specific test of renal function. The renal scan is useful in showing the location, size, and shape of the kidney and general blood perfusion.

While caring for a 77-year-old woman who has a urinary catheter, the nurse monitors the patient for the development of a UTI. What clinical manifestations is the patient most likely to experience? a. Cloudy urine and fever b. Urethral burning and bloody urine c. Vague abdominal discomfort and disorientation d. Suprapubic pain and slight decline in body temperature

c. The usual classic manifestations of UTI are often absent in older adults, who tend to experience nonlocalized abdominal discomfort and cognitive impairment characterized by confusion or decreased level of consciousness rather than dysuria and suprapubic pain.

Thirty percent of patients with kidney cancer have metastasis at the time of diagnosis. Why does this occur? a. The only treatment modalities for the disease are palliative. b. Diagnostic tests are not available to detect tumors before they metastasize. c. Classic symptoms of hematuria and palpable mass do not occur until the disease is advanced. d. Early metastasis to the brain impairs the patient's ability to recognize the seriousness of symptoms.

c. There are no early characteristic symptoms of cancer of the kidney and gross hematuria, flank pain, and a palpable mass do not occur until the disease is advanced. The treatment of choice is a partial or radical nephrectomy, which can be successful in early disease. Many kidney cancers are diagnosed as incidental imaging findings. Targeted therapy is the preferred treatment for metastatic kidney cancer. Radiation is palliative. The most common sites of metastases are the lungs, liver, and long bones.

What accurately describes a normal physical assessment of the urinary system by the nurse? a. Auscultates the lower abdominal quadrants for fluid sounds b. Palpates an empty bladder at the level of the symphysis pubis c. Percusses the kidney with a firm blow at the posterior costovertebral angle d. Positions the patient prone to palpate the kidneys with a posterior approach

c. To assess for kidney tenderness, the nurse strikes the fist of one hand over the dorsum of the other hand at the posterior costovertebral angle. The upper abdominal quadrants and costovertebral angles are auscultated for vascular bruits in the renal vessels and aorta and an empty bladder is not palpable. The kidneys are palpated through the abdomen, with the patient supine.

A woman with no history of UTIs who is experiencing urgency, frequency, and dysuria comes to the clinic, where a dipstick and microscopic urinalysis indicate bacteriuria. What should the nurse anticipate for this patient? a. Obtaining a clean-catch midstream urine specimen for culture and sensitivity b. No treatment with medication unless she develops fever, chills, and flank pain c. Empirical treatment with trimethoprim-sulfamethoxazole (TMP-SMX, Bactrim) for 3 days d. Need to have a blood specimen drawn for a complete blood count (CBC) and kidney function tests

c. Unless a patient has a history of recurrent UTIs or a complicated UTI, trimethoprim-sulfamethoxazole (TMPSMX) or nitrofurantoin (Macrodantin) is usually used to empirically treat an initial UTI without a culture and sensitivity or other testing. Asymptomatic bacteriuria does not justify treatment but symptomatic UTIs should always be treated.

The nurse teaches the patient having a vasectomy that what occurs after the procedure? a. The amount of ejaculate will be noticeably decreased. b. He may have difficulty maintaining an erection for several months. c. An alternative form of contraception must be used for 6 to 8 weeks. d. The testes will gradually decrease production of sperm and testosterone.

c. Until sperm distal to the anastomotic site is ejaculated or absorbed by the body, the semen will contain sperm and alternative contraceptive methods must be used. When a postoperative semen examination reveals no sperm, the patient is considered sterile. Following vasectomy, there is rarely noticeable difference in the amount of ejaculate because ejaculate is primarily seminal fluid. Vasectomy does not cause erectile dysfunction, nor does it affect testicular production of sperm or hormones.

What causes the gastrointestinal (GI) manifestation of stomatitis in the patient with CKD? a. High serum sodium levels b. Irritation of the GI tract from creatinine c. Increased ammonia from bacterial breakdown of urea d. Iron salts, calcium-containing phosphate binders, and limited fluid intake

c. Uremic fetor, or the urine odor of the breath, is caused by high urea content in the blood. Increased ammonia from bacterial breakdown of urea leads to stomatitis and mucosal ulcerations. Irritation of the gastrointestinal (GI) tract from urea in CKD contributes to anorexia, nausea, and vomiting. Ingestion of iron salts and calcium-containing phosphate binders, limited fluid intake, and limited activity cause constipation.

The extent of urinary obstruction caused by BPH can be determined by which diagnostic study? a. A cystometrogram b. Transrectal ultrasound c. Urodynamic flow studies d. Postvoiding catheterization

c. Urinary flow meters are used to measure the urinary flow rate, which is slowed with increased obstruction. Cystourethroscopy may also evaluate the degree of obstruction but a cystometrogram measures bladder tone. A transrectal ultrasound may determine the size and configuration of the prostate gland. Postvoiding catheterization measures residual urine.

In planning care for the patient with Crohn's disease, the nurse recognizes that a major difference between ulcerative colitis and Cron's is that Crohn's disease a. frequently results in toxic megacolon. b. causes fewer nutritional deficiencies than does ulcerative colitis. c. often recurs after surgery, whereas ulcerative colitis is curable with a colectomy. d. Is manifested by rectal bleeding and anemia more frequently than is ulcerative colitis.

c. often recurs after surgery, whereas ulcerative colitis is curable with a colectomy.

While obtaining subjective assessment data related to the musculoskeletal system, it is particularly important to ask a patient about other medical problems such as a.hypertension. b.thyroid problems. c.diabetes mellitus. d.chronic bronchitis.

c.diabetes mellitus The nurse should question the patient about past medical problems because certain illnesses are known to affect the musculoskeletal system directly or indirectly. These diseases include tuberculosis, poliomyelitis, diabetes mellitus, parathyroid problems, hemophilia, rickets, soft tissue infection, and neuromuscular disabilities.

The bone cells that function in the resorption of bone tissue are called a.osteoids b.osteocytes c.osteoclasts d.osteoblasts

c.osteoclasts Osteoclasts participate in bone remodeling by assisting in the breakdown of bone tissue.

A patient with cholelithiasis needs to have the gallbladder removed. Which patient assessment is a contraindication for a cholecystectomy? Low-grade fever of 100° F and dehydration Abscess in the right upper quadrant of the abdomen Activated partial thromboplastin time (aPTT) of 54 seconds

ctivated partial thromboplastin time (aPTT) of 54 seconds Multiple obstructions in the cystic and common bile duct An aPTT of 54 seconds is above normal and indicates insufficient clotting ability. If the patient had surgery, significant bleeding complications postoperatively are very likely. Fluids can be given to eliminate the dehydration; the abscess can be assessed, and the obstructions in the cystic and common bile duct would be relieved with the cholecystectomy.

The most important intervention for the patient with epidemic keratoconjunctivitis is a. cleansing the affected area with baby shampoo. b. monitoring spread of infection to the opposing eye. c. regular instillation of artificial tears to the affected eye. d. teaching the patient and family members good hygiene techniques.

d Rationale: Epidemic keratoconjunctivitis (EKC) is the most serious ocular adenoviral disease. EKC is spread by direct contact, including sexual activity. The nurse should teach the patient and caregiver the importance of good hygienic practices to avoid spreading the disease.

The nursing management of the patient with cholecystitis associated with cholelithiasis is based on the knowledge that a. shock-wave therapy should be tried initially. b. once gallstones are removed, they tend not to recur. c. the disorder can be successfully treated with oral bile salts that dissolve gallstones. d. laparoscopic cholecystectomy is the treatment of choice in most patients who are symptomatic. (Lewis 1042)

d Rationale: Laparoscopic cholecystectomy is the treatment of choice for symptomatic cholelithiasis.

What is important for the nurse to include in the postoperative care of the patient following tympanoplasty? a. Check the gag reflex. b. Encourage independence. c. Avoid changing the cotton padding. d. Instruct patient to refrain from forceful nose blowing.

d Rationale: Sudden pressure changes in the ear and postoperative infections can disrupt the surgical repair during the healing phase or cause facial nerve paralysis.

Patients with permanent visual impairment a. feel most comfortable with other visually impaired people. b. may feel threatened when others make eye contact during a conversation. c. usually need others to speak louder so they can communicate appropriately. d. may experience the same grieving process that is associated with other losses.

d Rationale: When the patient has lost visual function or even the entire eye, he or she will grieve the loss. The nurse should help the patient through the grieving process.

What are intrarenal causes of acute kidney injury (AKI) (select all that apply)? a. Anaphylaxis b. Renal stones c. Bladder cancer d. Nephrotoxic drugs e. Acute glomerulonephritis f. Tubular obstruction by myoglobin

d, e, f. Intrarenal causes of acute kidney injury (AKI) include conditions that cause direct damage to the kidney tissue, including nephrotoxic drugs, acute glomerulonephritis, and tubular obstruction by myoglobin, or prolonged ischemia. Anaphylaxis and other prerenal problems are frequently the initial cause of AKI. Renal stones and bladder cancer are among the postrenal causes of AKI.

Which characteristics are associated with urge incontinence (select all that apply)? a. Treated with Kegel exercises b. Found following prostatectomy c. Common in postmenopausal women d. Involuntary urination preceded by urgency e. Caused by overactivity of the detrusor muscle f. Bladder contracts by reflex, overriding central inhibition

d, e, f. Urge incontinence is involuntary urination preceded by urgency caused by overactivity of the detrusor muscle when the bladder contracts by reflex, which overrides central inhibition. Treatment includes treating the underlying cause and retraining the bladder with urge suppression, anticholinergic drugs, or containment devices. The other options are characteristic of stress incontinence. Patients may have a combination of urge and stress incontinence.

Increased intraocular pressure may occur as a result of a. edema of the corneal stroma. b. dilation of the retinal arterioles. c. blockage of the lacrimal canals and ducts. d. increased production of aqueous humor by the ciliary process.

d. Rationale: Excess aqueous humor production or decreased outflow can elevate intraocular pressure above the normal 10 to 21 mm Hg; this condition is called glaucoma.

A patient suspected of having acromegaly has an elevated plasma growth hormone (GH) level. In acromegaly, what would the nurse also expect the patient's diagnostic results to indicate? a. Hyperinsulinemia b. Plasma glucose of <70 mg/dL (3.9 mmol/L) c. Decreased GH levels with an oral glucose challenge test d. Elevated levels of plasma insulin-like growth factor-1 (IGF-1)

d. A normal response to growth hormone (GH) secretion is stimulation of the liver to produce somatomedin C, or insulin-like growth factor-1 (IGF-1), which stimulates growth of bones and soft tissues. The increased levels of somatomedin C normally inhibit GH but in acromegaly the pituitary gland secretes GH despite elevated IGF-1 levels. When both GH and IGF-1 levels are increased, overproduction of GH is confirmed. GH also causes elevation of blood glucose and normally GH levels fall during an oral glucose challenge but not in acromegaly.

Priority Decision: What is the most appropriate snack for the nurse to offer a patient with stage 4 CKD? a. Raisins b. Ice cream c. Dill pickles d. Hard candy

d. A patient with CKD may have unlimited intake of sugars and starches (unless the patient is diabetic) and hard candy is an appropriate snack and may help to relieve the metallic and urine taste that is common in the mouth. Raisins are a high-potassium food. Ice cream contains protein and phosphate and counts as fluid. Pickled foods have high sodium content.

Which test is required for a diagnosis of pyelonephritis? a. Renal biopsy b. Blood culture c. Intravenous pyelogram (IVP) d. Urine for culture and sensitivity

d. A urine specimen specifically obtained for culture and sensitivity is required to diagnose pyelonephritis because it will show pyuria, the specific bacteriuria, and what drug the bacteria is sensitive to for treatment. The renal biopsy is used to diagnose chronic pyelonephritis or cancer. Blood cultures would be done if bacteremia is suspected. Intravenous pyelogram (IVP) would increase renal irritation, but CT urograms may be used to assess for signs of infection in the kidney and complications of pyelonephritis.

Acute tubular necrosis (ATN) is the most common cause of intrarenal AKI. Which patient is most likely to develop ATN? a. Patient with diabetes mellitus b. Patient with hypertensive crisis c. Patient who tried to overdose on acetaminophen d. Patient with major surgery who required a blood transfusion

d. Acute tubular necrosis (ATN) is primarily the result of ischemia, nephrotoxins, or sepsis. Major surgery is most likely to cause severe kidney ischemia in the patient requiring a blood transfusion. A blood transfusion hemolytic reaction produces nephrotoxic injury if it occurs. Diabetes mellitus, hypertension, and acetaminophen overdose will not contribute to ATN.

A 72-year-old man tells the nurse that he cannot perform most of the physical activities he could do 5 years ago because of overall joint aches and pains. What can the nurse do to assist the patient to prevent further deconditioning and decrease the risk for developing musculoskeletal problems? a. Limit weight-bearing exercise to prevent stress on fragile bones and possible hip fractures. b. Advise the patient to avoid the use of canes and walkers because they increase dependence on ambulation aids. c. Advise the patient to increase his activity by more frequently climbing stairs in buildings and other environments with steps. d. Discuss use of over-the-counter (OTC) medications to decrease inflammation and pain so that exercise can be maintained.

d. Almost all older adults have some degree of decreased muscle strength, joint stiffness, and pain with motion. The use of mild antiinflammatory agents decreases inflammation and pain and can help the patient to maintain activity and prevent further deconditioning but other prescribed drugs and potential abdominal problems must be considered by the patient. Musculoskeletal problems in the older adult can be prevented with appropriate strategies, especially exercise. Stair walking can create enough stress on fragile bones to cause a hip fracture and use of ramps may help to prevent falls. Walkers and canes should be used as necessary to decrease stress on joints so that activity can be maintained.

What manifestation in the patient will indicate the need for restriction of dietary protein in management of acute poststreptococcal glomerulonephritis (APSGN)? a. Hematuria b. Proteinuria c. Hypertension d. Elevated blood urea nitrogen (BUN)

d. An elevated blood urea nitrogen (BUN) indicates that the kidneys are not clearing nitrogenous wastes from the blood and protein may be restricted until the kidney recovers. Proteinuria indicates loss of protein from the blood and possibly a need for increased protein intake. Hypertension is treated with sodium and fluid restriction, diuretics, and antihypertensive drugs. The hematuria is not specifically treated.

Which serum laboratory value indicates to the nurse that the patient's CKD is getting worse? a. Decreased BUN b. Decreased sodium c. Decreased creatinine d. Decreased calculated glomerular filtration rate (GFR)

d. As GFR decreases, BUN and serum creatinine levels increase. Although elevated BUN and creatinine indicate that waste products are accumulating, the calculated GFR is considered a more accurate indicator of kidney function than BUN or serum creatinine.

When caring for the patient with interstitial cystitis, what can the nurse teach the patient to do? a. Avoid foods that make the urine more alkaline. b. Use high-potency vitamin therapy to decrease the autoimmune effects of the disorder. c. Always keep a voiding diary to document pain, voiding frequency, and patterns of nocturia. d. Use the dietary supplement calcium glycerophosphate (Prelief) to decrease bladder irritation.

d. Calcium glycerophosphate (Prelief) alkalinizes the urine and can help to relieve the irritation from acidic foods. A diet low in acidic foods is recommended and if a multivitamin is used, high-potency vitamins should be avoided because these products may irritate the bladder. A voiding diary is useful in diagnosis but does not need to be kept indefinitely.

A 78-year-old man asks the nurse why he has to urinate so much at night. The nurse should explain to the patient that as an older adult, what may contribute to his nocturia? a. Decreased renal mass b. Decreased detrusor muscle tone c. Decreased ability to conserve sodium d. Decreased ability to concentrate urine

d. Decreased renal blood flow and altered hormone levels result in a decreased ability to concentrate urine that results in an increased volume of dilute urine, which does not maintain the usual diurnal elimination pattern. A decrease in bladder capacity also contributes to nocturia but decreased bladder muscle tone results in urinary retention. Decreased renal mass decreases renal reserve but function is generally adequate under normal circumstances.

Priority Decision: After a patient had a renal arteriogram and is back on the clinical unit, what is the most important action by the nurse? a. Observe for gross bleeding in the urine. b. Place the patient in high Fowler's position. c. Monitor the patient for signs of allergy to the contrast medium. d. Assess peripheral pulses in the involved leg every 30 to 60 minutes.

d. During a renal arteriogram, a catheter is inserted, most commonly at the femoral artery. Following the procedure the patient is positioned with the affected leg extended with a pressure dressing applied. Peripheral pulse monitoring is essential to detect the development of thrombi around the insertion site, which may occlude blood supply to the leg. Gross bleeding in the urine is a complication of a renal biopsy. Allergy to the contrast medium should be established before the procedure.

A patient rapidly progressing toward end-stage kidney disease asks about the possibility of a kidney transplant. In responding to the patient, the nurse knows that what is a contraindication to kidney transplantation? a. Hepatitis C infection b. Coronary artery disease c. Refractory hypertension d. Extensive vascular disease

d. Extensive vascular disease is a contraindication for renal transplantation, primarily because adequate blood supply is essential for the health of the new kidney. Other contraindications include disseminated malignancies, refractory or untreated cardiac disease, chronic respiratory failure, chronic infection, or unresolved psychosocial disorders. Coronary artery disease (CAD) may be treated with bypass surgery before transplantation and transplantation can relieve hypertension. Hepatitis B or C infection is not a contraindication.

While caring for the patient in the oliguric phase of AKI, the nurse monitors the patient for associated collaborative problems. When should the nurse notify the health care provider? a. Urine output is 300 mL/day. b. Edema occurs in the feet, legs, and sacral area. c. Cardiac monitor reveals a depressed T wave and elevated ST segment. d. The patient experiences increasing muscle weakness and abdominal cramping.

d. Hyperkalemia is a potentially life-threatening complication of AKI in the oliguric phase. Muscle weakness and abdominal cramping are signs of the neuromuscular impairment that occurs with hyperkalemia. In addition, hyperkalemia can cause the cardiac conduction abnormalities of peaked T wave, prolonged PR interval, prolonged QRS interval, and depressed ST segment. Urine output of 300 mL/day is expected during the oliguric phase, as is the development of peripheral edema.

A patient asks the nurse what the difference is between benign prostatic hyperplasia (BPH) and prostate cancer. Thebest response by the nurse includes what information about BPH? a. BPH is a benign tumor that does not spread beyond the prostate gland. b. BPH is a precursor to prostate cancer but does not yet show any malignant changes. c. BPH is an enlargement of the gland caused by an increase in the size of existing cells. d. BPH is a benign enlargement of the gland caused by an increase in the number of normal cells.

d. Hyperplasia is an increase in the number of cells and in benign prostatic hyperplasia (BPH), it is thought that the enlargement caused by the increase in new cells results from hormonal changes associated with aging. Hypertrophy refers to an increase in the size of existing cells. The hyperplasia is not considered a tumor, nor has BPH been proven to predispose to cancer of the prostate.

What indicates to the nurse that a patient with oliguria has prerenal oliguria? a. Urine testing reveals a low specific gravity. b. Causative factor is malignant hypertension. c. Urine testing reveals a high sodium concentration. d. Reversal of oliguria occurs with fluid replacement.

d. In prerenal oliguria, the oliguria is caused by a decrease in circulating blood volume and there is no damage yet to the renal tissue. It can be reversed by correcting the precipitating factor, such as fluid replacement for hypovolemia. Prerenal oliguria is characterized by urine with a high specific gravity and a low sodium concentration, whereas oliguria of intrarenal failure is characterized by urine with a low specific gravity and a high sodium concentration. Malignant hypertension causes damage to renal tissue and intrarenal oliguria.

What accurately describes the care of the patient with CKD? a. A nutrient that is commonly supplemented for the patient on dialysis because it is dialyzable is iron. b. The syndrome that includes all of the signs and symptoms seen in the various body systems in CKD is azotemia. c. The use of morphine is contraindicated in the patient with CKD because accumulation of its metabolites may cause seizures. d. The use of calcium-based phosphate binders in the patient with CKD is contraindicated when serum calcium levels are increased.

d. In the patient with CKD, when serum calcium levels are increased, calcium-based phosphate binders are not used. The nutrient supplemented for patients on dialysis is folic acid. The various body system manifestations occur with uremia, which includes azotemia. Meperidine is contraindicated in patients with CKD related to possible seizures.

A 66-year-old male patient is experiencing erectile dysfunction (ED). He and his wife have used tadalafil (Cialis) but because he experienced priapism, they have decided to change their treatment option to an intraurethral device. How should the nurse explain how this device works? a. The device relaxes smooth muscle in the penis. b. Blood is drawn into corporeal bodies and held with a ring. c. The device is implanted into corporeal bodies to firm the penis. d. The device directly applies drugs that increase blood flow in the penis

d. Intraurethral devices include the use of vasoactive drugs administered as a topical gel, an injection into the penis (intracavernosal self-injection), or a medication pellet (alprostadil) inserted into the urethra (intraurethral) using a medicated urethral system for erection (MUSE) device. A medication pellet inserted into the urethra using a MUSE device, a topical gel, or the intracavernosal self-injection of vasoactive drugs may be used for erectile dysfunction. The vasoactive drugs enhance blood flow into the penile arteries for erection. Erectogenic drugs (e.g., tadalafil [Cialis]) cause smooth muscle relaxation and increase blood flow to promote an erection. Blood drawn into corporeal bodies and held with a ring is achieved with a vacuum constriction device (VCD). Devices implanted into corporeal bodies to firm the penis are penile implants. Androgen or testosterone replacement therapy may also be used for erectile dysfunction.

During the assessment of a patient with acute abdominal pain, the nurse should a. perform deep palpation before auscultation. b. obtain blood pressure and pulse rate to determine hypervolemic changes.. c. auscultate bowel sounds because hyperactive bowel sounds suggest paralytic ileus. d. Measure body temperature because an elevated temperature may indicate an inflammatory or infectious process.

d. Measure body temperature because an elevated temperature may indicate an inflammatory or infectious process.

In caring for the patient with AKI, what should the nurse be aware of? a. The most common cause of death in AKI is irreversible metabolic acidosis. b. During the oliguric phase of AKI, daily fluid intake is limited to 1000 mL plus the prior day's measured fluid loss. c. Dietary sodium and potassium during the oliguric phase of AKI are managed according to the patient's urinary output. d. One of the most important nursing measures in managing fluid balance in the patient with AKI is taking accurate daily weights.

d. Measuring daily weights with the same scale at the same time each day allows for the evaluation and detection of excessive body fluid gains or losses. Infection is the leading cause of death in AKI, so meticulous aseptic technique is critical. The fluid limitation in the oliguric phase is 600 mL plus the prior day's measured fluid loss. Dietary sodium and potassium intake are managed according to the plasma levels.

Prevention of calcium oxalate stones would include dietary restriction of which foods or drinks? a. Milk and milk products b. Dried beans and dried fruits c. Liver, kidney, and sweetbreads d. Spinach, cabbage, and tomatoes

d. Oxalate-rich foods should be limited to reduce oxalate excretion. Foods high in oxalate include spinach, rhubarb, asparagus, cabbage, and tomatoes, in addition to chocolate, coffee, and cocoa. Currently, it is believed that high dietary calcium intake may actually lower the risk for renal stones by reducing the intestinal oxalate absorption and therefore the urinary excretion of oxalate. Milk, milk products, dried beans, and dried fruits are high sources of calcium. Organ meats are high in purine, which contributes to uric acid lithiasis.

What is an explanation that the nurse should give to the patient who asks what his diagnosis of paraphimosis means? a. Painful, prolonged erection b. Inflammation of the epididymis c. Painful downward curvature of an erect penis d. Retracted tight foreskin preventing return over the glans

d. Paraphimosis is tightness of the foreskin and the inability to pull it forward from a retracted position to return it over the glans. It is usually associated with poor hygiene techniques. Painful, prolonged erection is priapism. Epididymitis is inflammation of the epididymis. A painful downward curvature of an erect penis is chordee.

The patient complains of "wetting when she sneezes." How should the nurse document this information? a. Nocturia b. Micturition c. Urge incontinence d. Stress incontinence

d. Stress incontinence is involuntary urination with increased pressure when sneezing or coughing and is seen with weakness of sphincter control. Nocturia is frequent urination at night. Micturition is the evacuation of urine. Urge incontinence is involuntary urination is preceded by urinary urgency.

Which disease causes connective tissue changes that cause glomerulonephritis? a. Gout b. Amyloidosis c. Diabetes mellitus d. Systemic lupus erythematosus

d. Systemic lupus erythematosus causes connective tissue damage that affects the glomerulus. Gout deposits uric acid crystals in the kidney. Amyloidosis deposits hyaline bodies in the kidney. Diabetes mellitus causes microvascular damage affecting the kidney.

When teaching a patient testicular self-examination, the nurse instructs the patient to report which finding? a. An irregular-feeling epididymis b. One testis larger than the other c. The spermatic cord within the testicle d. A firm, nontender nodule on the testis

d. Testicular tumors most often present on the testis as a lump or nodule that is very firm, is nontender, and cannot be transilluminated. There may also be scrotal swelling and a feeling of heaviness. All of the other options are normal findings.

What indicates to the nurse that a patient with AKI is in the recovery phase? a. A return to normal weight b. A urine output of 3700 mL/day c. Decreasing sodium and potassium levels d. Decreasing blood urea nitrogen (BUN) and creatinine levels

d. The blood urea nitrogen (BUN) and creatinine levels remain high during the oliguric and diuretic phases of AKI. The recovery phase begins when the glomerular filtration returns to a rate at which BUN and creatinine stabilize and then decrease. Urinary output of 3 to 5 L/ day, decreasing sodium and potassium levels, and fluid weight loss are characteristic of the diuretic phase of AKI.

The cremasteric reflex is absent in which problem of the scrotum and testes? a. Hydrocele b. Varicocele c. Spermatocele d. Testicular torsion

d. The cremasteric reflex is elicited by light stroking of the inner aspect of the thigh in a downward direction with a tongue blade. In testicular torsion, or a twisted spermatic cord that supplies blood to the testes and epididymis, this reflex is absent on the swollen side. Varicocele is dilation of the veins that drain the testes. Hydrocele is scrotal lymphedema from interference with lymphatic drainage of the scrotum. Spermatocele is a sperm-containing cyst of the epididymis.

What is the most common screening intervention for detecting BPH in men over age 50? a. PSA level b. Urinalysis c. Cystoscopy d. Digital rectal examination

d. The prostate gland can be easily palpated by rectal examination and enlargement of the gland is detected early if yearly examinations are performed. If symptoms of prostatic hyperplasia are present, further diagnostic testing, including a urinalysis, prostate-specific antigen (PSA), and cystoscopy, may be indicated.

What occurs with chronic bacterial prostatitis but not with acute prostatitis? a. Postejaculatory pain b. Frequency, urgency, and dysuria c. Symptoms of a urinary tract infection d. Enlarged, boggy prostate on palpation

d. The prostate with chronic bacterial prostatitis feels enlarged and firm, often described as boggy, and is tender. The other options are true of both chronic and acute prostatitis.

A patient with suprapubic pain and symptoms of urinary frequency and urgency has two negative urine cultures. What is one assessment finding that would indicate interstitial cystitis? a. Residual urine greater than 200 mL b. A large, atonic bladder on urodynamic testing c. A voiding pattern that indicates psychogenic urinary retention d. Pain with bladder filling that is transiently relieved by urination

d. The symptoms of interstitial cystitis (IC) imitate those of an infection of the bladder but the urine is free of infectious agents. Unlike a bladder infection, the pain with IC increases as urine collects in the bladder and is temporarily relieved by urination. Acidic urine is very irritating to the bladder in IC and the bladder is small but urinary retention is not common.

A patient with bladder cancer undergoes cystectomy with formation of an ileal conduit. During the patient's first postoperative day, what should the nurse plan to do? a. Measure and fit the stoma for a permanent appliance. b. Encourage high oral intake to flush mucus from the conduit. c. Teach the patient to self-catheterize the stoma every 4 to 6 hours. d. Empty the drainage bag every 2 to 3 hours and measure the urinary output.

d. Urine drains continuously from an ileal conduit and the drainage bag must be emptied every 2 to 3 hours and measured to ensure adequate urinary output. Fitting for a permanent appliance is not done until the stoma shrinks to its normal size in a few weeks. With an ileal conduit, mucus is present in the urine because it is secreted by the ileal segment as a result of the irritating effect of the urine but the surgery causes paralytic ileus and the patient will be NPO for several days postoperatively. Self-catheterization is performed when patients have formation of a continent Kock pouch.

The couple has not been able to become pregnant. The wife has not been diagnosed with any infertility problems. Which treatment will the nurse expect to teach the couple about if the problem is the most common testicular problem causing male infertility? a. Antibiotics b. Semen analysis c. Avoidance of scrotal heat d. Surgery to correct the problem

d. Varicocele is the most common testicular cause of infertility. Surgical ligation of the spermatic vein is done to correct the problem. Antibiotics are used if there is an infection but this is not as common as a varicocele. Semen analysis is the first study done when investigating male infertility but it is not a treatment. Avoidance of scrotal heat is a lifestyle change that may be used with idiopathic infertility.

When grading muscle strength, the nurse records a score of 3, which indicates a.no detection of muscular contraction. b.a barely detectable flicker of contraction. c.active movement against full resistance without fatigue. d.active movement against gravity but not against resistance.

d. active movement against gravity but not against resistance Muscle strength score of 3 indicates active movement only against gravity and not against resistance

A nursing intervention that is most appropriate to decrease post operative edema and pain following an inguinal herniorrhaphy is a. applying a truss to the hernia site. b. allowing the patient to stand to void. c. supporting the incision during coughing d. applying a scrotal support with ice bag.

d. applying a scrotal support with ice bag.

Which of the following should a patient be taught after a hemorrhoidectomy? a. Take mineral oil prior to bedtime b. eat a low-fiber diet to rest the colon c. administer oil-retention enema to empty the colon. d. Use prescribed pain medication before a bowel movement.

d. use prescribed pain meds before a bowel movement.

To prevent muscle atrophy, the nurse teaches the patient with a leg immobilized in traction to perform (select all that apply) a.flexion contractions. b.tetanic contractions. c.isotonic contractions. d.isometric contractions. e.extension contractions.

d.isometric contractions Isometric contractions increase the tension within a muscle but do not produce movement. Repeated isometric contractions make muscles grow larger and stronger. Muscular atrophy (i.e., decrease in size) occurs with the absence of contraction that results from immobility.

4. The nurse's responsibility for a patient with a suspected disc herniation who is experiencing acute pain and muscle spasms is a.encouraging total bed rest for several days. b.teaching the principles of back strengthening exercises. c.stressing the importance of straight-leg raises to decrease pain. d.promoting the use of cold and hot compresses and pain medication.

d.promoting the use of cold and hot compresses and pain medication. If the acute muscle spasms and accompanying pain are not severe and debilitating, the patient may be treated on an outpatient basis with nonsteroidal antiinflammatory drugs (NSAIDs; e.g., acetaminophen) and muscle relaxants (e.g., cyclobenzaprine [Flexeril]). Massage and back manipulation, acupuncture, and the application of cold and hot compresses may help some patients. Severe pain may necessitate a brief course of opioid analgesics. A brief period (1 to 2 days) of rest at home may be necessary for some people; most patients do better with a continuation of their regular activities. Prolonged bed rest should be avoided. All patients during this time should refrain from activities that aggravate the pain, including lifting, bending, twisting, and prolonged sitting.

Which characteristics describe transurethral resection of the prostate (TURP) (select all that apply)? a. Best used for a very large prostate gland b. Inappropriate for men with rectal problems c. Involves an external incision prostatectomy d. Uses transurethral incisions into the prostate e. Most common surgical procedure to treat BPH f. Resectoscopic excision and cauterization of prostate tissue

e, f. The transurethral resection of the prostate (TURP) is the most common surgical procedure to treat BPH and uses a resectoscopic excision and cauterization of prostate tissue. A simple open prostatectomy is used for a large prostate and has an external incision. Transurethral incision into the prostate to expand the urethra for a small to moderate-sized prostate is done with a transurethral incision of the prostate (TUIP).

Delegation Decision: Which nursing interventions could be delegated to unlicensed assistive personnel (UAP) (select all that apply)? a. Assess the need for catheterization. b. Use bladder scanner to estimate residual urine. c. Teach patient pelvic floor muscle (Kegel) exercises. d. Insert indwelling catheter for uncomplicated patient. e. Assist incontinent patient to commode at regular intervals. f. Provide perineal care with soap and water around a urinary catheter.

e, f. The unlicensed assistive personnel (UAP) may assist the incontinent patient to void at regular intervals and provide perineal care. An RN should perform the assessments and teaching. In long-term care and rehabilitation facilities, UAP may use bladder scanners after they are trained.

Dermatologic manifestation(s) of Addison's disease can include (select all that apply):

loss of body hair generalized hyperpigmentation

During the assessment of a patient, you note an area of red, sharply defined plaques covered with silvery scales that are mildly itchy on the patient's knees and elbows. you recognize this finding as:

psoriasis

In teaching a patient who is using topical corticosteroids to treat acute dermatitis, the nurse whould tell the patient that (select all that apply)

short-term use of topical corticosteroids usually does not cause systemic side effects abruptly discontinuing the use of topical corticosteroids may cause a reappearance of the dermatitis

A patient with acne vulgaris tells the nurse that she has quit her job as a receptionist because she believes her facial appearance is unattractive to customers. Thenursing diagnosis that best describes this patient is:

social isolation related to decreased activities secondary to fear of rejection

In teaching a patient with malignant melanoma about this disorder, the nurse recognizes that the patient's prognosis is most dependent on:

the thickness of the lesion

Which safe sun practices would the nurse include in the teaching care plan for a patient who has photosensitivity (select all that apply)?

wear protective clothing apply sunscreen liberally and often avoid exposure to the sun, especially during themidday


Set pelajaran terkait

AMH 2020 Ch.22,23,24 Study Guide

View Set

VISUAL AIDS (Chapter 13 Comm Quiz)

View Set

FoF Attempt #1 Wrong answers and helpful information

View Set

Microeconomics Chapter 5: Elascity

View Set

Conceptual Physics Ch 28 Light Waves

View Set